Pretest Fam med Qs

Réussis tes devoirs et examens dès maintenant avec Quizwiz!

A 44-year-old man is seeing you for a routine diabetic check. He was diagnosed with type 2 diabetes 2 years ago. He is worried because his grandmother went blind as a complication from her diabetes. Which of the following statements about diabetic retinopathy is true? a. The risk of retinopathy increases with increased hemoglobin A 1C levels. b. It generally takes 10 to 20 years to see signs of retinopathy in a diabetic patient. c. A daily aspirin decreases the risk of retinopathy development. d. The first sign of retinopathy is usually the growth of new vessels on the retina. e. Retinopathy is an uncommon cause of visual loss in this day and age

A A. The risk of retinopathy increases with increased hemoglobin A 1C levels Diabetic retinopathy is the leading cause of blindness in the United States. The risk increases with the length of time that the patient has had diabetes, and the condition worsens with increasing hemoglobin A 1C levels. In type 2 diabetics, it can be seen at diagnosis. Aspirin has no effect on eye complications. It follows a predictable pattern, with mild background abnormalities followed by increased vascular permeability and hemorrhage. Proliferative changes occur late in the course.

You are caring for a patient with a poor lipid profile. His HDL is low, his LDL is high, and his triglycerides are also high. Which of the following medications would have the most beneficial effect on his LDL? a. Lovastatin b. Colestipol c. Ezetimibe d. Fenofibrate e. Cholestyramine

A

373. You are seeing a 19-year-old college student complaining of recurrent and persistent cough. She has been treated for "bronchitis" several times, and you are concerned that her true diagnosis is asthma. Which of the following is most important in the diagnosis of asthma? a. History b. Allergy testing c. Chest x-ray d. Pulmonary function tests with and without bronchodilator therapy e. Provocative testing with methocholine

A. history of asthma

You are caring for a 32-year-old woman who is concerned about her sex life. She reports very little desire for sexual activity, and it's starting to interfere with her relationship. Upon questioning, she describes no sexual fantasies. Which of the following best describes her situation? a. She has hypoactive sexual desire disorder. b. She has sexual aversion disorder. c. She has sexual arousal disorder. d. She has dyspareunia. e. She has androgen deficiency.

A. hypoactive sexual desire disorder

A 26-year-old woman presents with weight gain, lethargy, dry skin, sweatiness, cold intolerance, and thinning hair. You suspect hypothyroidism and order the appropriate laboratory tests. Her TSH is high, and her free T 3 and free T 4 are both low. Which of the following is the most likely diagnosis? a. Primary hypothyroidism b. Secondary hypothyroidism c. Iodine deficiency d. Thyroid hormone resistance e. Subclinical hypothyroidism

A. primary hyper Primary hypothyroidism is common, usually a result of Hashimoto thyroiditis or after Graves disease. In this case, the TSH would be elevated, and the free T 3 and T 4 would be low. Secondary hypothyroidism is related to hypothalamic or pituitary dysfunction. Iodine deficiency is a cause of primary hypothyroidism. Subclinical hypothyroidism is when the TSH is elevated, but the T 3 and T 4 are normal. Thyroid resistance would present with the TSH, T 3, and T4 all being elevated.

A 22-year-old man is seeing you to discuss his low back pain. He is athletic and exercises regularly. He denies any inciting event, does not have pain with movement, and denies radiation of the pain. Given this information, which of the following is the most likely diagnosis? a. Spondylolisthesis b. Low back strain c. Degenerative osteoarthritis d. Lumbar disk herniation e. Neoplasm

A.Spondylolisthesis MCC of back pain in dudes less than 26 years old if it were a lumbar strain, there would be an event that started it

You are discussing varicella-zoster vaccination with a 24-year-old US-born male health care worker who does not ever remember having chicken pox. He does not have any contraindications to vaccine administration. Which of the following statements is true?

After vaccination, he may develop a varicella infection, but when it occurs, it does not appear to be contagious.

You are caring for a 45-year-old obese man who is complaining of poor sleep. He reports that he can fall asleep relatively quickly, but wakes up hours later unable to return to sleep for the rest of the night. He has stopped using caffeine, but this has not improved his symptoms. His medications include propranolol, hydrochlorothiazide, and naproxen as needed. He is a smoker, and drinks two to three glasses of wine nightly after work. Which of the following is the most likely reason for his sleep problems? a. Obesity b. Propranolol c. Hydrochlorothiazide d. Naproxen e. Alcohol

Alcohol

You did screening cholesterol tests on a 35-year-old man and found his results to be: Total cholesterol: 220 mg/dL (H) LDL: 125 mg/dL (H) HDL: 34 mg/dL (L) Triglycerides: 307 mg/dL (H) C-reactive protein: 2.4 mg/dL (H) Which of his laboratory results is the best predictor of an adverse outcome in this patient? a. Total cholesterol b. LDL c. HDL d. Triglycerides e. C-reactive protein

C. HDL Of all the lipid values, low HDL is the single best predictor of an adverse outcome. However, high HDL does not guarantee immunity from coronary artery disease. C-reactive protein levels predict risk for myocardial infarction and stroke even better than LDL levels do, but the level indicated in this question puts the parson at average risk. Levels above 3.00 indicate a high risk for myocardial infarction or stroke

You are caring for an adult who has carried the diagnosis of ADHD since his early school years. Of the following symptoms, which typically remain stable through the lifespan of the person with ADHD? a. Hyperactivity b. Impulsivity c. Inattention d. Oppositional behavior e. Conduct disorder

C. Inattention The symptoms of ADHD can change over time. While hyperactivity and impulsivity tend to peak between the ages of 6 and 10, inattention remains relatively stable through the lifespan of the illness. Oppositional behavior and conduct disorders may be comorbid, but are not necessarily features of ADHD

You are caring for a 30-year-old woman who has had asthma since childhood. Currently, she reports symptoms three or four times a week, but never more than once a day. Sometimes her symptoms cause her to skip her usual exercise regimen. She wakes in the night approximately three or four times a month to use her inhaler and return to bed. Which of the following classifications best characterizes her asthma? a. Mild intermittent b. Moderate intermittent c. Mild persistent d. Moderate persistent e. Severe persistent

C. mild persistent mild intermittent = <2xweek at night <2xmonth mild persistent = >2xweek, >once a day moderate persistent = daily sx + SABA Severe persistent = continual sx, limit Physical activity, frequent exacerbations ad night sx

You are seeing a 17-year-old girl who reports intense vaginal itching and urinary frequency. She has been sexually active for 6 months. On examination, you note frothy yellow-green discharge with bright red vaginal mucosa and red macules on the cervix. What is the saline preparation of the discharge most likely to show? a. Sheets of epithelial cells "studded" with bacteria b. "Moth-eaten" epithelial cells c. Motile triangular organisms with long tails d. Many white blood cells e. Hyphae

C. motile triangular organisms with long tails, which is significant for trichomonas vaginalis + strawberry cervix "studded epi cells" = clue cells = BV moth eaten = acid base disturbance WBC = upper genital infx

You are caring for a 35-year-old woman in the hospital, admitted for cellulitis. She also has a long history of migraine headaches. On day 2 of her hospitalization, she becomes diaphoretic, restless, and irritable. Within hours, she is complaining of severe pain, abdominal cramps, and diarrhea. Which of the following would most likely be present in her urine toxicology screen? a. Cocaine b. Marijuana c. Opiates d. 3,4-Methylenedioxymethampheatmine (MDMA or ecstasy) e. Benzodiazepines

C. opiates

A 28-year-old man presents to your office to discuss weight management. You determine his body mass index (BMI) to be 28.2 kg/m 2. How should you classify this patient? a. His BMI classifies him as being underweight. b. His BMI places him within the normal range. c. His BMI classifies him as being overweight. d. His BMI classifies him as obese. e. His BMI classifies him as morbidly obese.

C. overwiehgt

You are seeing a 27-year-old male migraine sufferer. His attacks happen approximately monthly, and he would like to discuss abortive therapy. Which of the following options is the best initially prescribed option?

C. triptan

A 38-year-old man comes to the office to discuss his headache symptoms. He describes the headaches as severe and intense, "like an ice pick in my eye!" The headaches begin suddenly, are unilateral, last up to 2 hours, and are associated with a runny nose and watery eye on the affected side. He gets several attacks over a couple of months, but is symptom-free for months in between flare-ups. Which of the following is the best approach for prophylactic management of the attacks? a. SSRIs b. Triptans c. NSAIDs d. Calcium channel blockers e. Ergotamine

Calcium channel blockers Nifedipine

You are following a patient after an acute hepatitis B infection. His serologies are shown below: • HBsAg: Positive • HBeAg: Positive • IgM anti-HBc: Negative • IgG anti-HBc: Positive • Anti-HBs: Negative • Anti-HBe: Negative Which of the following terms best describes his disease status? a. Acute infection, early phase b. Acute infection, recovery phase c. Chronic infection, replicating virus d. Chronic infection, nonreplicating virus e. Previous exposure with immunity

Chronic infection, replicating virus

You are caring for a 19-year-old man who has been treated for mild intermittent asthma since childhood. He has been controlled using a short acting bronchodilator as needed. Over the past month, he has been using his inhaler more than four times a week, and has had to wake up in the middle of the night to use his inhaler on three occasions. In the past, he was intolerant of the side effects associated with an inhaled corticosteroid. Which of the following is the most appropriate treatment option? a. Long-acting β-agonist b. Leukotriene receptor antagonist c. Cromolyn (Intal) d. Theophylline e. Oral corticosteroids

B. Leukotriene inhibitor

A 41-year-old sedentary man with frequent flare-ups of back pain presented to you 6 weeks ago with the acute onset of low back pain radiating to the left leg. His neurologic examination at the time was normal, but he did not respond to conservative therapy. X-rays are normal. Which of the following is the most appropriate next step? a. Flexion and extension radiographs b. Magnetic resonance imaging (MRI) c. Electromyelography d. Bone scan e. A complete blood count (CBC) and erythrocyte sedimentation rate (ESR)

B. MRI

An alcoholic patient of yours is interested in pharmacologic therapy to help him with his sobriety. His counselor recommended he try naltrexone, and he asks you how that medication works in alcoholism. Which of the following is the best answer for your patient? a. If the person taking naltrexone ingests alcohol, it causes an adverse reaction. b. Naltrexone reduces the reinforcing effects of alcohol. c. Naltrexone blocks the effects of alcohol by binding to alcohol-receptor sites on cells. d. Naltrexone saturates the alcohol-receptor sites on cells by acting as an alcohol agonist. e. Naltrexone changes the binding sites on alcohol, making it unable to bind to cells.

B. Narcan reduces the reinforcing effects of alcohol, not allowing patient to become "drunk"

You are taking care of a 47-year-old woman with cirrhosis. She asks you about transplantation as a definitive treatment option. Which of the following is an absolute contraindication to transplantation? a. Active alcoholism b. Portal vein thrombosis c. Hepatitis B surface antigen positivity d. HIV positivity e. Extensive previous abdominal surgery

B. Portal Vein Thrombosis Absolute contraindications to liver transplantation include portal vein thrombosis, severe medical illness, malignancy, hepatobiliary sepsis, or lack of patient understanding. Relative contraindications are active alcoholism, HIV or hepatitis B surface antigen positivity, extensive previous abdominal surgery, and a lack of a personal support system

You are evaluating a 36-year-old obese woman who complains of fatigue. She denies polydipsia, polyuria, polyphagia, or weight loss. Which of the following laboratory reports confirms the diagnosis of diabetes? a. A random glucose reading of 221 mg/dL b. A random glucose reading of 221 mg/dL, and another, on a later date, of 208 mg/dL c. A fasting glucose measurement of 128 mg/dL d. A glucose reading, taken 2 hours after a 75-g glucose load, of 163 mg/dL e. A fasting glucose of 114 mg/dL, and a reading of 184 mg/dL 2 hours after a 75-g glucose load

C. A fasting glucose measurement of 128 mg/dL

You are treating an elderly postmenopausal woman with osteoporosis. She recently suffered an acute osteoporotic vertebral fracture and is suffering from secondary pain. Which of the following osteoporosis treatments also has analgesic effects with respect to bone pain? a. Estrogen b. Combination of calcium and vitamin D c. Calcitonin d. Alendronate (Fosamax) e. Raloxifene (Evista)

C. Calctionin Calcitonin directly inhibits osteoclastic bone resorption and is considered a reasonable treatment alternative for patients with established osteoporosis in whom estrogenreplacement therapy is not recommended. It has the unique characteristic of producing an analgesic effect with respect to bone pain and is often prescribed for patients who have suffered an acute osteoporotic fracture

In a pre-employment screen, one of your patients tested positive for cocaine use. He presents to you and would like to discontinue his use, but reports having significant problems with the withdrawal symptoms. Which of the following symptoms is expected from cocaine withdrawal? a. Tachycardia b. Hypertension c. Depression d. Paranoia e. Insomnia

C. Depression

You are evaluating a 28-year-old man who is concerned about depression. He reports increased irritability, depressed mood, decreased enjoyment from usual activities, and sleep and appetite disturbances for 6 weeks. He reports a history of alcohol use, and currently has 6 beers a day on the weekdays, with up to 12 on the weekends. Which of the following is the most appropriate next step in treating his depression? a. Treat with a selective serotonin reuptake inhibitor (SSRI). b. Treat with bupropion. c. Recommend detoxification and abstinence. d. Recommend detoxification and abstinence and start an SSRI. e. Recommend detoxification and abstinence and start bupropion

C. Detox/abstinence

You are performing a review of systems on a 40-year-old female patient, and find that several symptoms are pointing toward thyroid disease. Of the following symptoms and signs, which is most commonly seen and reported in people with hyperthyroidism? a. Weight loss b. Tremor c. Fatigue d. Anorexia e. Increased sweating

C. Fatigue Hyperthyroidism results from elevated levels of thyroid hormones and is less common than hypothyroidism in the general population. While most people think of fatigue as being a symptom of hypothyroidism, it is actually the second most common reported symptom for hyperthyroidism. Weight loss, tremor, anorexia, and increased sweating all occur with less frequency. Only tachycardia is more commonly reported

A 43-year-old obese patient comes to your office with a painful, inflamed, swollen elbow. He reports that the pain began suddenly last evening, without a known precipitant or trauma. The pain is exquisite, and does not allow him to move his elbow at all—in fact, even the pressure of his bed sheet on his elbow was painful. On examination, he has an elbow effusion with warmth, erythema, and intense pain with movement. Which of the following is most likely the cause? a. Rheumatoid arthritis (RA) b. Osteoarthritis c. Gout d. Stress fracture e. Cellulitis

C. Gout

You are thinking about starting a type 2 diabetic on insulin therapy to improve her glucose control. Which of the following insulin preparations has the longest duration of action? a. Aspart (Novolog) b. Lispro (Humalog) c. Regular d. Neutral protamine hagedorn (NPH) e. Inhaled insulin (Exubera)

D. NPH It is important to thoroughly understand the action of the different types of insulin preparations in order to make therapeutic decisions about diabetic patients and their control. Lispro's and Aspart's duration of action is between 3 and 5 hours. Inhaled insulin lasts between 3 and 5 hours as well. Regular insulin lasts between 4 and 12 hours. Neutral protamine hadedorn (NPH) lasts for around 10 to 20 hours. The preparations with the longest duration of action include Lantus and Levemir, with a duration of around 24 hours

You are caring for an insomniac and counseling him regarding the importance of sleep hygiene. Which of the following is an important aspect of sleep hygiene that you should share with the patient? a. Exercise before bed. b. Eat a meal close to bedtime. c. Ingest a moderate amount of alcohol to help with sleep. d. Go to bed when you are tired. e. If you have difficulty falling asleep, watch television in bed until you become sleepy.

Go to bed when you are tired

A 45-year-old woman comes to your office after her husband noticed that her "skin turned yellow." She reported "flu symptoms" a couple of weeks ago, with weakness, loss of appetite, nausea, and abdominal pain for 2 days. She has felt better since, but noted the skin discoloration yesterday. On examination, her skin tone, conjunctivae, and mucous membranes are yellow-tinged. Serologies indicate acute hepatitis A infection. Which of the following is true about this infection? a. She is most infectious while she is jaundiced. b. Most infected adults are asymptomatic. c. This infection never results in chronic hepatitis. d. Relapses are common. e. Fecal shedding of the virus continues until liver enzymes have normalized.

Never results in chronic hepatitis

You are seeing a 58-year-old smoker for a routine health examination. You have counseled him on discontinuing tobacco use, and he is considering that alternative. He denies coughing, shortness of breath, or hemoptysis. Which of the following is a recommended screen for lung cancer in this patient?

No screening recommended

A patient that you follow has recently started volunteering at a drug treatment hospital and requires hepatitis B vaccination. You find that he is hepatitis B surface antibody positive. Which of the following would be the best guideline to follow in this case?

No vaccination is necessary based on his laboratory evaluation.

One of your patients, a 70-year-old man, complains of frequently dribbling urine throughout the day. On occasion, he loses a large amount of urine without warning. He is otherwise healthy and takes no other medications. Based on his profile and symptoms, which of the following terms best describes his symptoms? a. Functional incontinence b. Senile incontinence c. Urge incontinence d. Stress incontinence e. Overflow incontinence

Overflow incontinence

One of your patients has tried and failed behavioral therapy for incontinence. He describes a strong urge to urinate, followed by involuntary loss of urine. Which of the following would be the best medication for him to use? a. Oxybutynin (Ditropan) b. Pseudoephedrine (Sudafed) c. Trimethoprim-sulfamethoxazole (Bactrim, Septra) d. Finasteride (Proscar) e. Terazosin (Hytrin)

Oxybutynin (Ditropan)

A 42-year-old man that you treat suffers from cluster headaches. He would like a medication to take when he has an attack (abortive therapy). Which of the following would be best for treatment of the acute episodes? a. Indomethacin, 120 mg by mouth b. Oxycodone, 5 to 10 mg by mouth c. Sumatriptan, 50 to 100 mg by mouth d. Ergotamine, 1 to 2 mg by mouth e. Hundred percent oxygen, administered via a nasal canula

Inhaled Oxygen

On a routine urinalysis, a 30-year-old man was found to have hematuria. His urinalysis is negative for casts and protein, but is positive for moderate blood. His urine culture is negative. IV pyelogram and serum creatinine are both normal and you send urine cytology just to be safe. It is also normal. Which of the following is most appropriate in this case? a. Reassurance and periodic monitoring b. Renal ultrasound c. Cystoscopy d. ASO titer e. Renal biopsy

Rassurance

You are evaluating a 32-year-old woman complaining of amenorrhea. She has mild hypertension, hypothyroidism, GERD, and depression. On evaluation, her prolactin level was found to be 89 ng/mL (H). Which of the following medications would be most likely to cause the elevated prolactin level? a. Proton pump inhibitors b. SSRIs c. Thiazide diuretics d. ACE inhibitors e. Thyroid hormone replacement

SSRIs

A 44-year-old mother of two reports leakage of a small amount of urine with sneezing. Recently, it began to occur with exercise. She denies recent life stressors. Which of the following best describes the type of incontinence she is experiencing? a. Functional incontinence b. Senile incontinence c. Urge incontinence d. Stress incontinence e. Overflow incontinence

Stress Incontinence

You are treating a 40-year-old woman for incontinence. She would prefer not to use medications, and would like to try pelvic floor strengthening (Kegel) exercises. Which of the following types of incontinence has shown the best response to pelvic floor strengthening exercises? a. Functional incontinence b. Stress incontinence c. Urge incontinence d. Overflow incontinence e. Mixed incontinence

Stress incontinence

You are caring for a family and find that the mother would like her children vaccinated against influenza. Her children are aged 4 months, 24 months, and 5 years. Which of the following represents current immunization recommendations for influenza?

The 24-month-old and the 5-year-old should be vaccinated.

You are caring for a patient who is complaining of a headache. Which of the following, if present, represents a "red flag" and necessitates a workup?

a. Headache that presents after the age of 50 years

While you are working in the emergency room, a 17-year-old patient presents with a cat bite. He was helping a neighbor get his cat out of a tree 3 hours ago, and was bitten on the hand. On examination, you note erythema with some purulent discharge around a fairly deep wound with a jagged laceration. You irrigate the wound thoroughly and do not see tendon involvement. What is the best option in this case?

a. Hospitalization.

198. You are doing a well-child check on a 9-month-old infant. His growth chart reveals failure to thrive. When you ask his mother about his eating habits, she reports that he frequently refuses the bottle. His mother says that he is so irritable he often won't eat. On examination, he is small, but shows no physical abnormalities. However, the child does not smile or vocalize and seems withdrawn. When you ask his mother about this, she shrugs her shoulders and says, "that's just how he is." Which of the following tests, if any, would be best to reveal the diagnosis in this case?

a. No testing is necessary

A 23-year-old sexually active woman visits a free clinic reporting a sudden onset of dysuria that began 2 days ago. On further questioning, she also reports urinary frequency, some back pain, and a pink discoloration to her urine. She denies vaginal discharge or irritation and has been afebrile. The clinic has no microscope or urine dipsticks available. Based on this history, what is her most likely diagnosis?

a. Acute bacterial cystitis

A 36-year-old man has had recurrent bouts of sinusitis. He develops at least three sinus infections per year and wants to discuss prevention. Which of the following conditions is the most likely precipitating factor for his recurrent sinusitis? a. Allergic rhinitis b. GERD c. Cigarette smoking d. Environmental pollutants e. Immunodeficiency

a. Allergic rhinitis, MC

You are evaluating a 30-year-old male patient in the office with hematochezia. He has had chronic constipation, and reports bright red blood from his rectum associated with extremely painful bowel movements. After defecation, he complains of a dull ache and a feeling of "spasm" in the anal canal. The pain resolves within a few hours. On external examination, no abnormalities are noted. Which of the following is his most likely diagnosis?

a. Anal fissure

You are seeing a 25-year-old patient complaining of a left-sided earache. She describes the pain as deep, and it worsens with eating. Her ear examination is normal, but she has tenderness and crepitus during palpation of the left temporomandibular joint. Which of the following would be the most appropriate next step?

a. Antibiotic therapy

You are talking with a 24-year-old man who reports an outbreak of a mildly pruritic rash. The rash initially began with a large pink patch on his chest, to the left of his sternum breast. About a week later, he noted a more generalized eruption. The rash is shown below. Which of the following treatments is indicated? a. Antihistamines b. Antibiotics c. Antivirals d. Antifungals e. Cyclosporine

a. Antihistamines the pt describes pityriasis rosea single herald patch then generalized eruption

You are evaluating a 61-year-old man in the office who is complaining of chest pain. Given his history and risk factors, you are concerned about myocardial ischemia, and order an ECG. Which of the following ECG features, if present, would most markedly increase the likelihood of an acute myocardial infarction?

a. Any ST-segment elevation greater than or equal to 1 mm

You are seeing a hypertensive 56-year-old woman who is complaining of a "fluttering in her chest." She describes a rapid heart rate and to her it seems irregular. She is otherwise well, and denies shortness of breath, light-headedness pedal edema, or other acute symptoms. On examination, her pulse rate is rapid and irregular. Which of the following is her most likely diagnosis? a. Atrial fibrillation b. Paroxysmal supraventricular tachycardia (PSVT) c. Stable ventricular tachycardia d. Stimulant abuse e. Hyperthyroidism

a. Atrial fibrillation

A 60-year-old man is being evaluated for fatigue, weakness, and exercise intolerance. Laboratory assessment reveals: Hemoglobin: 9.1 mg/dL (L) Serum iron: 46 μg/dL (L) Ferritin: 9 ng/mL (L) Total iron binding capacity (TIBC): 626 μg/dL (H) Mean corpuscular volume (MCV): 76 fL (L) What is the most common cause of this condition?

a. Blood loss

You are evaluating a patient with cervical dystonia. Which of the following treatment options is best supported by evidence? a. Botulinum toxin b. Physical therapy c. Stretching techniques d. Use of a cervical collar e. Ice/heat

a. Botulin Toxin

A 45-year-old woman in your practice is complaining of amenorrhea. During the workup, you discover her testosterone and dehydroepiandrosterone sulphate (DHEA-S) levels are elevated. Which of the following should be your next step? a. CT scanning of the adrenal glands b. Hysteroscopy c. Hysterosalpingogram d. MRI of the brain e. Karyotyping

a. CT scan adrenal glands

You are seeing a 15-month-old boy for a well-child check. His parents have no concerns and his developmental history is normal. His growth chart is shown. Which of the following is the most likely observation?

a. Familial short stature

You are seeing a 21-year-old patient with pruritic, erythematous papules in clusters on his ankles and legs. He noticed the rash the day after he stayed in a hotel on his way back from a Spring Break vacation in Florida. Based on this history and description, which is the most likely culprit?

a. Flea bites

You are seeing a 14-year-old girl who hurt her ankle while dancing yesterday. She reports that her ankle "twisted in" causing immediate pain and the inability to bear weight. In the office, she has bruising and tenderness over the anterior talofibular ligament (ATFL) with acute swelling. She is unable to bear weight due to the pain. Which of the following is the most appropriate next step? a. Obtain x-rays of her ankle. b. Encourage early mobilization. c. Prescribe rest, ice, compression, and elevation. d. Use an NSAID to help with the pain and inflammation. e. Begin physical therapy

a. Get ankle X-Ray as an important NEXT STEP. all others are good too

You are seeing a 28-year-old woman who is complaining of constipation. She reports that her symptoms have been present since she can remember, and no dietary changes have seemed to benefit her. She has never tried pharmacologic therapy in the past. Which of the following would be the best first-line therapy for her?

a. Psyllium (Metamucil)

A 32-year-old woman reports that she sometimes "skips heartbeats." Her medical and social histories include moderate daily caffeine use, but are otherwise unremarkable. Her physical examination and 12-lead ECG are normal, as are her CBC, electrolytes, and TSH. Which of the following is the next appropriate step in her workup? a. Reassure her and continue observation. b. Perform ambulatory ECG monitoring (a 24-Holter monitor, or a continuous loop event recorder). c. Electrophysiology consultation. d. Stress testing. e. Echocardiography.

a. Reassurance, continue observation

You are seeing a 6-month-old boy whose mother reports that he has had diarrhea for almost 2 weeks. He has had four to six bowel movements a day, with a loose to liquid consistency. His mother stays at home with him and the child is not in day care. His symptoms began after his young cousins visited for Christmas. Which of the following is the most likely cause of his diarrhea?

a. Rotavirus

You have been treating a 43-year-old woman with rheumatoid arthritis for years. You and her rheumatologist have had her illness in relatively good control. She presents to you with a red eye and significant eye pain. She denies trauma. Upon further questioning, she complains of decreased vision and headache. She describes the pain as deep and boring. Her examination reveals diffuse injection of the deeper vessels with minimal discharge. Her pupils react normally. Which of the following is her most likely diagnosis? a. Scleritis b. Episcleritis c. Corneal abrasion d. Acute glaucoma e. Iritis

a. Scleritis decreased vision, deep boring eye pain, and HA, commonly associated with rheumatoid condition

A 23-year-old woman presents to your office to discuss fatigue. She describes a "lack of energy" and "tiredness," but denies weakness or hypersomnolence. Which of the following is the next step in the workup? .

a. Screen for depression

You are seeing a 36-year-old woman with a complaint of nipple discharge. Which of the following characteristics of the discharge is most suspicious for breast cancer?

a. Spontaneous discharge

You are examining a newborn whose mother has a positive screen for hepatitis B surface antigen (HBsAg). Which of the following is true regarding this situation? a. When acquired early in life, the large majority of those infected with hepatitis B will have chronic disease. b. If the child has a normal immune system, his likelihood of developing chronic disease is small. c. A higher percentage of adults infected with hepatitis B will develop chronic disease as compared with children. d. A high percentage of children acutely infected will develop fulminant liver disease. e. When hepatitis B is transmitted perinatally, the child generally develops the typical symptoms of acute hepatitis.

a. When acquired early in life, the large majority of those infected with hepatitis B will have chronic disease.

You are considering treatment for a 19-year-old female patient with primary dysmenorrhea. Which of the following should be your first-line therapy? a. Use of NSAIDs during menses b. Use of NSAIDs daily c. Use of opiates during menses d. Use of an SSRI daily e. Use of combined oral contraceptive pills daily

a. use of NSAIDS during menses

You are talking with a 24-year-old woman complaining of a headache. She reports that before she has the headache, she experiences visual symptoms associated with slight nausea. When the headache occurs later, it is throbbing, pulsating, and unilateral. During the headache, she experiences light sensitivity. Sleep improves the symptoms. Her symptoms are disrupting her daily life and you decide to try prophylactic therapy. Which of the following is the most studied prophylactic agent to use?

a. β-Blockers

A screening urinalysis in a female patient reveals asymptomatic bacteriuria. In which of the following patients would treatment be indicated?

b. A pregnant 26-year-old woman

You are caring for a 26-year-old man with vertigo. You have diagnosed him with a peripheral vestibular disorder, and are considering treatment options. Which of the following would be the first-line therapy?

b. Antihistamines

You are seeing a 3-year-old child whose mother says she is having trouble in breathing. This is her third episode of difficulty in breathing in the last year. On examination, you note nasal flaring and sternal retractions with accessory muscle use. You auscultate expiratory wheezes bilaterally. Which of the following is the most likely diagnosis?

b. Asthma

You are caring for a 27-year-old woman complaining of pelvic pain. She reports localized pain on the left side that has increased in severity over the last 2 days. She also reports nausea. On examination, you note a tender adnexal mass on the left. Which of the following is the most likely cause? a. PID b. Ectopic pregnancy c. Ovarian cyst d. Uterine leiomyoma e. Appendicitis

b. Ectopic pregnancy: nausea is common, pain may sometimes radiate to shoulder

You are evaluating an infant for poor weight gain. History from his mother reveals that he has frequent "wet burps" after eating. He coughs during and after eating, and his mother has heard him "wheeze" on occasion. He has not had diarrhea. Which of the following tests, if any, would be best to reveal the diagnosis?

b. Esophageal pH probe.

You are evaluating a 55-year-old patient who is complaining of a hand tremor. The tremor is bilateral and symmetric and is not accentuated during movement. It seems to be worse when holding a pen. When the patient squeezes your hand, the tremor is noticeable, but does not seem to worsen. Which of the following best characterizes this condition? a. Intention tremor b. Essential tremor c. Parkinsonian tremor d. Tic e. Chorea

b. Essential tremor

You are seeing a 19-year-old African-American student who reports that he can "feel his heartbeat." It happens with exercise and is associated with some light-headedness and shortness of breath. On examination, his heart has a regular rate and rhythm, but you hear a holosystolic murmur along his left sternal border. It increases with Valsalva maneuver. Which of the following is the most likely cause of his symptoms? a. Mitral valve prolapse b. Hypertrophic obstructive cardiomyopathy c. Dilated cardiomyopathy d. Atrial fibrillation e. CHF

b. HOCM associated with a fib/vtach, systolic ejection murmur, worsens with valsalva

In the evaluation of a 55-year-old man complaining of dizziness, you perform the Dix-Hallpike (Nylen-Barany) maneuver several times. You had the patient sit on the edge of the examining table and lie down suddenly with the head hanging 45° backward and turned to either side. With this maneuver, the vertigo was reproduced immediately and symptoms did not lessen regardless of repetition. The direction of the nystagmus changed with changing the direction that the head is turned, and the symptoms were of mild intensity. Which of the following is the most likely cause of the vertigo?

a. Stroke

You are treating a 16-year-old patient with a sore throat. She has had 3 days of symptoms and does not have nasal congestion or cough. She also reports laryngitis. On examination, she has an erythematous pharynx without exudate. Which of the following is the most appropriate therapy based on the symptoms described? a. Supportive care b. Penicillin (PenVK) c. Amoxicillin (Amoxil) d. Erythromycin (Emycin) e. Clindamycin (Cleocin)

a. Supportive care Laryngitis with pharyngitis is generally associated with a viral infection, and only supportive care is needed. Antibiotic therapy is not indicated in this case.

You are seeing a 7-year-old girl whose parents brought her in to have her bed-wetting evaluated. She has been toilet trained during the day since the age of 4, but still wets the bed at night. Her father wet the bed until the age of 8 years. Her physical examination reveals no abnormalities and her urinalysis is normal. Which of the following statements is true regarding this situation?

a. This problem is likely due to maturational delay

You are seeing a 13-month-old Caucasian boy. His growth chart is shown on the next page. His past medical history and physical examination are otherwise unremarkable, and he is meeting his developmental milestones. Which of the following is most likely to reveal the cause of his growth pattern?

a. Thorough dietary history

An 18-year-old woman is seeing you for back pain, frequency, and dysuria. She has never had a UTI in the past, and though she recently became sexually active, she denies vaginal discharge or risk for sexually transmitted infection. In this setting, when would a urine culture be necessary?

b. If a urine dipstick was positive for leukocyte esterase only

You are caring for a 35-year-old woman who works in a veterinary hospital. She noted an itchy rash on her calf approximately 3 weeks ago. It started as a small pink circular lesion, but is spreading, and has not responded to over the counter ointments. The rash is shown below. Which of the following is the most likely causative agent? a. Trichophyton rubrum b. Trichophyton tonsurans c. Trichophyton metagrophytes d. Microsporum canis e. Candida albicans

a. Trichophyton rubrum

One of your patients has been on β-blocker therapy for migraine prophylaxis. Her symptoms are not controlled and she is interested in trying another prophylactic medication. Which class of antidepressants has the strongest evidence base for prophylactic use in migraines?

a. Tricyclic antidepressants

You are seeing a young child whose mother brings him in with a rash. It developed on his upper lip underneath his nose. He has had cold symptoms with a runny nose recently. His picture is shown below. Which of the following is the most likely cause of this rash? a. Infection with group A β-hemolytic streptococci b. Infection with S aureus c. Infection with an Enterococcus species d. Infection with H influenzae e. Infection with a Pseudomonas species

b. Staph aureus d/t impetigo "golden crust" tx: mupirocin

You are seeing a 14-year-old boy who was brought emergently to your office after developing severe testicular pain while weight lifting. He had a sudden onset of severe pain without fever. On examination, his cremasteric reflex is absent, and Prehn sign is negative. His urinalysis is normal. Which of the following is the most likely diagnosis? a. Epididymitis b. Testicular torsion c. Inguinal hernia d. Orchitis e. Testicular cancer

b. Testicular torsion cremasteric reflex absent - prehn sign = torsion +prehn sign in epi, hernia, orchitis, cancer

A 53-year-old woman is seeing you because of chronic nausea and vomiting. She has a 15-year history of type 2 diabetes mellitus. Her symptoms are worse after eating and on occasion she will vomit food that appears to be undigested. Her weight is stable and she does not appear dehydrated. Which of the following is the best treatment for her condition? a. An anticholinergic medication, like scopolamine (Transderm Scop) b. An antihistamine, like promethazine (Phenergan) c. A benzamide, like metoclopramide (Reglan) d. A cannabinoid, like dronabinol (Marinol) e. A phenothiazine, like chlorpromazine (Thorazine)

c. A benzamide, like metoclopramide (Reglan)

You are determining which of the patients in your practice should receive the quadrivalent HPV vaccination. This vaccine is inappropriate for which of the following patients?

c. A pregnant 14-year-old

You are evaluating a 38-year-old man complaining of swelling of his right leg. He denies dyspnea. On examination, he is obese. You note pitting edema on the right without signs of trauma, erythema, or inflammation. Which of the following would be the most appropriate next step in the evaluation of his edema?

b. Lower extremity Doppler

You are evaluating a 25-year-old woman who reports frequent UTIs since getting married last year. In the last 12 months, she has had five documented infections that have responded well to antibiotic therapy. She has tried voiding after intercourse, she discontinued her use of a diaphragm, and tried acidification of her urine using oral ascorbic acid, but none of those measures decreased the incidence of infections. At this point, which of the following would be an acceptable prophylactic measure?

b. Single-dose antibiotic therapy once daily at bedtime for 12 months

You are caring for a 3-year-old boy who goes to daycare while his parents are at work. His mother brought him to see you because the daycare will not take him back until he's had a doctor evaluate his eye symptoms. He developed an acute redness of the left eye, associated with runny nose, cough, and increased irritability. On examination, his eye is red and watery. The discharge is clear, and he has mild eyelid edema. Which of the following is the most common cause for his condition? a. Coxsackie virus b. Parainfluenza virus c. Adenovirus d. Rhinovirus e. Herpesvirus

c. Adenovirus is MC virus causing conjuctivitis

You are seeing a 78-year-old man who was brought to the office by his daughter. The daughter says her father is becoming increasingly forgetful. His medical history is significant for a 20-year history of type 2 diabetes and well-controlled hypertension. On examination, he is mildly hypertensive with otherwise normal vital signs. He is oriented to time, place, and person, but is unable to complete "serial sevens" on a mini-mental status examination. Which of the historical features make this diagnosis more consistent with dementia as opposed to delirium? a. His history of hypertension b. His history of diabetes c. His current level of orientation d. His inability to complete serial sevens e. The recent onset of his symptoms

c. His current level of orientation

A 33-year-old obese but otherwise healthy woman comes to your office complaining of fatigue. It has been present for about 2 weeks, and she describes feeling "tired" with "no energy." She denies any change in her work or home schedule. Given this brief history, which of the following is most likely?

c. Hypothyroidism

The mother of a 9-month-old infant brings him in for irritability. The child has been fussy and has not been sleeping well for 2 days. His highest temperature has been 100°F, and he has had a clear runny nose and cough. On examination, the child is crying and irritable. Which physical examination finding, by itself, is insufficient to diagnose acute otitis media?

c. Impaired tympanic membrane mobility

You are evaluating a 33-year-old woman complaining of palpitations. Which of the following characteristics, if present, increase the likelihood that the symptoms are cardiac in etiology? a. The fact that the patient is female b. The fact that the patient has a sister with similar symptoms c. Her description of the symptoms as an "irregular heartbeat" d. The fact that her father has a history of heart disease e. The fact that the episodes last less than 1 minute

c. Irregular heartbeat

A 55-year-old male patient comes to you with concerns about a nodule underneath his right eye. The nodule is shown below. He reports that it "popped up" a couple of weeks ago and has been growing ever since. Which of the following is the most likely diagnosis? a. Verruca vulgaris b. Molluscum contagiosum c. Keratoacanthoma d. Nodular basal cell carcinoma e. Squamous cell carcinoma

c. Keratocanthoma

You are evaluating a 40-year-old woman with a new onset of bilateral lower extremity edema. She denies dyspnea, and on examination has no rales or JVD. On evaluation, she has an abdominal fluid wave. Which of the following should be the next step in the evaluation of her edema?

c. Liver function tests

You are seeing a 45-year-old obese diabetic woman who reports bilateral lower extremity peripheral edema. In addition to diabetes, she has arthritis, hypertension, and depression. Which of the following medications is the likely cause of her edema?

c. Naproxen

279. You are caring for a 23-year-old woman complaining of pelvic pain. She reports one-sided pain that is diffuse and dull, but occasionally sharp. Menses have been normal. She denies fever. Based on this history alone, which of the following is the most likely cause of the pain? a. Pelvic inflammatory disease (PID) b. Ectopic pregnancy c. Ovarian cyst d. Uterine leiomyoma e. Appendicitis

c. Ovarian Cyst mobile adnexal mass with peritoneal signs

You are seeing a 17-year-old patient who began menstruating at age 14, and has been relatively regular since age 15. She made an appointment to be seen today because she stopped having periods 2 months ago. She denies sexual activity. Which of the following is the most likely cause of her secondary amenorrhea? a. Polycystic ovarian syndrome b. Functional hypothalamic amenorrhea c. Pregnancy d. Hypothyroidism e. Hyperprolactinemia

c. Pregnancy

You are caring for a 25-year-old male who presents to you for evaluation of a new lesion found on his groin. On examination, you find a single small umbilicated flesh-colored papule, 4 mm in size, in his pubic region. Which of the following is true? a. A family history of skin cancer is likely in this person. b. The patient is probably immunocompromised. c. This is likely to be a sexually transmitted infection. d. This can be spread through aerosolized droplets. e. Treatment for this condition must be surgical.

c. STI is likely Molluscum cantangiosum caused by the POX virus

You are evaluating a 6-year-old girl brought in by her parents to discuss bed-wetting. She was toilet trained in the daytime at the age of 3 years, and was dry at night at about 3 1/2 years of age. Four months ago, her parents had another child, and the 6-year-old began to wet the bed at night. She has no medical condition that would account for the change. Which of the following terms correctly describes this condition?

c. Secondary monosymptomatic enuresis

You are caring for a 16-year-old girl with moderate acne. Her current regimen includes topical retinoids, benzoyl peroxide gel, and oral tetracycline, but after 4 months on this regimen, she has not had improvement. You are considering treatment with oral isotretinoin (Accutane). In addition to ensuring that pregnancy is prevented during her therapy, which of the following must occur during her therapy? a. She must avoid Tylenol use. b. She must stop wearing her contacts. c. She must stop her tetracycline. d. She must be screened for depression every 3 months. e. She must not use topical glucocorticoids.

c. She must stop her tetracycline can cause pseudomotor cerbri when used together with ro accutane

You are doing a preoperative history and physical examination on a 58-year-old woman who will be undergoing a thyroidectomy later in the month. She is obese, sedentary with type 2 diabetes and hyperlipidemia. She reports that she is unable to walk two blocks without stopping to rest. She denies chest pain with activity. What type of cardiac evaluation should she have prior to undergoing her surgical procedure?

c. She should have a noninvasive stress test prior to surgery.

You are caring for a 32-year-old female smoker with a history of allergic rhinitis who presents to discuss upper respiratory symptoms. She reports congestion, facial pressure, nasal discharge and headache. Her symptoms have been present for 5 days and they have not improved with decongestants. Sinus palpation causes significant pain. Which of the following is true regarding the diagnosis of sinusitis in this patient? a. Pain with sinus palpation is valuable in making her diagnosis. b. Lack of improvement with decongestants is valuable in making her diagnosis. c. Symptom duration is valuable in making her diagnosis. d. Her history of allergic rhinitis is valuable in making her diagnosis. e. Plain sinus x-rays are valuable in the diagnosis of sinusitis.

c. Symptom duration is valuable in making her dx

You are evaluating a 14-year-old girl with pelvic pain. She denies being sexually active and you do not suspect abuse. On pelvic examination, you confirm that she has never been sexually active, see no discharge, and find no cervical motion tenderness, but feel an ovarian mass on the right side. Which of the following is the most appropriate next step in this situation? a. Reassurance and use of NSAIDs for pain control b. Reassurance and repeat pelvic examination in 6 to 8 weeks c. Transvaginal pelvic ultrasound d. CT scanning of the abdomen and pelvis e. MRI evaluation of the pelvis

c. TVUS *** first test ordered

You are discussing recurrent vaginal discharge with a patient. She has been evaluated three times in the last 6 months and has had a vaginal candidiasis each time. She has symptom-free intervals, but the infection seems to be recurrent. She is monogamous, sexually active with a male partner, takes birth control pills, and denies other complaints or known illnesses. Which of the following is most likely to improve this patient's condition? a. Test her for diabetes, and control her elevated glucose level. b. Ask her to discontinue her birth control pills. c. Treat her sexual partner. d. Test her for HIV, and begin therapy. e. Ask her to add fiber to her diet.

c. Treat her sexual partner , although it is not a STI, it can cause balanitis in men

You are caring for a 42-year-old woman complaining of incontinence. She reports often having a strong, immediate need to void, followed by an involuntary loss of urine. She says her symptoms develop so suddenly, she often urinates while trying to get to the bathroom. Which of the following best describes the type of incontinence she is experiencing? a. Functional incontinence b. Senile incontinence c. Urge incontinence d. Stress incontinence e. Overflow incontinence

c. Urge Incontinence

You are counseling a 23-year-old woman who is interested in starting COC pills. Which of the following is true regarding risks associated with COC use?

c. Users of COC pills have an increased risk of venous thromboembolism.

You are seeing a 32-year-old otherwise healthy woman who is complaining of palpitations. She describes the sensation as a "flip flop" in her chest. They only last an instant and are not associated with light-headedness or other symptoms. She denies other symptoms. Which of the following is the most likely etiology of her complaint? a. Atrial fibrillation b. PSVT c. Ventricular premature beats d. Stimulant abuse e. Hyperthyroidism

c. Ventricular premature beats

A new mother brings her infant to see you to discuss his vomiting. He is 4 weeks old and is exclusively breast-fed. He vomits with every meal. On examination, his abdomen is distended with normal bowel sounds, and he appears dehydrated. He has lost 4 oz since his visit with you 2 weeks ago. Which of the following is the most likely diagnosis? a. Allergy to breast milk b. GERD c. Pyloric stenosis d. Intussusception e. Small-bowel obstruction

c. pyloric stenosis

You suspect Parkinson disease in a patient that you've been caring for. You test the glabella tap reflex and find that the patient consistently blinks after 15 taps. Which of the following is most accurate regarding this finding? a. This is a normal finding and is reassuring. b. This is pathognomonic for Parkinson disease. c. This is a common finding in Parkinson disease. d. This is a common finding in patients with essential tremors. e. This is a poor prognosis.

c. this is a common finding in Parkinson Disease. Caused by tapping patients forehead

A 33-year-old woman is seeing you with a chief complaint of "dizziness." Upon further characterization, she describes "unsteadiness" and a feeling that "her balance is off." Based on this description, which of the following terms should be used to characterize her complaint?

d. Disequilibrium

A 61-year-old man comes to see you for shortness of breath. He has a history of hypertension, type 2 diabetes, and hyperlipidemia. He quit smoking 4 years ago after a more than 30-pack-year history. On physical examination, he is not in respiratory distress, but he has diffuse wheezing in the bilateral lower lobes of his lungs. His cardiac examination demonstrates an S 4 and he demonstrates JVD. Which of the following treatments would likely relieve his symptoms? a. Antibiotic therapy b. Epinephrine c. Steroid therapy d. Diuretics e. Anticoagulation

d. Diuretics Pt is suffering from CHF as he has all the Risk Factors epi - allergy steroids - asthma anticoag- PE abx - infx

A patient presents for evaluation of a new rash. You suspect atopic dermatitis. Which feature of the rash, if present, would lead you to suspect a different cause for the rash? a. Lesions that are present on both antecubital areas, both cheeks, and both legs b. Severe pruritus c. Erythematous lesions with flaking d. First outbreak of this rash occurring as an adult e. Rash on the flexural surfaces of the body

d. First outbreak of this rash occurring as an adult

You are seeing a young man who is complaining of a patch of hair loss. He denies pulling the hair and complains that his scalp is itchy and flakey. His scalp is shown below. Which of the following is the treatment of choice for this condition? a. Selenium sulfide lotion, applied daily for 4 to 8 weeks b. Ketoconazole (Nizoral) shampoo applied daily for 4 to 8 weeks c. Clotrimazole (Lotrimin) cream, twice daily for 4 to 8 weeks d. Griseofulvin tablets, daily for 4 to 8 weeks e. Fluconazole (Diflucan) tablets, daily for 4 to 8 weeks

d. Griseofulvin tablets, daily for 4 to 8 weeks

One of your patients is dying of end-stage breast cancer. She is complaining of dyspnea. Which of the following treatment options would be most beneficial?

d. Opioids

You are seeing a 32-year-old nurse who was treated in an urgent care for bacterial conjunctivitis. Despite the appropriate use of ciprofloxacin ophthalmic solution over the last 4 days, her purulent discharge and erythema have not improved. What should be the next step in treatment of this patient? a. Ciprofloxacin ointment b. Polymyxin-trimethoprim ophthalmic solution c. Oral ciprofloxacin d. Oral sulfamethoxazole-trimethoprim e. Immediate ophthalmologic referral

d. Oral Bactrim, which is used to treat MRSA in all parts of the body

You are evaluating a 56-year-old African-American man complaining of fatigue. He describes this as a lack of stamina, but has motivation to do things. Sleep refreshes him, but he tires quickly at work. His physical examination is unremarkable. Which of the following should be included in your initial workup to help ascertain the diagnosis?

d. Prostate cancer screen

You are seeing a 34-year-old man with urinary symptoms. He reports frequency, urgency, and moderate back pain. He is febrile and acutely ill. He has no penile discharge. His urinalysis shows marked pyuria. He has never had an episode like this before, and has no known urinary tract abnormalities. Which of the following is the most likely diagnosis?

d. Pyelonephritis

You are caring for a 26-year-old generally healthy woman. She is sexually active and currently in a monogamous relationship. You recently completed her annual examination. Her Pap smear reports "atypical squamous cells of undetermined significance." Human papillomavirus (HPV) testing was negative. Which of the following is the most appropriate next step?

d. Repeat the Pap smear in 1 year.

A 16-year-old camp counselor sees you to evaluate a severely pruritic rash. You note pruritic erythematous papules in between his fingers, on his wrists, and around his waist. For which of the following is this distribution characteristic?

d. Scabies

Four weeks ago, you treated a 22-year-old woman for acute bronchitis. Although she feels much better, the cough has persisted. She has used bronchodilators, antihistamines, and antitussives. Which of the following is the best course of treatment at this time?

e. An oral steroid taper

A 28-year-old woman comes to see you for a tender and erythematous area on her breast. She is nursing her 6-week-old son. You diagnose mastitis. Which of the following is true regarding this condition?

e. Antibiotic therapy is indicated.

You are evaluating a 63-year-old diabetic man who noted unilateral lower extremity edema. He denies dyspnea or recent trauma. On evaluation, you note pitting edema on the right with well-demarcated erythema from the ankle to the mid thigh. Which of the following is the best treatment option?

e. Antibiotics

You are evaluating a 9-month-old Caucasian girl for poor weight gain. She has gone from the 75th percentile to the 10th percentile in height and weight. She has had recurrent respiratory infections and diarrhea, but cultures obtained have been negative. Which of the following will be the most useful test in this setting?

d. Sweat chloride test

You are seeing a 6-year-old boy who is being evaluated for ADHD. Surveys completed by his teachers and his parents are consistent with the diagnosis. Which of the following blood tests should you obtain prior to making the diagnosis? a. No blood tests are necessary. b. Complete blood count. c. Sedimentation rate. d. Thyroid-stimulating hormone. e. Toxicology screen

d. TSH When screening for ADHD, a thorough history and physical should be completed, and behavioral rating forms should be reviewed. Laboratory testing should be obtained to rule out other causes of symptoms, and include blood chemistries, a thyroid stimulating hormone, and a lead level. Most children with ADHD will have normal laboratory values, but these tests can exclude other causes of symptoms. 493

You are evaluating a 44-year-old man with painless, large volume intestinal hemorrhage. You suspect a Meckel diverticulum as the possible cause. Which of the following is the best test to confirm this diagnosis?

d. Technetium-99m pertechnetate scintigraphic study

You are caring for a 19-year-old who came in for evaluation of his sore throat. He has been ill for 24 hours. In addition to the sore throat, he has a fever, but no cough. On physical examination, his temperature is 101°F, tender anterior cervical adenopathy, and tonsillar exudate. Which of the following is the next best step in his care? a. Reassurance and observation for a few more days. b. Perform a rapid streptococcal screening test. c. Perform a throat culture. d. Treat with penicillin. e. Treat with steroids.

d. Treat with penicillin Centor Criteria gives 1 point to: - tonsillar exudates - tender anterior cervical adenopathy - fever - lack of cough (these 4 strongly suggest infx) no lab testing, treat.

You are evaluating a 9-month-old child with recurrent wheezing. His mother also reports that he vomits after formula as well. Which of the following is the best test to determine the cause of his wheezing? a. Pulmonary function testing b. Chest x-ray c. Upper GI barium swallow d. Upper endoscopy e. Twenty-four hour pH probe

e. 24hr pH probe testing (gold standard for GERD) Pt described likely has GERD which is a common cause of wheezing in infants. Also, the vomiting up of formula is typical of GERD

You are discussing cancer screening with a female patient. She has no family history of breast cancer, and routine risk analysis indicates that she is not at increased risk for the disease. According to the US Preventive Services Task Force (USPSTF), at what age should she start getting routine mammograms?

e. 50 years

You are evaluating a 21-year-old woman with an erythematous, tender, and edematous hand. She reports that while playing with her cat 3 days ago, he bit her and punctured the skin. The area around the bite is inflamed, and there is a purulent discharge from the puncture site. Which of the following is the most likely infecting organism?

d. Pasteurella multocida

You are caring for a 33-year-old woman without medical concerns. She is married and monogamous, and on oral contraceptives. Her Pap test 8 months ago reported "atypical squamous cells of undetermined significance." You repeated the Pap test 4 months later, and it was normal. You repeated it last week, and the results once again reported "atypical squamous cells of undetermined significance." HPV testing is unavailable. Which of the following is the most appropriate next step?

d. Perform colposcopy.

Your patient is known to have genital herpes. He is starting to date a new partner, and she does not have genital herpes. Which of the following statements regarding this situation is true? a. In this case, it is recommended to use antiviral therapy only when the infection is active, but to start the medication at the first sign of an outbreak. b. Daily therapy with an antiviral agent has been demonstrated to eliminate asymptomatic viral shedding. c. Daily therapy with an antiviral agent can change the natural course of the infection in the affected patient. d. Daily therapy with an antiviral agent can be associated with resistance to the antiviral medication used. e. Daily therapy with an antiviral agent may reduce the risk of HIV transmission and acquisition.

e. Daily suppression reduces the risk of HIV transmission and acquisition

You care for a child whose mother wants him immunized against influenza. He is currently 18 months old and has never had the influenza vaccine in the past. Which of the following is correct regarding this situation?

e. He should receive two doses of TIV.

You are evaluating a 56-year-old farmer who is complaining of dyspnea. His history includes being hospitalized for bronchiolitis as a young child leading to childhood asthma, and a history of pneumonia 2 years ago, for which he was also hospitalized. He has a 20-pack-year history of smoking. Which of the following increase his risk for having restrictive lung disease as the cause of his dyspnea?

e. His occupation as a farmer

You have been following a 15-month-old male infant. At 9 months, his height was at the 25th percentile while his weight was at the 5th percentile. At his 12-month visit, his weight and height are unchanged, so you asked his family to bring in a detailed dietary history and counseled them on a healthy diet. At his 15-month visit, his weight is up slightly, and his vital signs are as follows: Blood pressure: 62/32 mm Hg (low) Heart rate: 72 beats/min Respiratory rate: 16 breaths/min Temperature: 98.8°F Which of the following is the best therapeutic option for this child?

e. Hospital admission

You are evaluating a 33-year-old woman with chronic pelvic pain. She reports cyclic pain, generally during the premenstrual period and during her menses. She has been trying to conceive for 15 months without success. Her pelvic examination is normal. Which of the following tests would be most helpful in determining the cause of her pain? a. CBC b. ESR c. CA-125 levels d. Transvaginal pelvic ultrasound e. MRI

e. MRI ***** The patient described has symptoms and signs suggestive of endometriosis

You are seeing a 23-year-old sexually active male who presents to have a testicular mass evaluated. He noticed it approximately 3 months ago, but was afraid to have it evaluated. It is painless and has caused him no symptoms, and he does not think it has enlarged since he first noticed it. On examination, you note a nodule on his left spermatic cord. What is the appropriate diagnostic test in this situation? a. Scrotal ultrasound. b. Abdominal and pelvic CT. c. Urinalysis. d. Urethral smear. e. No testing is necessary.

e. No testing necessary as it is a spermatocele (asymptomatic nodules)

You are evaluating an 18-year-old male with a sore throat. It has been present for 3 days and is associated with fever, aches, and fatigue. On examination, he has an exudative pharyngitis, soft palate petechiae, and posterior cervical adenopathy. Which of the following is the most likely diagnosis? a. Group A streptococcal infection b. Group A streptococcal colonization c. Corynebacterium diphtheriae infection d. Gonorrhea infection of the throat e. Infectious mononucleosis

e. Infectious mononucleosis cervical LAD is the defining factor bc all the others can cause the other symptoms palatal petechiae is usually Group A strep

A 36-year-old woman comes to your office complaining of recurrent dysuria. This is her fourth episode in the past 10 months. Initially, her symptoms were "classic" for a UTI. She was treated without obtaining urine dipstick or microscopic evaluation. For the second episode, her urinalysis was positive for blood only. Her culture was negative, as was evaluation for nephrolithiasis. The third episode was similar, also with a negative culture. All episodes have resolved with a standard course of antibiotic therapy. Which of the following is the most appropriate next step?

e. Use daily single-dose antibiotic therapy for prophylaxis

You are evaluating a 5-year-old girl whose mother brought her in to evaluate jaundice. Laboratory evaluation reveals a conjugated hyperbilirubinemia. Which of the following is the most likely cause of her problem? a. G6PD deficiency b. Gilbert disease c. Crigler-Najjar syndrome d. Wilson disease e. Viral hepatitis

e. Viral hepatitis

You are seeing a 48-year-old man who complains of nausea and vomiting. He is nauseated before breakfast, and he describes the vomiting as "severe" and "projectile." His symptoms are associated with headache and dizziness, but improve throughout the day. Which of the following is the most likely diagnosis? a. Gastroparesis b. Cholelithiasis c. Pancreatitis d. Vestibular disorder e. Brain tumor

e. brain tumor

You are caring for a 13-year-old girl with acne. She is becoming increasingly concerned about her appearance and is worried about getting teased at school because of her skin. After assessment, you diagnose her with mild acne. Of the following treatment regimens, which would be best for her at this time? a. Topical antibiotics b. Benzoyl peroxide gel c. Topical antibiotics and benzoyl peroxide gel d. Topical retinoids e. Topical antibiotics, benzoyl peroxide gel, and topical retinoids

e. combo of topic abx, benzoyl peroxide, and topical retinoids

A 68-year-old man complains of fatigue. He has a history of hypertension, well-controlled with hydrochlorothiazide. He has recently lost 30 lb on a high-protein, low-carbohydrate diet. He drinks 2 to 3 beers daily, and smokes 10 cigarettes daily. Laboratory evaluation reveals a macrocytic anemia and vitamin B 12 deficiency. Which of the following is the most likely cause?

e. inadequate vitamin B 12 absorption

You receive a telephone call from the mother of a 19-year-old patient. During the day, she complained of a headache, body aches, and a lowgrade fever. She went to bed 30 minutes ago and her mother is now finding it difficult to arouse her. Which of the following tests would be most likely to reveal the diagnosis? a. Urinalysis b. CBC c. Toxicology screen d. Pregnancy test e. Lumbar puncture

e. lumbar puncture

You are caring for a 20-year-old male patient who is concerned about facial acne. He has had moderate symptoms since his teenage years, but has not ever tried a formalized treatment regimen. Which of the following tests or sets of tests is best to help guide treatment of his disorder? a. Free testosterone. b. Dihydroepiandrosterone sulfate (DHEAS). c. Free testosterone and DHEAS. d. Free testosterone, DHEAS, follicle-stimulating hormone, and luteinizing hormone. e. No laboratory examinations are required.

e. no labwork is necessary the only time we would order labwork is if we suspect an endocrine issue/abnormality

You are caring for a 45-year-old man who reports a 2- to 3-year history of episodic flushing of the cheeks, nose, and forehead. Over the last several months, this has been more constant, and he has developed papules and some pustules on his cheeks. His picture is shown below: Which of the following is the most effective treatment for this condition? a. Topical metronidazole cream b. Topical sodium sulfacetamide c. Topical antibiotics d. Topical steroids e. Oral antibiotics

e. oral antibiotics is the most effective tx

You are evaluating 34-year-old woman reports amenorrhea for 4 months. She has never been "regular," but has never gone this long without a period. Her laboratory evaluation is normal, including a negative pregnancy test. You give her medroxyprogesterone acetate (Provera) for 7 days, and the next week, she reports having a period. Which of the following is the most likely cause of her amenorrhea? a. Premature ovarian failure b. Ovarian neoplasm c. Turner syndrome d. Asherman syndrome e. Polycystic ovarian syndrome

e. polycystic ovarian syndrome *progestin challenge makes woman ovulate and have a menstruation

You are seeing a 16-year-old student complaining of ear pain. His pain has been present for 2 days. He denies fever and has no symptoms of upper respiratory infection. On examination, his ear canal is tender, erythematous, and swollen. His tympanic membrane is obscured by discharge and debris. Which of the following is the treatment of choice for this patient?

e. Oral steroids

You are seeing a 26-year-old male patient complaining of a red eye who says, "I think I have pink eye." He reports increased redness, tearing, discharge, photophobia, and pain. Which of his reported symptoms would be more suggestive of something other than conjunctivitis? a. Redness b. Tearing c. Discharge d. Photophobia e. Pain

e. Pain, suggestive of a more serious problem such as: - acute angle closure glaucoma - uveitis - scleritis - keratitis - foreign body - corneal abrasion

You are seeing a sedentary, obese 41-year-old woman who presents to you with acute shortness of breath. She has tachycardia, but no other abnormal examination findings. You order a D-dimer and it comes back low. Which of the following is the most appropriate option?

e. Reassure the patient that her symptoms are not concerning at this time.

A 32-year-old mother of two young children presents to your office for evaluation of her left eye. She reports redness of the white part of her eye, with a watery discharge. She reports mild itching and a sensation as if something is in her eye. She denies a history of allergies and reports no concurrent allergic symptoms. Examination reveals a palpable preauricular lymph node. Fluorescein staining does not reveal corneal dendrites. Which of the following should be the treatment of choice in this case? a. Antiviral eye drops b. Antibacterial eye drops c. Corticosteroid eye drops d. Combination antibiotic/corticosteroid eye drops e. Supportive care

e. Supportive Care adenovirus is MC and the condition is self limiting

A 59-year-old male patient is presenting for preoperative testing before undergoing a hernia repair. He has a history of coronary artery disease and hyperlipidemia, but no other significant medical history. He had a positive stress test 4 years ago that was followed by an angioplasty. He has been asymptomatic ever since. Which of the following is the best answer regarding the type of cardiac evaluation should he have prior to undergoing his procedure?

e. Surgery should be deferred.

A 22-year-old healthy male sees you for "diarrhea." He reports frequent loose stools without bleeding. You determine that he likely has a virally mediated process and recommend supportive care. Which of the following dietary measures should you recommend?

e. The patient can eat rice and potatoes

You are discussing preventive health screening with a 19-year-old college student. He has no family history of hypertension, coronary artery disease, diabetes, or cancer. At what age should you consider screening for lipid disorders?

35 years old

A 38-year-old HIV-positive man follows up in your office for routine care. Unfortunately, his antiretroviral therapy is failing, and his CD4 count is falling. At his last two visits, his CD4 count has been less than 65 lymphocytes/mm 3. Prophylaxis for which of the following should be instituted at this time? a. Mycobacterium avium complex b. Fungal infections c. Herpes simplex d. Herpes zoster e. Cytomegalovirus

A Prophylaxis against M avium complex (MAC) should be instituted once the patient's CD4 count drops less than 75 to 100 lymphocytes/mm 3. Prophylaxis against Pneumocystis pneumonia should be considered once the CD4 count drops less than 200 lymphocytes/mm 3. Prophylaxis for fungal disease has been studied, but there was no benefit in the group that had prophylaxis with regard to mortality. Prophylaxis for herpes simplex and herpes zoster is not generally done. CMV prophylaxis can be instituted in those with CMV IgG positivity and with CD4 counts less than 50 lymphocytes/mm 3, but it is generally not done because ganciclovir (the primary prophylactic agent) can cause neutropenia

You have diagnosed a 66-year-old woman with heart failure. She has a history of hypertension, but has never had heart failure before. Which of the following tests is routinely indicated in the initial evaluation of a person with a new diagnosis of heart failure? a. Echocardiogram b. Holter monitor c. Left heart catheterization d. Treadmill stress test e. Pharmacologic stress test

A Routine laboratory testing in a person with the new diagnosis of heart failure includes an electrocardiogram, a CBC, a urinalysis, serum creatinine, potassium and albumin levels, and thyroid function studies. An echocardiogram is imperative to help identify structural abnormalities of the heart and to measure the ejection fraction. Holter monitoring is not routinely warranted, as it would not identify a cause for heart failure, but would be used to identify an arrhythmia. Catheterization or stress testing may be important if ischemia or ischemic cardiomyopathy is identified as a cause, but is not a routine initial test

You are treating a 45-year-old man for hypertension. Since beginning therapy, he complains of urinary leakage and urgency. Which antihypertensive class is most likely to cause this? a. Thiazide diuretics b. ACE inhibitors c. β-Blockers d. Calcium channel blockers e. α-Blockers

Beta Blockers

You are evaluating a 56-year-old generally healthy man who is seeing you after finding blood in his urine. He denies pain, dysuria, frequency, or urgency. He is a smoker, and has worked for years in the printing industry. Which of the following is the most likely cause of his hematuria? a. Acute prostatitis b. Chronic prostatitis c. Cystitis d. Urinary stones e. Bladder carcinoma

Bladder Carcinoma

You are seeing an 18-year-old man who has had a cough for 2 weeks. It started like a typical "cold," but has persisted. Over the last 3 days, the cough has come in "spasms" and he barely has time to catch his breath during the coughing episodes. Nasopharyngeal swab confirms the diagnosis of pertussis. Which of the following treatments is recommended?

A 5-day course of azithromycin

A 30-year-old woman with frequent back problems was putting her groceries into her trunk and had a recurrence of low back pain. She has tried acetaminophen for 2 days without relief. On examination, her range of motion is limited, and she has tenderness to palpation of the lumbar paraspinal muscles. Which of the following treatment options is best? a. NSAIDs and return to normal activity b. Opiate analgesia and limited activities c. Oral corticosteroids d. Bed rest for 3 to 5 days e. Spinal traction

A. NSAIDS and return to normal activity

You are caring for a woman who describes primary orgasmic dysfunction and comes to you for advice. Which of the following therapies has shown to be most effective in treating this condition? a. Directed self-stimulation b. The "stop-start" technique c. Group therapy d. Hypnotherapy e. Sensate focus

A. Directed self stimulation

An 18-year-old morbidly obese patient in your office is found to have a fasting glucose of 314 mg/dL. Which of the following test results would indicate that he is a type 1 diabetic? a. Low levels of C-peptide b. Markedly elevated levels of C-peptide c. Elevated levels of microalbumin in the urine d. A markedly elevated hemoglobin A 1C e. Nerve conduction studies showing mild peripheral neuropathy

A. Low levels of C-peptide are found in DM1 patients all the other levels are seen in type 1 and 2 DM

You have diagnosed a 49-year-old man with CHF because of left ventricular systolic dysfunction. In addition to acute diuresis, which of the following is the best first-line agent to use for treatment, in the absence of contraindications? a. ACE inhibitors b. β-Blockers c. Calcium channel blockers d. Nitrates e. Hydralazine

A. ACE inhibitor Many clinical trials have shown that ACE inhibitors decrease symptoms, improve quality of life, decrease hospitalizations, and reduce mortality in patients with NYHA class II to IV heart failure. In addition, they slow the progression to heart failure among asymptomatic patients with left ventricular systolic dysfunction. All patients with heart failure should be prescribed an ACE inhibitor unless they have a contraindication. β-Blockers are helpful, but not necessarily as a first-line agent. Nitrates and hydralazine can be used in patients who do not tolerate ACE inhibitors, as can ARBs. Some calcium channel blockers (nifedipine, diltiazem, and nicardipine) may worsen systolic dysfunction

You are caring for a type 2 diabetic whose measures of control have been worsening despite maximal doses of oral medications. You are considering adding insulin therapy to help with her post-prandial hyperglycemia and want to start a preparation with a rapid onset of action. Which of the following insulin types has the most rapid onset of action? a. Aspart (Novolog) b. Regular c. Neutral protamine hagedorn (NPH) d. Detemir (Levemir) e. Glargine (Lantus)

A. Aspart Aspart, lispro (Humalog), and glulisine (Apidra) have the most rapid onset of action, between 15 and 30 minutes. Regular insulin has an onset between 30 and 60 minutes. Neutral protamine hagedorn (HPH) has an onset between 1 and 2 hours, as does glargine (Lantus) and detemir (Levemir).

A 45-year-old woman presents to your office for evaluation. She reports that over the last few weeks, she has noted an enlarging mass in the front of her neck. She feels well, has had no changes in her health, and denies symptoms of hyper- or hypothyroidism. She also denies recent viral illness. On examination, you note a diffusely enlarged thyroid that is tender to touch. Which of the following is her most likely diagnosis? a. Hashimoto thyroiditis b. Subacute lymphocytic thyroiditis c. Subacute granulomatous thyroiditis d. Suppurative thyroiditis e. Invasive fibrous thyroiditis

A. Hashimoto thyroiditis The most common cause of thyroiditis is chronic lymphocytic thyroiditis (also called Hashimoto thyroiditis). It is the most common cause of goiter in the United States. Generally seen in middle-aged women, this generally presents with enlargement of the thyroid, and most often there is associated tenderness. Subacute lymphocytic thyroiditis is less common, and although an acute increase in thyroid size is seen, it is generally nontender. Subacute granulomatous thyroiditis usually follows a viral illness and is also associated with a mildly painful gland. Suppurative thyroiditis is rare, and is associated with fever, a swollen thyroid, and clinical manifestations of a bacterial illness. Invasive fibrous thyroiditis presents as a gradually increasing gland that is firm, but is nontender

A 64-year-old woman comes to see you as a new patient. She is interested in finding the cause of her hand deformities. Upon inspection, you see the joints in her hands are nodular and enlarged as in the picture below . She does not complain of significant disability. Which of the following laboratory findings is likely in her case?

A. Heberdens node (DIP) Bouchard (PIP)Osteoarthritis like Percys hands

A 39-year-old diabetic man asks you questions about his diet. Which of the following is true? a. A high-fiber diet improves glycemic control b. A low-carbohydrate diet improves glycemic control c. A high-protein diet improves glycemic control d. Sucrose should not be included in the diabetic diet e. A formalized dietary program is more likely to produce long-term sustained effects

A. High fiber diet improves glycemic control Glycemic control is dependent on the total caloric intake, not the type of calorie taken in. Lowcarbohydrate and high-protein diets have not been shown to improve glucose control more than weight loss from other methods. Sucrose does not need to be eliminated, but it may raise blood sugar more quickly after ingestion. Formal dietary programs are not more likely to produce long-term sustainable results unless exercise is a large component of the plan. Increased fiber does improve glycemic control.

You are discussing weight management with an overweight 33-year-old woman. She has tried for years to lose weight, but despite multiple attempts, remains overweight. Which of the following is indicated in the workup of her weight concerns? a. History and physical alone b. CBC c. TSH d. Serum electrolytes e. Luteinizing hormone to follicle-stimulating hormone ratio

A. Hx and Physical Alone The history and physical examination are of utmost importance when evaluating the obese patient. Less than 1% of obese patients have a secondary nonpsychiatric cause for their obesity. Hypothyroidism and Cushing syndrome are important examples that can generally be detected by history and physical (but would need additional testing if historical features or physical findings point in that direction). Laboratory evaluation is necessary, however, to assess the medical consequences of obesity, and include fasting glucose, LDL, HDL, and triglyceride levels

You are treating a 48-year-old woman with arthritis in her knees. The pain keeps her from exercising, and she is becoming concerned about the limitations in her activity. Of the following, which would be the most appropriate first-line agent for her pain control? a. Ibuprofen, 600 mg tid b. Celecoxib, 200 mg daily c. Tramadol, 50 mg q4-6h d. Amitriptyline, 50 mg at night e. Gabapentin, 300 mg tid

A. Ibuprofen 600mg TID

You are caring for a patient with newly diagnosed diabetes. He is currently on no medications, and you have just obtained a screening lipid profile. You find that the patient's LDL cholesterol is 180 mg/dL. Given this patient's history, what is the patient's LDL treatment goal? a. Less than or equal to 70 mg/dL. b. Less than or equal to 100 mg/dL. c. Less than or equal to 130 mg/dL. d. Less than or equal to 160 mg/dL. e. There is no recognized treatment goal.

A. Less than or equal to 70 mg/dL. Treatment goals for persons with dyslipidemias should be established based on the patient's clinical status and other risk factors. For patients with known coronary artery disease or diabetes, the treatment goal for LDL cholesterol is less than or equal to 70 mg/dL. If a patient has no known coronary disease, the 10-year risk for coronary disease should be estimated using a readily available National Cholesterol Education Program (NCEP) risk calculator (available online). If the 10-year risk is greater than 20%, the LDL treatment goal should be less than or equal to 100 mg/dL. If the risk is between 10% and 20%, the LDL treatment goal should be less than 130 mg/dL. If the risk is less than 10%, the treatment goal should be less than 160 mg/dL.

You suspect that a 56-year-old male patient is abusing alcohol. Which of the following is the most *specific* laboratory test to confirm this? a. MCV b. ALT c. AST d. GGT e. lactate dehydrogenase (LDH)

A. MCV is most specific for alcohol abuse

The daughter of one of your patients accompanies her mother to the office to discuss her concerns. The mother seems to have had progressive cognitive failure over the last year. According to the daughter, she has become less able to remember things or care for herself. Which of the following would be most helpful in the diagnosis of Alzheimer disease? a. A mini-mental status examination b. Computed tomography of the brain c. Magnetic resonance imaging of the brain d. Lumbar puncture e. Blood work including serum chemistries, thyroid function testing, vitamin B 12 levels, a rapid plasma regain, and liver function tests

A. Mini Mental Status Exam There are no laboratory tests of sufficient accuracy to independently confirm a diagnosis of Alzheimer disease. The laboratory workup, which does include the blood work listed in answer e, is primarily done to rule out other conditions that can cause or contribute to the presenting symptoms. Structural imaging is important to rule out other causes as well and may include either a CT scan or an MRI. Positron-emission tomography (PET) scanning and evaluation of cerebrospinal fluid may be useful in very complicated cases, but are not used for diagnosis. The mini-mental status examination, the clock drawing test, or other brief screening tools may be most useful in the diagnosis.

You decide to treat a severely depressed patient with fluoxetine. The response is dramatic and on follow up he reports that he feels great. He has got a lot of energy—in fact he hasn't slept in 2 days. He just bought a new car despite losing his job. You suspect acute mania. Which of the following is the best choice of medications to control the acute symptoms? a. Neuroleptics b. Lithium c. Valproic acid d. Carbamazepine e. Lamotrigine

A. Neuroleptics In some bipolar patients, the diagnosis is made after the initiation of an antidepressant allows the patient to cycle into a manic phase. All the medications listed in this answer can be used to help bipolar disorder, but only the neuroleptics will be of benefit in the acute phase. Lithium, valproic acid, carbamazepine, and lamotrigine are all excellent options for maintenance once the acute mania is under control

Working up a tremor in a patient, you found that the patient does have Parkinson disease. Which of the following medications has been shown to delay functional impairment and disease progression? a. Selegiline (Eldepril) b. Carbidopa-levodopa (Sinemet) c. Bromocriptine (Parlodel) d. Pramipexole (Mirapex) e. Ropirinole (Requip)

A. Selegiline (Eldepril) MAOis have shown to delay functional impairment and disease progression

You have performed a screening lipid profile on an otherwise healthy man. His results indicate elevated triglycerides, a low HDL, a high LDL, an elevated total cholesterol, and an elevated very-low-density lipoprotein (VLDL). You would like to rescreen him in the fasting state. Which of the following laboratory values is likely to decrease in the fasting state? a. Serum triglycerides b. HDL c. LDL d. Total cholesterol e. VLD

A. Serum triglycerides Blood lipids change acutely in response to food intake. The triglyceride level is lowest in the fasting state and rises by an average of 50 mg/dL postprandially. As the triglyceride level rises, the total and LDL cholesterol each fall. Thus total and LDL cholesterol tend to be higher when fasting. HDL varies little whether fasting or not.

You are caring for an HIV-infected woman. She had a normal Pap test 2 weeks ago. Which of the following is true? a. She needs a repeat Pap test in 6 months. b. She needs a repeat Pap test in 12 months. c. She needs a colposcopy. d. She needs to have a colposcopy instead of her next Pap test. e. She should have prophylactic cone biopsy of the cervix.

A. She needs a repeat Pap test in 6 months The incidence of cervical dysplasia in HIV positive women is 40%. More HIV infected women die of cervical cancer than do from AIDS, therefore Pap testing should be done every 6 months. Some practitioners perform routine colposcopy regardless of the Pap results, but the evidence does not support that recommendation for all women at this point. Cone biopsy should be reserved for cases of serious dysplasia.

A 32-year-old woman is seeing you because her mother has been diagnosed with osteoporosis. She asks you what type of exercise will help her prevent the development of the disease. According to recommendations, which of the following exercises is most appropriate to help her maintain bone mass? a. Tennis b. Swimming c. Cycling d. Skating e. Skiing

A. Tennis

Despite lifestyle changes, a 37-year-old patient of yours still has blood pressures above goal. She has no other medical concerns and no abnormalities on physical examination or initial laboratory evaluation. Which of the following medications is best as an initial first-line monotherapy, according to the Joint National Committee 7 (JNC 7)? a. A thiazide diuretic b. An ACE inhibitor c. An angiotensin receptor blocker d. A calcium channel blocker e. A β-blocker

A. Thiazide Diuretic JNC 7 recommended that low-dose diuretics are the most effective first-line treatment for preventing the occurrence of cardiovascular morbidity and mortality. However, recent AHA scientific statement reported that evidence supports the use of ACE inhibitors, ARB, calcium channel blockers, or thiazide diuretics singly or in combination as first-line therapy.

You are following a patient after an acute hepatitis B infection. His serologies are shown below: • HBsAg: Positive • HBeAg: Positive • IgM anti-HBc: Positive • IgG anti-HBc: Negative • Anti-HBs: Negative • Anti-HBe: Negative Which of the following terms best describes his disease status? a. Acute infection, early phase b. Acute infection, recovery phase c. Chronic infection, replicating virus d. Chronic infection, nonreplicating virus e. Previous exposure with immunity

A. acute infection, early phase

You are caring for a patient who would like to quit smoking. She failed nicotine patches. Which of the following is an appropriate next step? a. Add nicotine gum to the patch. b. Use clonidine. c. Use a tricyclic antidepressant. d. Use an SSRI. e. Use a selective serotonin and norepinephrine reuptake inhibitor.

A. add nicotine gum to the patch

An alcoholic patient of yours is interested in pharmacologic therapy to help him in his sobriety. His counselor recommended he try disulfiram, and he asks you how that medication works in alcoholism. Which of the following is the best answer for your patient? a. If the person taking disulfiram ingests alcohol, it causes an adverse reaction. b. Disulfiram reduces the reinforcing effects of alcohol. c. Disulfiram blocks the effects of alcohol by binding to alcohol receptor sites on cells. d. Disulfiram saturates the alcohol receptor sites on cells by acting as an alcohol agonist. e. Disulfiram changes the binding sites on alcohol, making it unable to bind to cells.

A. alcohol + disulfram = causes adverse reaction

The joint aspirate from the inflamed first metatarsal-phalangeal joint of a 35-year-old woman reveals needle-shaped negatively birefringent crystals. The patient is intolerant to nonsteroidals. Which of the following is the most appropriate initial treatment? a. Colchicine b. Corticosteroids c. Opiates d. Allopurinol e. Probenecid

A. colchicine

You are caring for a 42-year-old woman who was diagnosed with rheumatoid arthritis (RA) 8 years ago. You are concerned about potential extra-articular manifestations of her disease. Which of the following signs or symptoms, if present, would signal extra-articular manifestations of RA? a. Cough b. Congestive heart failure (CHF) c. Gastrointestinal (GI) distress d. Peripheral neuropathy e. Renal failure

A. cough

You are evaluating a 39-year-old otherwise healthy man with a family history of ischemic heart disease. He describes chest pressure that radiates to his jaw when he walks up steps at work. You order an ECG in the office, shown in below . Which of the following is the test of choice to determine if his chest pain is because of cardiac ischemia?

B. The standard provocative test for ischemic heart disease is an exercise treadmill test. However, certain ECG abnormalities make the standard ETT unreadable. These include left ventricular hypertrophy with strain, left bundle branch block (shown in the question), and ST-segment baseline abnormalities in the precordial leads. In this case, a thallium ETT is preferred, as long as the patient can exercise.

A 62-year-old smoker comes to your office for treatment. After a thorough history and physical examination, you believe he has chronic obstructive pulmonary disease. He quit smoking 8 months ago, but has not had any other treatment. Which of the following is the best first-line therapy for his condition? a. A short-acting β-agonist (albuterol) b. An inhaled anticholinergic (ipratropium) c. An inhaled corticosteroid d. Oral theophylline e. Oxygen

B. Inhaled ipratroprium anticholinergic

You are seeing an African-American man with newly diagnosed diabetes. His blood pressure at the last visit was 118/76 mm Hg, and at this visit it is 112/72 mm Hg. Which of the following statements is true regarding the use of an ACE inhibitors in this patient? a. An ACE inhibitor should be added to his regimen because he is diabetic, regardless of his blood pressure. b. An ACE inhibitor should be added to his regimen based on his blood pressure readings. c. An ACE inhibitor should not be added to his regimen unless his blood pressure goes above 120 systolic. d. An ACE inhibitor should not be added to his regimen unless he has microalbuminuria. e. An ACE inhibitor should not be given to this patient if his creatinine is elevated.

B. An ACE inhibitor should be added to his regimen based on his blood pressure readings ACE inhibitors are clearly the first choice for blood pressure control in diabetic patients. They control blood pressure effectively, help prevent progression of renal disease, and are indicated in the presence of coronary disease and CHF. Although compelling, there is insufficient evidence to recommend ACE inhibitors in all diabetic patients. They are indicated for diabetics with systolic blood pressures greater than 100 mm Hg. They can be used irrespective of creatinine levels, though potassium should be monitored as creatinine rises. Providers do not need to wait to see microalbuminuria prior to initiating therapy

You have maximized oral therapy for a type 2 diabetic in your office. She works hard at diet and exercise, and is on maximal doses of oral hypoglycemics, but her glycosolated hemoglobin is 8.6%. You decide to add insulin to her regimen. She is currently 67 in tall and weighs 100 kg. How much NPH should you give her at night as an addition to her current regimen? a. 5 units b. 10 units c. 15 units d. 20 units e. 25 units

B. 10 units Patients with type 2 diabetes may require insulin therapy if diet, exercise, and oral hypoglycemic agent do not provide appropriate control. A low dose of NPH is commonly used, estimating 0.1 U/kg of body weight, as an addition to the current regimen

A 55-year-old man comes to your office after not being seen by a physician in more than 10 years. He is found to be hypertensive, and his creatinine is found to be 2.3 mg/dL (H). Which medication is most likely to control his blood pressure and decrease the likelihood of progression of his renal disease? a. A thiazide diuretic b. An ACE inhibitor c. A calcium channel blocker d. A β-blocker e. An aldosterone antagonist

B. ACEI Several clinical trials have documented the benefit of ACE inhibitors in patients with hypertension and chronic kidney disease. Angiotension receptor blockers are also beneficial

You are treating a 61-year-old man for hypertension. He is not responding well to combination therapy with a thiazide diuretic and a β-blocker. On physical examination, you note an abdominal bruit. Which of the following tests is most likely to help you evaluate him further? a. Chest x-ray b. Angiotensin-converting enzyme inhibitor renal scan c. Urinary metanephrines and vanillymandelic acid levels d. Aortic CT scan e. Echocardiogram

B. ACEI- scan The patient described in the question has physical examination findings consistent with renal artery stenosis. An ACE-inhibitor renal scan or renal magnetic resonance angiography would evaluate this. Urinary metanephrines and vanillymandelic acid levels would help rule out pheochromocytoma. A chest x-ray would be helpful if coarctation of the aorta were suspected. An aortic CT would help to or quantify an aortic aneurysm, and an echocardiogram would help to identify left ventricular hypertrophy or systolic dysfunction

You are caring for a man with asthma. He is currently taking an inhaled corticosteroid twice daily and using his short-acting β-agonist as needed. Over the past 3 months, he has required escalating doses of his inhaled corticosteroid, and now he is at the maximum dosage, still using his "rescue" inhaler more than he would like. Which of the following is the best medication to add to his regimen? a. A burst and rapid taper of oral steroids b. A long-acting β-agonist c. Cromolyn (Intal) d. Ipratropium (Atrovent) e. Theophylline

B. Add a LABA theophylline is a third line agent atrovent for COPD

You are seeing a 49-year-old man with a known history of hypercholesterolemia and hypertension who has had recent complaints of chest pain. He reports a chest pressure, described as "heaviness" in the substernal area. It is not associated with activity, but will occur intermittently throughout the day. Which of the following is the best way to describe what the patient is feeling? a. Classic angina b. Atypical angina c. Anginal equivalent d. Nonanginal pain e. Atypical nonanginal pain

B. Atypical angina occurs when the patient experiences pain that has the quality and characteristics of angina, or occurs with exertion, but not both. For example, atypical angina may be a sense of heaviness not consistently related to exertion or relieved by rest, or it may be pain with an atypical character (sharp or stabbing) but predictably brought on by exercise and relieved by rest. Classic angina has both features. Anginal equivalent occurs when dyspnea is the sole or major manifestation. Nonanginal pain has neither the quality nor the precipitants of angina. "Atypical nonanginal pain" is not a term used to describe chest pain.

A 26-year-old male college graduate is seeing you for an office visit. He is concerned that he may have adult attention-deficit hyperactivity disorder (ADHD). Which of the following is true regarding this condition? a. The symptoms are likely to be more pronounced in adults as compared with children. b. Children diagnosed with ADHD commonly continue to have symptoms into adulthood. c. Sleep disturbance is a distinctive feature of adult ADHD. d. Appetite disturbance is a distinctive feature of adult ADHD. e. The symptom picture of adult ADHD mimics that in children

B. Children diagnosed with ADHD commonly continue to have symptoms into adulthood. Of children diagnosed with ADHD, 50% to 75% will continue to exhibit symptoms into adulthood. In adults, symptoms of ADHD may be more subtle, and symptoms may actually change. Deficits in executive function tend to be more salient (poor organization or time management) and the "hyperactivity" of childhood may be replaced by restlessness. Patients with impulsivity as a child may replace that with difficulty monitoring behavior or modulating emotional intensity

What is the MOA of niacin? a. It substantially decreases LDL. b. It substantially raises HDL. c. It has no effect on triglycerides. d. Its side effects generally prevent it from being used. e. It can't be used in patients who have concurrent diabetes

B. It substantially raises HDL Niacin was the first lipidlowering agent associated with decreased total mortality. It moderately decreases LDL, can increase HDL by 20% to 25%, and moderately decreases triglycerides. It causes a prostaglandin-mediated flushing that patients often describe as "hot flashes." This side effect can be easily moderated by having the patient take a NSAID or aspirin at least an hour before taking the niacin. Although niacin can increase blood sugar, it is safe for diabetics to use.

***You are seeing a patient who was discharged from the hospital. She initially presented to the emergency room with dyspnea and was found to be in CHF. They admitted her for diuresis and initiation of appropriate first-line therapy. Since being released, she reports that she is comfortable at rest, but that ordinary activity results in mild dyspnea. According to the New York Heart Association (NYHA) functional classification, which class of heart failure best describes this patient? a. Class I b. Class II c. Class III d. Class IV e. Class V

B. Class 2 Class I: patients have no limitation of activity. Class II: patients have slight limitations, are comfortable at rest, but have fatigue, palpitations, dyspnea, or angina with ordinary activity. Class III: patients are also comfortable at rest, but less-than-ordinary activity causes symptoms. Class IV: patients have symptoms at rest and increased symptoms with even minor activity. There is no "class V" in this system.

You are evaluating a patient whose BMI is 44 kg/m 2. You would like the patient to consider weight-loss surgery, specifically a Roux-en-Y gastric bypass. Which of the following is true regarding this procedure? a. The operative mortality rate for this procedure in the first 30 days is near 5%. b. Complications from this procedure occur in approximately 40% of the cases. c. The procedure can be expected to help the patient lose up to 30% of initial body weight. d. Nutritional deficiencies after surgery are rare. e. This surgery is reserved for people with BMI greater than 30 kg/m 2.

B. Complications from this procedure occur in approximately 40% of the cases Bariatric surgery is an increasingly more common treatment option for severe obesity. In the United States, the most common procedure performed is the Roux-en-Y gastric bypass. The procedure can result in substantial weight loss, up to 50% of the initial weight in some studies. Complications are common and occur with about 40% of the cases. Operative mortality is actually quite low 0% to 1% in the first 30 days. Nutritional deficiencies are common postoperatively, and patients require life-long supplementation. Because of the risks of the surgery, bariatric surgery is limited to those with a BMI >40 kg/m 2, or >35 kg/m 2 if there are obesity-related comorbidities present

You have just diagnosed a 35-year-old man with hypertension. He is otherwise healthy and has no complaints. Which of the following is indicated in the initial evaluation? a. TSH level assessment b. Resting electrocardiogram c. Stress test d. Echocardiogram e. Renal ultrasound

B. EKG Baseline laboratory screening is important to assess for end-organ damage and identify patients at high risk for cardiovascular complications. The routine tests for a newly diagnosed hypertensive patient include: hemoglobin and hematocrit, potassium, creatinine, fasting glucose, calcium, a fasting lipid profile, urinalysis, and a resting electrocardiogram. Other tests are not indicated unless physical examination or history makes them likely to be positive

You have been treating a 46-year-old woman for type 2 diabetes for 2 years with an insulin regimen. She is compliant with her diet and medications, and exercises regularly. She is 65 in tall and weighs 200 lb. Her most recent HbA 1C is 9.0% which is elevated from 8.8% 3 months ago. When you adjusted her insulin regimen in the past, she has hypoglycemic episodes, so at her visit 2 weeks ago, you added the insulin-sensitizing agent pioglitazone to her regimen, and decreased her insulin dosage by 50%. The patient presents today complaining of a problem that she attributes to the new medication. Which of the following is the most likely complaint? a. Symptomatic hypoglycemia b. Edema and weight gain c. Cough d. Paradoxical hyperglycemia e. GI intolerance

B. Edema Thiazolidinediones sensitize peripheral tissues to insulin. They can be used as monotherapy, with insulin or in combination with metformin. When used as monotherapy, they can decrease the HbA 1C by about 1 to 2 percentage points. When added to the regimen of patients on insulin, it can reduce the insulin dosage by 30% to 50%. In the case mentioned, if you decreased the insulin dose by 50%, you would be unlikely to cause symptomatic hypoglycemia or hyperglycemia in this patient. Cough and GI intolerance are not commonly seen. Edema is a common side effect, occurring in about 3% to 4 % of patients using the medication. The edema occurs even more frequently in patients receiving concomitant insulin. Other side effects include anemia and weight gain

You are performing a screening physical examination on a 47-year-old man. He is generally healthy, and his review of systems is negative. His mother has type 2 diabetes, and he is overweight. Which of the following is generally accepted as the test of choice to screen for type 2 diabetes? a. A random glucose test b. A fasting glucose c. A urinalysis to screen for glycosuria d. A 1-hour glucose tolerance test e. A 3-hour glucose tolerance test

B. Fasting glucose : more accurate and is generally recommended

A 23-year-old man comes to your office to discuss premature ejaculation. He has had this condition since beginning sexual activity at 17 years of age. He has tried behavioral methods, but these have not been successful. Which of the following medications is most likely to help this condition? a. Alprostadil b. Fluoxetine c. Bupropion d. Silendafil e. Atenolol

B. Fluoxetine helps in premature ejaculation

A 36-year-old man sees you to discuss a lack of sexual interest. He is not having sexual fantasies and is unmotivated to begin sexual activity. He does not report depressive symptoms and has no other physical complaints. His physical examination is normal. Which of the following laboratory tests is most appropriate? a. Total testosterone b. Free testosterone c. Thyroid-stimulating hormone (TSH) d. Prolactin e. Prostate-specific antigen (PSA)

B. Free testosterone = more acurate

You are seeing a 28-year-old man with significantly elevated triglycerides. You are considering gemfibrozil (Lopid) therapy. What is the mechanism of action of gemfibrozil? a. Sequesters bile acids b. Changes hepatic metabolism of lipoprotein c. Inhibits HMG-CoA reductase d. Interferes with cholesterol absorption in the gut e. Decreases secretions of triglycerides by the liver

B. Gemfibrozil changes the hepatic metabolism of lipoproteins and is a logical choice for the patient with low HDL and elevated triglycerides.

You are seeing a 40-year-old woman who reports the gradual onset of low back pain over several months. The pain is associated with morning stiffness that improves throughout the day. On examination, there are no neurologic deficits. Which of the following is the most likely cause? a. Back strain b. Inflammatory arthropathy c. Disk herniation d. Compression fracture e. Neoplasm

B. Inflammatory Arthropathy

You are caring for a patient who has a problem with alcohol abuse. Upon direct questioning, he says that he drinks because he continually recounts stressful memories from being in the Iraq war. Which of the following medications is the best choice to treat his disorder? a. Bupropion b. Sertraline c. Alprazolam d. Valproic acid e. Venlafaxine

B. Sertraline The disorder described is posttraumatic stress disorder (PTSD)—a syndrome characterized by reexperiencing a traumatic event. Alcohol and drugs are commonly used by the patient to self-treat. Antidepressants are helpful to ameliorate the symptoms, with sertraline and paroxetine having FDA indications for treatment of this disorder. Alprazolam can be used, but there is significant concern for dependency problems. Sometimes antiepileptic medications can be used, but more studies are needed, and the FDA has not approved them for this disorder

You are following a type 2 diabetic woman in her fifties. Six months ago, you checked her lipid profile. At that time, her total cholesterol was 245 mg/dL, her low-density lipoprotein (LDL) was 148 mg/dL, her highdensity lipoprotein (HDL) was 30 mg/dL, and her triglycerides were 362. She has tried lifestyle modifications, but despite losing weight and exercising, her profile hasn't substantially changed. Which of the following is the first-line treatment for this patient? a. Continued lifestyle modifications b. A 3-hydroxy-3-methylglutaryl-CoA (HMG-CoA) [P1] reductase inhibitor (a "statin") c. Niacin d. Fibric acid derivatives e. Bile acid resin

B. Statin Statins are the drug of choice in treating hyperlipidemia in diabetes. They have been shown to decrease the risk of coronary events and are excellent in lowering LDL. They do have less effect on the triglyceride levels, but in many patients, the decrease is enough to get patients to goal. Niacin will decrease triglycerides, raise HDL, and lower LDL, but may increase insulin resistance. Niacin is often used in combination with a statin or alone in patients with statin side-effects. Fibric acid derivatives lower triglycerides and raise HDL, but have minimal effects on LDL. Bile acid resins sequester bile acids in the GI tract. They can increase triglyceride levels and are generally not used in diabetics.

You are treating an adolescent girl who has been diagnosed with ADHD. Her mother has heard reports of stimulant abuse and is concerned about her daughter's prescription. Which of the following is true regarding the abuse of stimulants in the adolescent population? a. There is no risk of substance abuse in adolescents treated for ADHD. b. There is a decreased risk of substance abuse in adolescents treated for ADHD. c. The risk of substance abuse in adolescents treated for ADHD is the same as adolescents who are not treated for ADHD. d. There is an increased risk of substance abuse in adolescents treated for ADHD. e. Adolescents should not be prescribed stimulants.

B. There is a decreased risk of substance abuse in adolescents treated for ADHD Stimulant medications for ADHD are controlled substances. While it is true that they can be abused, the preparations do not produce the euphoric effect of other stimulant medications if taken by the oral route. Some people do produce a euphoric effect by crushing and snorting the short-acting preparations. Regardless, stimulants are acceptable to prescribe in adolescents. In fact, studies have shown that treating ADHD in adolescents actually decreases the risk of substance abuse when compared to children not treated.

When examining a 35-year-old, you notice a firm 3-cm thyroid nodule. His thyroid studies are normal, and he is clinically euthyroid. Radionucleotide imaging demonstrates uptake in the thyroid nodule. Which of the following is the most likely diagnosis? a. Colloid cyst b. Thyroid adenoma c. Thyroid carcinoma d. Metastatic disease e. Neurofibroma

B. Thyroid adeonma Once a thyroid nodule is found, the next step in the workup is radionucleotide imaging. If a nodule takes up radiotracer, it is termed a "hot" nodule. Colloidal cysts and tumors do not take up tracer and are "cold" nodules. Therefore, "hot" nodules are more likely benign. Neurofibromas would also be "cold." Definitive diagnosis can be made through needle aspiration

You are evaluating a 20-year-old woman complaining of vaginal discharge. She reports vaginal itch and white discharge. She has no history of vaginal infections in the past and has never been sexually active. A potassium hydroxide (KOH) preparation of the discharge is shown below. Which of the following is the best treatment option for this condition? (Reproduced, with permission, from Wolff K, Johnson RA .Fitzpatrick's Color Atlas and Synopsis of Clinical Dermatology. 6th ed. New York, NY: McGraw-Hill; 2005:717 .) a. Reassurance that this is a normal variation and that no treatment is necessary. b. Topical azole applications. c. Topical metronidazole. d. Oral clindamycin. e. Doxycycline.

B. Topical azole application vaginal candidiasis with chessy exudate KOH = multiple hyphae tx = topical azole or ORAL fluconazole 1 time dose

You are seeing a patient in the office for the first time. She has had recent episodic shortness of breath and is concerned that she has developed asthma. Which of the following features, if present, is the strongest predisposing factor in the development of asthma? a. Family history of asthma b. History of atopy c. A history of childhood pneumonia d. Exposure to cigarette smoke e. Exposure to environmental pollution

B. history of atopy

You are seeing a 32-year-old woman for fatigue. Your differential diagnosis includes major depressive disorder, but she does not describe a depressed or irritable mood. Which of the following symptoms of depression must be present in order to diagnose a major depressive disorder in someone without depressed mood? a. Sleep changes b. Loss of interest or pleasure in usually enjoyable activities c. Guilt or feelings of worthlessness d. Loss of energy e. Change in appetite

B. loss of interest or changes in Depression is commonly seen in primary care settings. In fact, it is estimated that only about 20% of depression-related health care occurs in mental health care settings. Nonpsychiatrists write approximately 80% of the prescriptions for antidepressants. Patients with major depressive disorder often present with vague physical symptoms rather than emotional complaints. To make the diagnosis of depression using DSM criteria, the patient must describe either depressed mood for most of the day nearly every day for at least 2 weeks, or loss of interest in usually enjoyable activities. Irritable mood may take the place of depressed mood to make the diagnosis as well. In addition to one of those two symptoms, the patient must experience other symptoms of depression, including sleep changes, feelings of guilt or worthlessness, loss of energy, loss of concentration, change in appetite, psychomotor speeding or slowing, or suicidal thoughts, plans, or intent.

You are evaluating a 62-year-old man who is complaining of joint pain. His pain involves his left knee, right ankle, and both hands. He reports that his symptoms have been present for years, but are worsening. He has more pain with activity. On examination, you note some swelling in the joints with mild tenderness and crepitus. Which of the following is the most likely cause of his symptoms? a. Rheumatoid arthritis b. Osteoarthritis c. Gout d. Tendonitis e. Fibromyalgia

B. osteoarthirits

You are evaluating a patient with knee swelling and pain. You perform an arthrocentesis to help determine the diagnosis. The fluid analysis reveals rhomboid-shaped positively birefringent crystals. Which of the following is the most likely diagnosis? a. Gout b. Pseudogout c. Infectious arthritis d. Osteoarthritis e. Rheumatoid arthritis

B. pseudo gout

You are discussing migraine management with a 30-year-old woman. She wants to use prophylactic medications, but had debilitating fatigue and symptoms of depression on β-blockers. Which of the following medications is an acceptable alternative? a. Nifedipine b. Verapamil c. Diltiazem d. Amlodipine e. Nicardipine

B. verapamil

A 16-year-old girl comes to your office complaining of blood in her urine. She is asymptomatic and not menstruating. Urinalysis reveals grossly pink urine, but urine dipstick is negative for blood. Which of the following foods is the likely cause? a. Spinach b. Strawberries c. Raspberries d. Beets e. Carrots

Beets

You screened a 52-year-old, at-risk woman for osteoporosis using a DEXA scan. You received a T-score and a Z-score in the report. Which of the following indicates osteoporosis? a. Equal T- and Z-score b. T-score of +2.5 c. T-score of −2.5 d. Z-score of +2.5 e. Z-score of −2.5

C

You are considering adding an angiotension II receptor blocker (ARB) to the regimen of one of your patients with congestive heart failure. Which of the following statements is true regarding the use of ARBs in CHF? a. ARBs and ACE inhibitors have the same effects on the neurohormonal mechanisms involved in heart failure. b. Adding an ARB to an ACE inhibitor reduces mortality in patients with CHF. c. Adding an ARB to an ACE inhibitor can reduce hospitalizations in patients with CHF. d. Using an ARB instead of an ACE inhibitor increases mortality in CHF. e. Using an ARB instead of an ACE inhibitor increases hospitalizations in patients with CHF

C Adding an ARB to an ACE inhibitor can reduce hospitalizations in patients with CHF

You are treating a patient for heart failure because of systolic dysfunction with daily diuretics and an ACE inhibitor. He is continuing to have symptoms with activity, but they do not seem to be related to volume overload. Adding which of the following medications has been shown to reduce symptoms and improve mortality? a. Metolazone (Zaroxolyn) b. Spironolactone (Aldactone) c. Metoprolol (Toprol XL) d. Nifedipine (Procardia) e. Digoxin (Lanoxin)

C β-Blockers inhibit the adverse effects of sympathetic nervous system activation in heart failure patients. Studies have shown that three β-blockers (bisoprolol, metoprolol, and carvedilol) can reduce symptoms, improve quality of life, and reduce mortality. Adding diuretics does not change mortality. Nifedipine can worsen symptoms. Digoxin improves symptoms, but does not decrease mortality

You have just diagnosed osteoporosis in a postmenopausal woman. She is considering treatment alternatives and wonders about the bisphosphonates. Which of the following is the best description of how this class of medications works? a. They increase calcium absorption in the GI tract. b. They block the activity of the cytokines that stimulate bone reabsorption. c. They bind to bone surfaces to inhibit osteoclast activity. d. They stimulate osteoblasts and increase bone formation. e. They mimic estrogen's effect on bone

C Bisphosphonates work by binding to the bone surface and inhibiting osteoclastic activity. Vitamin D increases absorption of calcium in the GI tract. Estrogen and selective estrogen receptor modulators (raloxifene or Evista) work by blocking the activity of cytokines. Fluoride stimulates osteoblasts, but does not result in the formation of normal bone

You are treating a patient for chronic pain. She is taking NSAIDs, anticonvulsants, and a fairly high dose of long-acting opioid, but her pain is becoming increasingly hard to control. You are reluctant to increase her dose of narcotic, and she is already on maximal doses of her other therapies. Assuming the patient's pain is legitimate, which of the following options is most appropriate? a. Discontinue the opioids. b. Increase the opioids. c. Change to a lower dose of a different opioid. d. Add a second anticonvulsant. e. Add an antidepressant.

C In chronic nonmalignant pain, there is evidence that continued escalating opioid doses results in worsened analgesic response. This is because NMDA receptors are upregulated and lead to tolerance, while pain receptors become increasingly more sensitive to stimuli. In situations of tolerance to medication, it is appropriate to switch from one opioid agent to another, usually starting at half the equivalent dose of the alternative medication. Stopping the opioid would result in withdrawal, and increasing the dose would be inappropriate because of the physiologic effects described above. There is no evidence to support the addition of a second anticonvulsant, and although antidepressants have been shown to help in chronic pain, it is more likely that changing opioid would provide better pain control

You care for a patient who contracted hepatitis C after a blood transfusion many years ago. Her liver disease has progressed, and she now has end-stage disease. Which of the following will be the most likely cause of death in this patient? a. Liver failure b. Hepatocellular carcinoma c. Bleeding varices d. Encephalopathy e. Renal failure

C Varices occur secondary to chronic high pressure in the portal veins. Bleeding from varices is the most common cause of death in cirrhotic patients. The other potential causes of death listed are less common.

You are seeing a 67-year-old man who is following up in your office 2 days after being involved in a motor vehicle collision. He was the restrained driver and was going 45 mi/h when he ran into a car that was stopped in front of him. Thankfully, he denies head injury or loss of consciousness. He was ambulatory after the event and only had slight neck pain, but was taken to the emergency department for evaluation. They released him that evening without any imaging studies. On follow-up with you, he denies neck pain or numbness and tingling in his extremities. His range of motion is appropriate for his age. What testing, if any, should be done at this time? a. No testing is needed. b. He should have C-spine radiographs. c. He should have a CT scan of the neck. d. He should have an MRI of the neck. e. He should have an EMG.

C spine radiographs

You are screening a 35-year-old woman who presents with tachycardia, nervousness, tremor, palpitations, heat intolerance, and weight loss. You suspect Graves disease. What single test is best for differentiating Graves disease from other causes of hyperthyroidism? a. TSH b. TSH with free T 4 and free T 3 c. Thyroid receptor antibodies d. Radionucleotide imaging of the thyroid e. Thyroid ultrasound

C. Thyroid receptor antibodies are very specific and differentiate Graves disease from other causes 306 Family Medicine of hyperthyroidism. The TSH and free thyroid hormones are nonspecific, and only identify hyperthyroidism. Radionucleotide imaging is helpful in Graves, showing diffuse uptake, but is not necessarily specific. Thyroid ultrasonography can identify nodules, but is also a nonspecific test for differentiating causes of hyperthyroidism.

You are caring for a type 1 diabetic who has been hospitalized with diabetic ketoacidosis, and determining an appropriate insulin regimen for her. She has required 60 units of insulin per day to maintain adequate control in the hospital. You decide to use insulin glargine (Lantus) and aspart (Lispro) in combination. What should her Lantus dose be? a. 10 units b. 20 units c. 30 units d. 40 units e. 50 unit

C. 30 units When using Lantus and Lispro, approximately 40% to 50% of the total daily insulin requirements should be given as Lantus, with the remaining 50% to 60% of insulin given as Lispro before each meal, based on a preprandial glucose reading

You are caring for a 38-year-old woman with a long history of intravenous drug abuse. She was diagnosed with HIV 2 years ago, and has been doing well on therapy without disease progression. You order a purified protein derivative skin test for tuberculosis (TB). What amount of induration indicates a positive test? a. Any induration indicates a positive test. b. 3 mm. c. 5 mm. d. 10 mm. e. 15 mm

C. 5mm is considered positive test in HIV Usually, induration of 15 mm (10 mm in high-risk patients) indicates a positive test. In HIV-infected individuals, 5 mm is considered a positive test. PPD tests should be placed on the initial visit in HIV-infected patients, as there is an increased risk of progression from latent to active TB in infected individuals. Some high-risk individuals (homeless individuals or those who use injection drugs) should be tested annually.

A 48-year-old male patient suffered from a stroke. After full recovery, he follows up at your office. Which of the following medication options has been proven to lower his blood pressure and prevent recurrent stroke?

C. ACEI + Diuretic

You are caring for a 48-year-old construction manager with a history of chronic back pain due to osteoarthritis of the lumbar spine. His symptoms have been controlled on nonsteroidal anti-inflammatory medications (NSAIDs) for several years, but they are no longer as effective as they once were. Imaging studies have not changed and his laboratory work is normal. You are considering adjusting his pain control regimen. Of the following, which would be the best option? a. Add a muscle relaxant to his NSAID. b. Add an opioid to his NSAID. c. Add a tricyclic antidepressant to his NSAID. d. Add a slelctive serotonin reuptake inhibitor to his NSAID. e. Schedule him for facet joint corticosteroid injections

C. Add a TCA to his NSAID regimen

You are caring for a 56-year-old woman who cares for her 78-year-old mother-in-law who suffers with Alzheimer disease. She asks about her risk for the disease. Of the following, which is the strongest risk factor for the development of Alzheimer disease? a. Family history b. Female gender c. Advancing age d. Low levels of education e. Cardiovascular risk factors

C. Advancing Age Epidemiologic studies have identified several genetic and environmental risk factors for Alzheimer disease. Increasing age is the strongest risk factor. By age of 65, 1% of the general population meets diagnostic criteria, and the prevalence doubles every 5 years thereafter. Family history is another major risk factor, and individuals with a first-degree relative with the disease are four times more likely to develop the disease themselves. Other risk factors include female gender, low levels of education, cardiovascular risk factors, and a history of head trauma

You are treating a 55-year-old obese diabetic for his neuropathy. It's extremely painful and not responsive to NSAID therapy. Of the following, which is the best option for pain control? a. Celecoxib, 200 mg daily b. Tramadol, 50 mg q4-6h c. Amitriptyline, 50 mg at night d. Oxycodone 15-30 mg q4-6h e. Fentanyl patch, 25 μg/q72h

C. Amitryptiline There are many agents to choose from to treat chronic neuropathic pain. If the pain is unresponsive to NSAIDs, one can choose a trial of a COX-2 inhibitor, but studies have shown that tricyclic antidepressants like amitriptyline are efficacious. Anticonvulsants would be another option. Tramadol and opioids may work to control the pain, but may increase risk of addiction and would not be best as a next step in this case

You are seeing a 36-year-old man complaining of shortness of breath. He reports symptoms associated with activity and relieved by rest. He is otherwise healthy, takes no medications, and denies chest pain or pressure. Which of the following is the best way to describe what the patient is feeling? a. Classic angina b. Atypical angina c. Anginal equivalent d. Nonanginal pain e. Atypical nonanginal pain

C. Anginal Equivalent An anginal equivalent occurs when a patient has no chest pain, but has other symptoms of cardiac ischemia (eg, dyspnea) that is predictably precipitated by exertion and relieved by rest. Atypical angina occurs when the patient experiences pain that has the quality and characteristics of angina, or occurs with 298 Family Medicine exertion, but not both. Nonanginal pain has neither the quality nor the precipitants of angina. "Atypical nonanginal pain" is not a term used to describe chest pain.

You are seeing a patient with a long-standing HIV infection. The patient has been unable to afford his medication regimen and has been off medication for several months. He presents with shortness of breath. Blood gasses obtained emergently reveal a PaO 2 of 60 mm Hg. His chest x-ray is shown below . Assuming the patient is not allergic, which of the following is the best first-line treatment? a. Azithromycin b. Trimethoprim-sulfamethoxazole c. Trimethoprim-sulfamethoxazole and corticosteroids d. Triple-drug treatment against TB e. Quadruple-drug treatment against TB

C. Bactrim The x-ray shown is suspicious for Pneumocystis pneumonia and treatment should be started immediately. The treatment of choice is trimethoprim-sulfamethoxazole (TMPSMX) for 3 weeks. Although it would seem that corticosteroids should be avoided in HIV-infected patients, it has been shown to improve the course of patients with moderate to severe pneumocystic pneumonia with an oxygen saturation less than 90% or a PaO 2 less than 65 mm Hg. Therefore, this patient should receive both

You decide to treat a 72-year-old man for Alzheimer dementia. You choose to use donepezil (Aricept), and begin therapy. With respect to disease progression, which of the following statements best describes donepezil's effect on Alzheimer dementia? a. It dramatically slows the progression of neurodegeneration. b. It modestly slows the progression of neurodegeneration. c. It has no effect on the progression of neurodegeneration. d. It modestly increases the progression of neurodegeneration. e. It dramatically increases the progression of neurodegeneration

C. It has no effect on the progression of neurodegeneration Three cholinesterase inhibitors are approved for the treatment of Alzheimer disease. They include donepezil (Aricept), galantamine (Reminyl), and rivastigmine (Exelon). They reduce the metabolism of acetylcholinesterase, thereby prolonging its action at cholinergic synapses. They are associated with modest improvements in cognition, behavior, activities of daily living, and global measurements of functioning. However, they do not change the progression of neurodegeneration

You are assessing a 59-year-old patient with an 80-pack-year history of smoking cigarettes. He stopped smoking 1 year ago. He reports a cough productive of white frothy sputum for the past 4 months. Reviewing his chart, you discover that he had a similar presentation last winter, with a cough that lasted more than 3 months. Given this information, which of the following tests is necessary for him? a. Complete blood count b. Arterial blood gas measurements c. Office spirometry d. Computerized tomographic scans of the chest e. An electrocardiogram

C. Office Spirometry

You are evaluating a 47-year-old man with erectile dysfunction. After a thorough history and physical examination, you order a morning free testosterone level. His level was low. What is the most appropriate next step? a. Begin testosterone injections. b. Begin topical testosterone replacement. c. Obtain follicle-stimulating hormone (FSH), luteinizing hormone (LH), and prolactin levels. d. Obtain a penile brachial index. e. Perform a nocturnal penile tumescence evaluation.

C. Order FSH LH and prolactin to determine the source if FSH/LH low w/ nl prolactin, then pit/hypo failure if FSH/LH high w/ nl prolactin, testicular failure if FSH/LH low, w/ high prolactin, then pituitary adenoma, order CT

You have been treating a patient for chronic pain since she finished her chemotherapy for breast cancer. She describes the pain as "pins and needles" in her lower legs. Based on her comments, what is the best description for this pain? a. Hypoesthesia b. Hyperesthesia c. Paresthesia d. Allodynia e. Nociceptive pain

C. Parasthesia Pain is the most common reason for which people seek medical care. Chronic pain is defined as recurrent or persistent pain lasting more than 3 months, and it affects around 15% of the US population. Nociceptive pain stems from tissue damage (such as arthritis and/or tumor). Neuropathic pain, as that described in this question, results from the sustained transmission of pain signals in the absence of ongoing tissue damage. It can be described as numbness (hypoesthesia), increased sensitivity (hyperesthesia), pins and needles (paresthesia), or severe pain usually from innocuous stimuli (allodynia)

You are caring for a 26-year-old man with dyslipidemia and a family history of early coronary arterial disease. Laboratory analysis reveals a low HDL. Which of the following interventions, if adopted by the patient, would raise his HDL levels to the greatest extent? a. Eat oat bran b. Lose weight c. Start exercising d. Quit smoking e. Reduce life stress

C. Start Exercising Several lifestyle modification efforts are known to increase HDL cholesterol. By losing weight, a person can expect to raise HDL by 5 to 10 points. Smoking cessation has the same approximate effect. Adopting an exercise program is even more effective, raising HDL by up to 15 points. Eating oat bran and decreasing life stress can lower LDL, but is not likely to raise HDL. Additionally, alcohol, in moderation, raises HDL cholesterol.

You have chosen to treat a 70-year-old man with ischemic heart disease using a β-blocker. Which of the following is the most appropriate endpoint for the use of β-blockers in this case? a. Use no more than the equivalent of 40 mg twice daily of propranolol. b. Use the amount necessary to achieve a blood pressure of 100/70 mm Hg or less. c. Use the amount necessary to keep the heart rate between 50 and 60 beats/min. d. Increase the dosage until fatigue limits use. e. Increase the amount until angina disappears.

C. Use the amount necessary to keep the heart rate between 50 and 60 beats/min All β-blockers, regardless of their selectivity, are equally effective in treating angina. About 20% of patients do not respond. The dose should be adjusted to achieve a heart rate of 50 to 60 beats/min.

Your patient is thinking of using varenicline (Chantix) to help with smoking cessation. Which of the following is true of this medication? a. Varenicline has many drug interactions. b. The dose of this medication would need to be changed in patients with liver disease. c. Common side effects include abnormal dreams. d. This medication is contraindicated in patients with a seizure disorder. e. Patients cannot smoke while taking varenicline.

C. abnormal dreams from varenciline (Chantix)

You are caring for a young woman who has had mild intermittent asthma for years. She uses a short-acting bronchodilator as needed, but in the past has only needed therapy once or twice a month. Over the past 2 months, she has noted that she is using her inhaler more. In fact, she uses it at least three times a week and on occasion has had to wake up in the middle of the night to use her inhaler. Which of the following is the most appropriate treatment option at this point? a. Change her short-acting β-agonist from albuterol (Proventil, Ventolin) to pirbuterol (Maxair). b. Add a long-acting β-agonist. c. Add an inhaled corticosteroid. d. Add a leukotriene receptor antagonist. e. Add cromolyn (Intal).

C. add inhaled CST (for persistent asthma) leukotriene = 2nd best option both ICST and leukotriene have replaced Cromolyn in current asthma therapy

You are seeing a 43-year-old hypertensive patient in your office. He is well-controlled with hydrochlorothiazide and is seeing you for a routine evaluation. His blood pressure at the visit is 118/76 mm Hg. Laboratory evaluation reveals a normal creatinine and a GFR greater than 90 mL/min, but he does have microalbuminuria. Which of the following interventions is indicated in this patient? a. Commend him on his excellent control and make no changes. b. Work to achieve better blood pressure control through diet and exercise. c. Increase his hydrochlorothiazide dose. d. Add an angiotensin-converting enzyme (ACE) inhibitor. e. Check a glycosolated hemoglobin level.

D. Add an ACE

You suspect that a 50-year-old female patient is abusing alcohol. Which of the following is the most *sensitive* laboratory test to confirm this? a. Mean corpuscular volume (MCV) b. Alanine aminotransferase (ALT) c. Aspartate aminotransferase (AST) d. γ-Glutamyl transferase (GGT) e. Ethyl glucuronide (EtG)

D. GGT is the most sensitive, and is elevated in a bunch of different disorders

You are concerned that one of your 65-year-old patients is developing dementia. Which of the following, if present, would lead you to suspect dementia rather than delirium or depression? a. Acute onset of symptoms b. Difficulty with concentration c. Signs of psychomotor slowing d. Good effort with testing, but wrong answers e. Patient complaint of memory loss

D. Good effort with testing but wrong answers Dementia is often difficult to distinguish from delirium or depression in the elderly. Delirium: generally acute in onset and associated with a loss of concentration. Dementia's onset is insidious, and concentration is less likely to be a problem. Depression is associated with psychomotor slowing, while dementia is generally not. While people with dementia may complain of memory loss, it is far more likely that the patient's family will complain of the patient having memory loss in dementia. Depressed patients usually present themselves complaining of memory loss. Depressed and delirious patients will generally show poor effort in testing, while demented patients will generally display good effort, but get wrong answers.

A 49-year-old African-American perimenopausal woman is seeing you after having fractured her wrist. Her past medical history is significant for oral contraceptive use for 20 years, obesity, and Graves disease leading to current hypothyroidism. She nursed two children for 6 months each. Which component of the patient's history puts her at increased risk for osteoporosis? a. African-American race b. Oral contraceptive use c. Obesity d. Graves disease e. Breast-feeding

D. Graves Disease Osteoporosis is because of poor acquisition of bone mass or accelerated bone loss. African Americans are less at risk than Caucasians or Asians. There is no evidence that oral contraceptive use increases risk. Obesity is considered to be protective because of increased estrogen production, as long as the person is not sedentary. Hyperthyroidism is a common cause of accelerated bone loss. Breast-feeding is a significant drain on calcium stores, but studies have shown that the associated bone mineral loss is completely reversed within 12 months of weaning

You are caring for a patient with a poor lipid profile. His HDL is low, his LDL is high, and his triglycerides are also high. Which of the following medications would have the most beneficial effect on his triglycerides? a. Lovastatin b. Colestipol c. Ezetimibe d. Fenofibrate e. Cholestyramine

D

A 48-year-old woman has been treated for type 2 diabetes for 6 years with metformin 2000 mg daily, and glyburide 10 mg daily. She is modestly compliant with her diet, medications, and exercise. She is 69 in tall and weighs 278 lb. Her most recent HbA 1C is 8.2% which has been relatively unchanged over the past 18 months. You are considering adding exenatide (a GLP-1 [glucagon-like peptide 1] receptor agonist) to her regimen. Which of the following most accurately reflects your patient's expected response to this addition to her regimen?

D Exenatide is a GLP-1 receptor agonist isolated from the saliva of the Gila monster. When it is given to patients with type 2 diabetes, it lowers blood glucose and HbA 1C. Adding it to a person taking metformin and a sulfonylurea further lowers HbA1C by 0.4% to 0.6%, and this is maintained up to 80 weeks. In addition, patients have the added benefit of weight loss, with the average weight loss for patients being around 10 lb.

You have prescribed niacin for a patient with elevated LDL and triglycerides. He reports nonadherence to this regimen because of significant flushing that occurs when he takes the medication. What would you recommend to avoid this side effect? a. Take the niacin at night. b. Take the niacin with food. c. Take the niacin with milk. d. Take aspirin before taking the niacin. e. Take a proton pump inhibitor before taking the niacin.

D Aspirin blocks much of the flushing that is associated with sustained-release niacin preparations. Taking niacin at night, with food, on an empty stomach or with milk, or with a proton pump inhibitor will not impact the side effects.

A 33-year-old woman is seeing you for weight management. At 5 ft 6 in tall and 230 lb, she reports a history of having difficulty with weight since her teenage years. The rest of her medical history is unremarkable. Using conventional dietary techniques, what is her chance of losing 20 lb and maintaining that weight loss for 2 years? a. 1% b. 5% c. 10% d. 20% e. 50%

D. 20% Unfortunately, only 20% of patients will lose 20 lb and maintain the weight loss for 2 years using conventional dietary techniques. Only 5% can maintain a 40 lb weight loss. Those who are successful report continued close contact with their health care provider. Most successful programs are multidisciplinary and include a low-calorie diet, behavior modification, exercise, and social support

Your patient asks you about pharmacotherapy to help him to prevent relapse of alcohol abuse. Which of the following medications is most effective for this purpose? a. Disulfiram b. Naltrexone c. Serotonergic drugs d. Acamprosate e. Tricyclics

D. Acamprosate

A 66-year-old diabetic man comes to your office with acute monoarticular arthritis. You suspect gout. Which of the following tests is the most helpful in establishing the diagnosis? a. Sedimentation rate b. C-reactive protein c. Serum uric acid levels d. Evaluation of joint aspirate e. Twenty-four-hour urine collection to measure uric acid excretion

D. evaluation of joint aspirate

A 44-year-old African American with type 2 diabetes transfers care to you. Reviewing her records, you find she is on the maximum dose of sulfonylurea, but her hemoglobin A 1C is 9.2% (H). Review of her baseline laboratory tests reveals normal liver enzymes and a creatinine of 2.3 mg/dL. Which of the following management options would be most beneficial? a. Change to another sulfonylurea. b. Add a biguanide. c. Add a meglitinide. d. Add a thiazolidinedione. e. Add an α-glucosidase inhibitor

D. Add thiazodelienione Oral therapy for type 2 diabetes can be complicated. No evidence supports changing sulfonylureas when one is not adequately controlling glucose levels. Biguanides act to decrease glucose output from the liver, and can decrease hemoglobin A 1C by 1.5% to 2%. However, biguanides should not be used if creatinine is higher than 1.5 mg/dL. Meglitinides increase insulin secretion and should only be taken before meals. They can reduce the hemoglobin A 1C by 0.5% to 2% and are most valuable if fasting sugar is adequate, but postprandial sugars are high. Since they increase insulin levels, they are more effective when used in combination with a medication that has a different mechanism of action. They are excreted in the liver, therefore are safe in renal failure. Thiazolidinediones decrease insulin resistance and are an excellent choice for those with insulin insensitivity. α-Glucosidase inhibitors inhibit the absorption of carbohydrates in the gut and can decrease the hemoglobin A 1C by 0.7% to 1%. They should be avoided if creatinine more than 2.0 mg/dL

You are discussing asthma control with a 22-year-old patient. She monitors her therapy closely and reports that her current peak flows are at about 80% of her best levels. Which of the following is the best approach to take at this point? a. Commend the patient on her diligent monitoring and excellent control. b. Reassure the patient that this is well within the normal range. c. Review the patient's medications and technique and review environmental control. d. Have the patient take additional medication, or add a medication to her regimen. e. Consider hospitalization.

D. Additional med or add med

You are treating a patient with the classic signs of dementia including cognitive decline. His caretaker reports that he has been having complex visual hallucinations and a tremor. On examination, he appears to have masked facies, has a slight tremor, and a shuffling gait. Which of the following medications should be avoided in this case? a. Cholinesterase inhibitors b. SSRI c. Tricyclic antidepressants d. Antipsychotics e. Benzodiazepines

D. Antipsychotics The patient described has dementia of Lewy body (DLB) type. This begins similarly to Alzheimer disease, but then patients develop complex visual hallucinations and spontaneous signs of parkinsonism. Patients with Alzheimer dementia develop delusions, but rarely have hallucinations. Antipsychotics should be avoided in this type of dementia, unless absolutely necessary, as there is concern for long-term neurologic damage. DLB responds to cholinesterase inhibitors, and the other medications listed are safe if used appropriately

You are treating a patient with COPD for an acute exacerbation. Assuming he has no allergies to medications, which of the following is true regarding antibiotic treatment in this case? a. He should be prescribed amoxicillin. b. He should be prescribed trimethoprim-sulfamethoxazole. c. He should be prescribed doxycycline. d. He should be prescribed azithromycin. e. No antibiotics are necessary.

D. Azithromycin

You are treating a patient with chronic and progressing renal disease. His GFR is decreasing and you fear he will need dialysis sometime within the next year. Which of the following will be the most likely cause of death in this patient? a. Renal failure b. Liver failure c. Neurovascular disease d. Cardiovascular disease e. Coagulopathy

D. CV Complications The long-term complications of chronic kidney disease are many. There is a higher risk for cardiovascular disease in this population as compared with the general population. In fact, patients with chronic kidney disease die, primarily due to cardiovascular disease, before reaching the need for dialysis. The reason for this is unclear, but may be related to the uremic milieu, underlying comorbidities and the hesitancy to perform diagnostic procedures in the setting of chronic kidney disease.

You are evaluating a 76-year-old woman on long-term glucocorticosteroid therapy for polymyalgia rheumatica. Which of the following is the diagnostic imaging test of choice to diagnose osteoporosis? a. Plain radiographs b. Single-photon absorptiometry c. Dual-photon absorptiometry d. Dual-emission x-ray absorptiometry (DEXA) scan e. Quantitative CT of bone

D. DEXA Plain radiographs are not sensitive enough to diagnose osteoporosis until total density has decreased by 50%. Single- and dual-photon absorptiometry provide poor resolution and are less accurate than other methods. DEXA scanning is most precise and is the test of choice. Quantitative CT scanning is the most sensitive, but exposes patients to significant levels of radiation.

You are following a 16-year-old girl with a suspected eating disorder. Which of the following, if present, would help differentiate anorexia nervosa from bulimia nervosa? a. Binge eating or purging. b. The use of laxatives, diuretics, or enemas. c. Self-evaluation is unduly influenced by body weight and shape. d. Episodic lack of control over eating. e. Inappropriate behaviors to prevent weight gain.

D. Epsidoic lack of control of eating Eating disorders are psychologic disorders in which the person afflicted has an altered perception of body weight or shape and disturbances of eating behavior. Distinguishing between anorexia and bulimia may be important from a treatment standpoint. Some characteristics are common to both eating disorders, while other characteristics may help to differentiate them. Both disorders involve self-evaluation that is unduly influenced by body weight and/or shape. While binge eating or purging are considered characteristics of bulimia, there is a binge eating/purging subtype of anorexia that involves that behavior as well. Both bulimics and binge eating/purging subtypes of anorexics may use diuretics, enemas, and laxatives. Both engage in inappropriate behaviors to prevent weight gain. However, bulimics sense a lack of control over eating during episodes of binging, while anorexics often feel a strong sense of control. This is a characteristic that may help distinguish the two

You are caring for a patient with a poor lipid profile. His HDL is low, his LDL is high, and his triglycerides are also high. Which of the following medications would have the most beneficial effect on his HDL? a. Lovastatin b. Colestipol c. Ezetimibe d. Fenofibrate e. Cholestyramine

D. Fenofibrate The different options for medical management of hyperlipidemia include preparations that affect the total cholesterol, the HDL, the LDL, and the triglycerides. Choice of medication depends on the desired endpoint. The following table outlines the expected increase in HDL that would be expected using the medications in the answer key

You are caring for a 56-year-old man who presents to you for an evaluation of chest pain. You determine that an exercise treadmill test is necessary. The patient completes stage III of a Bruce protocol, achieves a heart rate of 136 beats/min and has an ST-segment depression of 1 mm in the three inferior leads at a heart rate of 130 beats/min. These changes lasted 2 minutes into recovery. Which of the following features is a poor prognostic sign for the patient? a. Being unable to reach stage IV of a Bruce protocol b. Failure to achieve a heart rate of 140 beats/min c. Onset of ST-segment depression at a heart rate of 130 beats/min d. Having ST-segment depression in multiple leads e. Having ST-segment depression lasting 2 minutes into recovery

D. Having ST-segment depression in multiple leads Poor prognostic signs in an ETT include failure to complete stage II of a Bruce protocol, failure to achieve a heart rate greater than 120 beats/min (off β-blockers), onset of ST-segment depression at a heart rate less than 120 beats/min, having ST-segment depression greater than 2.0 mm, having ST-segment depression lasting more than 6 minutes into recovery, poor systolic blood pressure response to exercise, angina or ventricular tachycardia with exercise and ST-segment depression in multiple lead

Through counseling and education, you have convinced a 35-year-old man with dyslipidemia to quit smoking. If he remains a nonsmoker, how would you expect his lipid profile to change? a. His total cholesterol will decrease. b. His LDL will decrease. c. His fasting triglycerides will decrease. d. His HDL will increase. e. His VLDL will decrease.

D. His HDL will increase Smoking cessation increases HDL by 5 to 10 mg/dL, but does not affect LDL, VLDL, or triglycerides.

You are managing a 36-year-old woman with a new diagnosis of type 2 diabetes. Her hemoglobin A 1C was 7.2% at diagnosis. Her subsequent sugars were well-controlled using metformin, 1000 mg twice daily. At her visit 3 months later, her blood pressure is 100/72 mm Hg, her hemoglobin A1C is 6.0%, but her microalbumin screen is positive. Which of the following is the most appropriate response? a. Continue weight loss and recheck in 3 months. b. Limit dietary protein intake. c. Intensify diabetic therapy to more tightly control glucose. d. Initiate therapy with an ACE inhibitor. e. Refer to nephrology

D. Initiate therapy with an ACE-I The first indication of renal compromise in diabetics is an increase in GFR. Renal lesions develop and are followed by microalbuminuria. Uncorrected, this can lead to macroalbuminuria, then renal failure. ACE inhibitors have been shown to decrease end-stage renal disease and death by 41% in diabetics. Lifestyle changes including glucose control, weight loss, and decreased protein intake can help, but experts agree that the benefits of ACE inhibitors are well-documented. Nephrology referral would be indicated if the creatinine becomes elevated, or in the face of macroalbuminuria or microalbuminuria despite maximal therapy. Even in patients who are normotensive, low-dose ACE inhibitors are beneficial in the face of microalbuminuria

You are working with a 44-year-old man with difficult-to-manage dyslipidemia. He is taking atorvastatin (Lipitor) at maximum dosages, and you are considering adding ezetemibe (Zetia) to improve the lipid profile. How does ezetemibe work to help lower cholesterol? a. Sequestration of bile acids b. Changing hepatic metabolism of lipoproteins c. Inhibits HMG-CoA reductase d. Interferes with cholesterol absorption in the gut e. Decreases secretion of triglycerides by the liver

D. Interferes with cholesterol absorption in the gut Ezetemibe (Zetia) lowers cholesterol by interfering with the absorption of cholesterol in the gut. Used alone, it lowers LDL and triglycerides only modestly. When added to a low-dose statin, the combination lowers LDL as much as the maximum statin dose, but its combined use with a low-dose statin may produce fewer adverse effects.

436. You are doing a screening physical examination for a 40-year-old female patient. She does not have diabetes or known coronary artery disease. Based on National Cholesterol Education Program risk calculation, you determine that her risk for coronary disease is less than 10% in the next 10 years. Given this information, what is the patient's LDL treatment goal? a. Less than or equal to 70 mg/dL. b. Less than or equal to 100 mg/dL. c. Less than or equal to 130 mg/dL. d. Less than or equal to 160 mg/dL. e. There is no recognized treatment goal

D. Less than or equal to 160 mg/dL Treatment goals for persons with dyslipidemias should be established based on the patient's clinical status and other risk factors. If a patient has no known coronary disease, the 10-year risk for coronary disease should be estimated using a readily available National Cholesterol Education Program (NCEP) risk calculator (available online). If the 10-year risk is greater than 20%, the LDL treatment goal should be less than or equal to 100 mg/dL. If the risk is between 10% and 20%, the LDL treatment goal should be less than 130 mg/dL. If the risk is below 10%, the treatment goal should be less than 160 mg/dL.

You are caring for a patient who appears to have advanced Alzheimer dementia. Which of the following medications has been shown to result in statistically significant benefit in advanced cases of dementia? a. Donepezil b. Galantamine c. Rivastigmine d. Memantine e. Ginkgo biloba

D. Memantine Evidence from clinical trials has shown that donepezil can result in modest clinical improvement versus placebo in community-dwelling patients with dementia, but found no difference in the rates of institutionalization or progression of disease. Patients with more advanced disease have been shown to have statistically significant benefit from Memantine, with or without concomitant use of an acetylcholinesterase inhibitor. Ginkgo biloba has been shown to have mixed results in studies.

You have been treating a 68-year-old man suffering from chronic CHF with furosemide (Lasix), a β-blocker, and an ACE inhibitor. Despite this therapy, he continues with refractory edema. In his baseline state, he is comfortable at rest, but experiences some symptoms of heart failure with ordinary activity. Which of the following would be the best diuretic to add? a. Hydrochlorothiazide b. Triamterene c. Hydrochlorothiazide and triamterene combined (Dyazide, Maxzide) d. Metolazone (Zaroxolyn) e. Spironolactone (Aldactone)

D. Metazolone Some patients have difficulty maintaining optimal fluid balance, and a second diuretic is needed. In this case, adding metolazone can significantly increase diuresis in the outpatient treatment of heart failure with volume overload. Prolonged therapy should be avoided. Hydrocholorozide would not enhance diuresis, nor would triamterene. Spironolactone can be used, but is usually only considered for NYHA class III or IV patients or those with a serum potassium level less than 5.0 mmol/L

You are seeing a 44-year-old woman with a known history of asthma who has had recent complaints of chest pain. She reports a stabbing pain that seems to be worse with inspiration. It is not associated with activity, but will occur intermittently throughout the day. Which of the following is the best way to describe what the patient is feeling? a. Classic angina b. Atypical angina c. Anginal equivalent d. Nonanginal pain e. Atypical nonanginal pain

D. Nonanginal pain has neither the quality nor the precipitating features of angina. Typical descriptive terms of nonanginal pain include "stabbing," "shooting," "knifelike," "jabbing," and "tingling." Atypical angina occurs when the patient experiences pain that has the quality and characteristics of angina, or occurs with exertion, but not both. Anginal equivalent occurs when dyspnea is the sole or major manifestation. "Atypical nonanginal pain" is not a term used to describe chest pain

You are evaluating a patient with a painful, swollen knee. Joint aspirate reveals clear fluid with a WBC count of 5000/mm 3, 20% of which are PMN leukocytes. Which of the following is the most likely diagnosis? a. Gout b. Pseudogout c. Infectious arthritis d. Osteoarthritis e. Rheumatoid arthritis

D. Osteoarthritis

You are taking care of a 22-year-old woman with fever, aches, and fatigue. Her history reveals intravenous drug abuse, and you suspect acute HIV infection. Which of the following tests is best to rule out acute HIV? a. Enzyme-linked immunosorbent assay (ELISA) b. Western blot c. Immunofluorescent antibody test d. Quantitative plasma HIV RNA (viral load) e. CD4 lymphocyte count

D. Quantitative plasma HIV RNA Testing to establish the diagnosis of HIV infection usually requires an ELISA followed by a confirmatory Western blot of immunofluorescent antibody test. However, there is a "window period" of several weeks to 4 months between the infection and seroconversion when these tests may be negative. During this time, patients may be viremic and infectious, but not have sufficient levels of antibodies to result in positive tests. If there is strong clinical suspicion, plasma HIV RNA should be ordered. This, however, also needs to be interpreted with caution, as low level viremia may represent a false-positive test. CD4 count is not helpful in acute HIV disease.

You are working with an obese patient to help him lose weight. You are considering the use of orlistat (Xenical, Alli) to help the patient with weight reduction. Which of the following is the mechanism of action for this medication? a. It is an appetite suppressant. b. It blocks the uptake of both serotonin and norepinephrine in the central nervous system. c. It is a selective cannabinoid-1 receptor antagonist. d. It reduces fat absorption in the GI tract. e. It is a catecholaminergic amphetamine.

D. Reduces fat absorption in GI tract Medications to treat obesity are available over the counter and by prescription. While controversy exists, the NIH clinical guidelines state that medications may be used as part of a comprehensive weight management plan. Appetite suppressants can be amphetamines (but those carry a significant risk for abuse) or nonamphetamine. Sibutramine (Meridia) is a prescription serotonin/norepinephrine blocker. There is a selective cannabinoid-1 receptor antagonist called rimonabant under investigation that looks promising, but future 300 Family Medicine studies will help determine its place in the management of obesity. Orlistat blocks fat absorption from the GI tract

394. You are following a 54-year-old patient with hypertension and diabetes in your office. Despite good blood pressure and glycemic control, his GFR has started to decrease. GFR measurement was 74 mL/min 3 months ago. At this visit, GFR is 55 mL/min. Creatinine is within normal limits, and his serum potassium is 5.2 mmol/L (normal is up to 5.1 mmol/L). The patient denies any changes in urination or other problems. Which of the following is most appropriate at this stage? a. See the patient more frequently, at least monthly. b. Increase his ACE inhibitor. c. Add diuretic therapy. d. Refer to a nephrologist. e. Refer to a transplant surgeon.

D. Refer to nephro The patient's laboratory values and clinical picture is consistent with moderate renal failure (National Kidney Foundation stage 3). At this point, nephrology referral is indicated. Renal replacement therapy (transplant or dialysis) is indicated for severe renal insufficiency (GFR <15 mL/min).

You are examining a 24-year-old patient for the first time and find her blood pressure to be 155/92 mm Hg. On examination, you find that she has a very weak femoral pulses. Which of the following is true regarding this patient? a. Her blood pressure is not high enough to consider correction of her anatomical defect. b. She is too young to consider intervention regardless of her blood pressure. c. Correction of her deficit should eliminate the need for blood pressure medication. d. She is likely to have a bicuspid aortic valve. e. Her chest radiograph is likely to be normal.

D. She is likely to have a bicuspid aortic valve The patient in this question has coarctation of the aorta. The usual presentation is hypertension. Hypertension is present in the arms, but is low or normal in the legs. Femoral pulsations are weak or absent. Correction of the defect should be considered if the gradient is greater than 20 mm Hg, not based on level of blood pressure. Age is not a consideration for repair. Oftentimes, even after surgical correction, patients continue to be hypertensive for years, based on permanent changes in the rennin-angiotensin system. Her ECG will likely show left ventricular hypertrophy, and her chest radiograph will likely show rib notching. More than 50% of people with coarctation also have bicuspid aortic valve

You are currently evaluating a patient for unstable angina. He takes phenytoin for a seizure disorder, has high cholesterol, and is a current smoker. Which of the following would be the best therapeutic option to help with his smoking cessation plan? a. Behavioral intervention b. Nicotine replacement c. Bupropion d. Varenicline e. Clonidine

D. Verenciline is the best/most effective

You are evaluating a 74-year-old woman for the recent onset of incontinence. She has diabetes, controlled by diet but with recently increasing sugars, and hypertension, controlled with a combination of lisinopril/ hydrochlorothiazide. She has complained of constipation recently and has not had a bowel movement for 3 days. Microscopic analysis of her urine is positive for bacteria, but she does not report dysuria, urgency, or frequency. Which of the historical features mentioned is inconsequential in the workup of her incontinence? a. Hyperglycemia b. Diuretic use c. Constipation d. Bacteruria e. Postmenopausal state

D. bacteriuria

You are discussing treatment options for a 43-year-old woman with major depressive disorder. Which of the following is a true statement regarding the effectiveness of treatment for depressive disorders? a. Only about 25% of patients that receive medication alone will find the medication to be effective. b. Patients who find one medication ineffective are likely to find all medications ineffective. c. In order to prevent a relapse of depressive symptoms, patients should continue treatment for 3 to 4 months. d. In general, patients respond best to the combination of medication and counseling. e. Electroconvulsive therapy (ECT) is ineffective when compared with newer medical therapy.

D. meds + CBT Physicians have various treatment options for depression. Studies have shown that the combination of medication and therapy offer the best treatment outcomes. However, antidepressants alone are effective in about 50% to 60% of patients with major depression. If a patient fails to respond to one medication, he or she may respond to another. At least 80% of patients with major depression will respond to at least one antidepressant medication. In order to prevent relapse, treatment should continue for 6 to 9 months. ECT has a high rate of therapeutic success, but is reserved for those who do not respond to other modalities of treatment.

*******A 42-year-old man is seeing you to discuss sexual concerns. He complains of being unable to achieve an erection, despite having strong interest in sexual activity. Which of the following is true? a. This is most often because of an unrecognized mood disorder. b. This is most often because of a lack of attraction for his partner. c. This is most often because of stressors in the home and interpersonal conflict. d. This is most often because of a vascular problem. e. This is most often because of alcohol abuse.

D. most commonly a vascular issue

In an attempt to lower cholesterol through diet, you recommend that a 40-year-old man take fish oil. What is the lipid-lowering mechanism of action of fish oil? a. Sequesters bile acids b. Changes hepatic metabolism of lipoprotein c. Inhibits HMG-CoA reductase d. Interferes with cholesterol absorption in the gut e. Decreases secretion of triglycerides by the liver

E Fish oil is high in omega-3 fatty acids and has been shown to be beneficial in lowering cholesterol. Fish oils work by decreasing secretion of triglycerides by the liver.

449. You are seeing an HIV-positive patient who presents to discuss his current HIV therapy. His viral load is increasing and his CD4 count is falling. While obtaining his blood sample, your medical assistant sustained a needle stick injury. What is the best course of action in this situation? a. Immediately test the medical assistant for HIV antibodies and begin treatment if positive. b. Immediately test the medical assistant for HIV viral load and begin treatment if detectable. c. Immediately test the medical assistant for HIV antibodies and begin zidovudine therapy. d. Immediately test the medical assistant for HIV viral load and begin zidovudine therapy. e. Immediately test the medical assistant for HIV antibodies and begin at least two drug therapy.

E Immediately test the medical assistant for HIV antibodies and begin at least two drug therapy Postexposure prophylaxis against HIV can substantially decrease the risk of seroconversion after a needle stick injury. Health care workers should be tested for HIV as soon as possible after the needle stick to establish a negative baseline for potential worker's compensation claim should the worker subsequently seroconvert. Therapy should be initiated using at least two medications to which the source would unlikely be resistant. Some clinicians prefer triple therapy

You are caring for an obese 30-year-old woman who would like to consider pharmacotherapy for the treatment of her obesity. Which of the following medications, if any, demonstrates maintenance of weight loss once off the medication? a. Orlistat (Xenical) b. Phentermine c. Sibutramine (Meridia) d. Rimonabant e. None of the medications lead to maintenance of weight loss once off the medication

E. Several medications are approved in the United States for the pharmacotherapy of obesity. Unfortunately, none consistently demonstrate maintenance of weight loss once the medications are discontinued. Orlistat is a gastrointestinal lipase inhibitor and boasts a 9% average weight loss, but a significant regain after medications are discontinued. Phentermine is a noradrenergic agonist that allows its users to lose 3 to 4 kg more than placebo, but regain is also very common. Sibutramine is a mixed noradrenergic/serotonergic agonist that demonstrates a 5% to 8% weight loss, but a significant regain of weight once off medication. Rimonabant is a cannabinoid receptor antagonist approved in the United Kingdom that demonstrates 4 to 5 kg more weight loss than placebo, but demonstrates almost 100% regain of weight once medication is discontinued.

An obese 29-year-old woman is complaining of polyuria. Her workup, including serum glucose, is negative. She is not taking any prescription medications. Which of the following, if present in her history, is the most likely cause? a. Marijuana abuse b. Over-the-counter diet pill use c. Over-the-counter decongestant use d. Over-the-counter sleeping pill use e. Caffeine overuse

E. caffeine

You have seen a 36-year-old man with elevated blood pressure. On one occasion, his blood pressure was 163/90 mm Hg, and on a second occasion, his blood pressure was 158/102 mm Hg. You have encouraged lifestyle modifications including weight loss using exercise and dietary changes. Despite some modest weight loss, at his current visit, his blood pressure is 166/92 mm Hg. Which of the following is the best treatment strategy at this point? a. Use a thiazide diuretic. b. Use an ACE inhibitor. c. Use an angiotensin receptor blocker. d. Use a β-blocker. e. Use a two-drug combination of medications

E. 2 drug regimen combo of meds The patient in this question has stage 2 hypertension (systolic blood pressure ≥160 mm Hg, or diastolic blood pressure ≥90 mm Hg). Since lifestyle modifications have not helped, the next step is to institute drug therapy. JNC 7 guidelines state that in patients with stage 2 hypertension, two-drug combination therapy is indicated. The most common regimen would be a thiazide diuretic along with either an ACE inhibitor, ARB, β-blocker, or calcium channel blocker.

You are evaluating a 45-year-old man with liver disease. His laboratory evaluation reveals the following: AST: 52 U/L (H) ALT: 56 U/L (H) Alkaline phosphatase: 132 U/L (H) GGT: 188 U/L (H) Albumin: 2.9 g/dL (L) Bilirubin: 3.5 mg/dL (H) Prothrombin time: 14.9 seconds (H) Which of his laboratory results suggests that his liver disease is chronic? a. AST b. ALT c. GGT d. Alkaline phosphatase e. Albumin

E. ALbumin Laboratory studies that represent acute hepatocellular injury include AST, ALT, LDH, and alkaline phosphatase. Laboratory values that represent hepatic function include albumin, bilirubin, and prothrombin time. Tests of hepatic function are more suggestive of chronic disease as opposed to acute injury.

You have diagnosed a 35-year-old African-American man with hypertension. Lifestyle modifications helped reduce his blood pressure, but he was still above goal. You chose to start hydrochlorothiazide, 25 mg daily. This helped his blood pressure, but it is still 142/94 mm Hg. Which of the following is the best approach to take in this situation? a. Increase his hydrochlorothiazide to 50 mg/d. b. Change to a loop diuretic. c. Change to an ACE inhibitor. d. Change to a β-blocker. e. Add an ACE inhibitor.

E. Add ACEI The British Hypertension Society developed recommendations to help practitioners devise an optimal treatment regimen when combining antihypertensives. They recommend that persons younger than 55 years who are not black start an ACE inhibitor as first-line therapy (A). β-Blockers (B) can be used in this group, but are no longer considered ideal first-line therapy. In persons who are older than 55 years or black, the first-line therapy is either a calcium channel blocker (C) or a diuretic (D). If one medication does not control the blood pressure, the next step is to add an agent from the other category. For example, if you have an "A" or "B" medication, add a "C" or "D" medication. If that still doesn't control the blood pressure, use A (or B) + C + D. Those still resistant should consider an α-blocker or other agent

You are evaluating a 48-year-old man with liver disease. His laboratory evaluation is as follows: AST: 268 U/L (H) ALT: 114 U/L (H) Alk Phos: 140 U/L (H) Bilirubin: 2.3 mg/dL (H) GGT: 220 U/L (H) Which of the following is the most likely cause of his liver disease? a. Autoimmune hepatitis b. Hepatitis B c. Hepatitis C d. Hematochromatosis e. Alcoholic hepatitis

E. Alcoholic Hepatitis 2:1 Ratio of AST:ALT

You are caring for a 79-year-old woman with symptoms suggesting Alzheimer disease. Which of the following clinical features of Alzheimer disease is most likely to remain intact until the late stages of the disease? a. The ability to recall new information b. Word-finding ability c. The ability to draw complex figures (intersecting boxes or a clock) d. The ability to calculate (balance a checkbook) e. Appropriate social behavior

E. Appropriate Social Behavior Often, memory disturbances are the presenting symptom in Alzheimers disease. Remote memories are well-preserved initially, with the ability to recall new information being lost early in the illness. Difficulty with word-finding is also noted early. Decreased ability to recognize and draw complex figures is an early sign of problems, as is the loss of the ability to calculate. Social propriety and interpersonal skills often remain strikingly preserved until late in the illness.

You are treating a 48-year-old man for major depression. His medical history includes a head injury several years ago that has left him with a seizure disorder. Which of the following antidepressants would be contraindicated? a. Venlafaxine b. Nefazodone c. Mirtazapine d. Fluoxetine e. Bupropion

E. Buproprion While many of the newer antidepressants are well-tolerated, physicians should be familiar with the adverse effects and contraindications for their use. Nefazodone should not be used in patients with liver disease. Hypertension is a relative contraindication to venlafaxine. Patients experiencing hypersomnia and motor retardation should avoid nefazodone and mirtazapine. Patients who report agitation and insomnia should avoid bupropion and venlafaxine. Mirtazapine and tricyclic antidepressants are less preferred for patients with obesity. Bupropion is contraindicated for patients with seizure disorder

You have diagnosed a 30-year-old woman with depression. She is concerned that medical treatment may cause sexual dysfunction. In order to avoid sexual side-effects, which antidepressant would be the best choice? a. Amitriptyline b. Paroxetine c. Citalopram d. Sertraline e. Bupropion

E. Buproprion decreases orgasm threshold and causes less dysfx

You are caring for a 68-year-old smoker who complains of increasing shortness of breath with exertion and at rest. You observe that he is somewhat "barrel-chested," he breathes with pursed lips, and leans forward resting on his elbows when sitting in your office. On examination, he has decreased breath sounds and distant heart sounds. You are concerned about COPD and order office spirometry. Which of the following measurements is most sensitive to diagnose COPD? a. Total lung capacity (TLC) b. Forced vital capacity (FVC) c. Forced expiratory volume in 1 second (FEV 1) d. Forced expiratory flow rate over the interval from 25% to 75% of the total FVC (FEF25%-75%) e. FEV1:FVC ratio

E. FEV1:FVC ratio

You are caring for a married, monogamous 40-year-old man who complains of severe pain in his left testicle. You diagnose epididymitis. Which of the following is the most likely cause? a. Neisseria gonorrhoeae b. Chlamydia trachomatis c. Ureaplasma d. Mycoplasma e. Enterobacter

E. enterobacter >35 = enterobacter <35= chlamydia, gonorrhea

You are seeing a 20-year-old college student who reports that her left eye became pink over the last 24 hours. She is otherwise healthy and takes no medications except oral contraceptives. She reports redness, irritation, tearing, discharge, and itching. Which of her symptoms are more specific for an allergic etiology for her condition? a. Single eye involvement b. Irritation c. Tearing d. Discharge e. Itching

E. itching

You are thinking about starting a type 2 diabetic on insulin therapy to improve her glucose control. You would like to provide her with steady insulin action without much of a peak time. Which of the following insulin preparations provides the most stable insulin coverage without a peak time of maximum activity? a. Aspart (Novolog) b. Lispro (Humalog) c. Regular d. Neutral protamine hagedorn (NPH) e. Glargine (Lantus)

E. Glargine It is important to thoroughly understand the action of the different types of insulin preparations in order to make therapeutic decisions about diabetic patients and their control. Lispro's and Aspart's activity peaks early, between 30 and 60 minutes after injection. Regular insulin peaks between 2 and 3 hours after injection. Neutral protamine hagedorn (NPH) peaks 4 to 8 hours after injection. Glargine (Lantus) and Detemir (Levemir) do not have a predictable peak, and last for around 24 hours

You are caring for a 7-year-old boy whose parents and teachers have evaluated him for ADHD. Tests indicate that he is bored easily, daydreams frequently, and is inattentive. He has problems with alertness, impulsivity, and hyperactivity. Of those features, which is the most classic, distinguishing feature of ADHD? a. Easy boredom b. Daydreams c. Inattention d. Impulsivity e. Hyperactivity

E. Hyperactivity While all the symptoms listed are part of ADHD, hyperactivity across multiple settings is the classic, distinguishing feature of ADHD. Parents and teachers describe these children as being in constant motion. Hyperactivity is the most problematic feature for children with ADHD because it tends to be most disruptive and socially unacceptabl

You have diagnosed a 42-year-old patient with hypertension. He is 5 ft 9 in tall, weighs 230 lb, and admits to poor eating habits, drinking 4 alcoholic beverages daily, and no regular exercise. Which of the following lifestyle modifications, if instituted, will result in the largest systolic blood pressure reduction? a. Moderate alcohol consumption to no more than 2 drinks daily. b. Engage in physical activity for 30 minutes per day, most days of the week. c. Reduce dietary sodium intake to no more than 100 mEq/L per day. d. Adopt a DASH eating plan (a diet rich in fruits, vegetables, and low-fat dairy products with a reduced saturated and total fat content). e. Lose 10 lb.

E. Lose 10lbs Lifestyle modifications can help to manage hypertension. Weight reduction is most beneficial, and systolic blood pressure can fall from up to 20 mm Hg for each 10 lb of weight lost. A DASH diet can lower blood pressure between 8 and 14 mm Hg. Dietary sodium reduction, increased exercise and moderation of alcohol can be expected to lower systolic blood pressure less than 10 mm Hg

You are caring for a 31-year-old woman who complains of joint pain. She notes that her hands seem to be stiff in the morning, and that she seems to improve with time, movement, and heat. She reports more fatigue than usual as well. On examination, her wrists are swollen bilaterally, as are several of her metacarpal-phalangeal joints on each hand. You perform an appropriate workup and determine that her diagnosis is rheumatoid arthritis. Which of the following is the most appropriate next step? a. Control symptoms with nonsteroidal anti-inflammatory drugs (NSAIDs). b. Control symptoms with opiates. c. Use steroid treatment for flares, and NSAIDs for daily use. d. Use steroid injections to keep flares under control. e. Refer to rheumatology.

E. Refer to rheum for DMARD

A 62-year-old woman comes to your office complaining of dyspnea. She has a history of COPD, hypertension, and diabetes. She also smokes and drinks heavily. Her evaluation reveals that she is in heart failure. Which of the following interventions will lead to functional improvement in this patient? a. Optimizing the treatment of her COPD b. Optimizing the treatment of her hypertension c. Optimizing her glycemic control d. Discontinuing cigarette smoking e. Discontinuing alcohol use

E. Stop drinking ETOH Many noncardiac comorbid conditions may affect the proper diagnosis and clinical course of heart failure. All of the interventions in this question should be done, but only discontinuing alcohol use has actually been shown to improve function significantly. Optimally treating COPD is important, as exacerbations from heart failure are often difficult to distinguish from COPD exacerbations. Optimally treating diabetes and hypertension will minimize the negative effects of these conditions on the heart, but will not improve damage already done. Cigarette smoking should be discontinued, but generally does not lead to functional improvement. Those with alcoholic cardiomyopathy actually see improvement of the left ventricular function with abstinence.

You are medically treating an 85-year-old woman with stable angina, and choose to use nitrates. Which of the following is the most important consideration when using this medication? a. Headache b. Fatigue c. Interactions with β-blockers d. Interactions with calcium channel blockers e. Development of tolerance

E. Tolerance Development Tolerance is the most significant issue to consider when using nitrates for stable angina. Tolerance develops rapidly when long-acting nitrates are given. When using a patch, it is important to have intervals of 10 to 12 hours without the patch to retain the antianginal effect. Headache and fatigue may be important side effects, but are more of a nuisance than an important consideration. The medications can be used with β-blockers and calcium channel blockers

You are following a 54-year-old woman with diabetes. She has been very "brittle" and difficult to control. You are monitoring her urine microalbumin level and want to be alert to other changes that would suggest chronic renal insufficiency. If the patient were to develop chronic renal failure, which laboratory abnormality would you most likely see first? a. Hyperkalemia b. Hyponatremia c. Hyperphosphatemia d. Fall in plasma bicarbonate level e. Anemia

E. anemia (appears when GFR <60mL/minute) hyoerkalemia, hyponatremia, hyperphosphatemia and metabolic acidosis come later

You are caring for a 48-year-old Caucasian woman with a history of anorexia nervosa in her late twenties. She was an elite track and field athlete in her late teens and early twenties and was considered for the US Olympic team in her prime. Which of the following options is best for primary osteoporosis screening in this woman? a. History b. Physical examination c. Serum calcium d. Serum human osteocalcin levels e. Bone density imaging

E. bone density imaging

You are caring for a 35-year-old man who is complaining of fatigue and an inability to gain weight. Laboratory evaluation reveals a TSH of 6.0 mIU/L (H) but a normal free T 4. Which of the following is the best next step? a. Test for antithyroid peroxidase. b. Test for thyroid autoantibodies. c. Treat with levothyroxine. d. Treat with levothyroxine and T 3. e. Monitor at yearly intervals.

E. monitor annually Subclinical hypothyroidism is distinguished by an elevated TSH and a normal free T 4. This condition occurs in 4% to 8% of the general population. It will progress to clinical hypothyroidism at a rate of 2% to 5% per year. Risk for progression includes the presence of thyroid autoantibodies, old age, a female gender, and a TSH level greater than 10 mIU/L. Patients who do not progress are considered euthyroid with a reset thyrostat, and they should be monitored clinically and biochemically on an annual basis. Antithyroid peroxidase levels would help to diagnose autoimmune thyroiditis, but would not be helpful in this case. Starting thyroxine would be appropriate if the TSH was greater than 10 mIU/L, but not if the levels were only slightly elevated.

You are caring for a 22-year-old with moderate persistent asthma who has been well-controlled for several months. He developed an upper respiratory infection and his control worsened. He has not had a fever, but is coughing up sputum. In addition to stepping up his therapy, which of the following is true? a. You should begin a course of amoxicillin. b. You should begin a course of amoxicillin/clavulanate. c. You should begin a course of azithromycin. d. You should begin a course of ciprofloxacin. e. No antibiotics are necessary.

E. no antibiotics are necessary no fever or purulent sputum production

You are caring for a 42-year-old woman with a 10 days of congestion, purulent nasal discharge, and tooth pain symptoms. Based on clinical grounds, you diagnose her with sinusitis. According to studies, which of the following statements regarding treatment of this condition is true? a. Treatment with amoxicillin is superior to placebo. b. Treatment with erythromycin is superior to placebo. c. Treatment with ciprofloxacin is superior to placebo. d. Treatment with trimethoprim-sulfamethoxazole is superior to placebo. e. No significant difference has been demonstrated between any antibiotic and placebo

E. no significant difference has been demonstrated between any antibiotic and placebo

A 48-year-old man with type 2 diabetes returns for a follow-up appointment. He currently takes metformin but is not as well-controlled as he'd like to be. He'd like to know more about sitagliptin (Januvia). Which of the following best explains its mechanism of action? a. Inhibits glucagon release b. Increases the sensitivity of the body to insulin c. Inhibits hepatic gluconeogenesis d. Enhances gastric emptying e. Prolongs the action of endogenously released GLP-1

E. prolongs action of endogenously released GLP-1 Glucagon-like peptide 1 (GLP-1) is a gut-derived incretin hormone that stimulates insulin and suppresses glucagon secretion, delays gastric emptying, and reduces appetite and food intake. Sitagliptin is a dipeptidyl peptidase-4 (DPP-4) inhibitor which prolongs the activity of endogenously released GLP-1. Orally administered DPP-4 inhibitors, such as sitagliptin reduce HbA 1C by 0.5% to 1.0%, with few adverse events and no weight gain

A 70-year-old man with diabetes and long-term osteoarthritis in his knees is presenting for follow-up. He reports that his pain has become much more severe, and says he is having difficulty with ambulation and is becoming fairly inactive. In the past, he tried ibuprofen and naproxen, but those offered limited improvement and he developed secondary ulcers. He says that taking acetaminophen is like "taking a sugar pill"—it offers no help. He had some relief from steroid injections 3 months ago, and again 1 month ago, but they were short-lived. A recent x-ray is shown below . Which of the following is the next most appropriate step in the treatment of his condition? a. Use oral steroids. b. Try another steroid injection. c. Inject the knee joint with ketorolac (Toradol). d. Inject hyaluronic acid into his knee joints. e. Refer for knee replacements.

E. refer for knee rplacement

You have diagnosed a 66-year-old female patient of yours with chronic obstructive pulmonary disease (COPD). Which of the following therapies has been shown to improve the natural history of COPD? a. Smoking cessation b. Bronchodilators c. Inhaled steroids d. Antibiotics e. Supplemental oxygen

E. supplemental oxygen

You are seeing a 74-year-old man who is complaining that he is leaking urine. You have ruled out secondary causes and choose to measure his "postvoid" residual. It is 250 mL. Which of the following is true? a. Postvoid residual measurement has no place in the workup of incontinence. b. This amount is below what is expected, and leads one to suspect urge incontinence. c. This amount is about average, and is not helpful in determining this patient's type of incontinence. d. This amount is more than average, but is not helpful in determining this patient's type of incontinence. e. This amount is more than average, and would lead one to suspect overflow incontinence.

E. this amount is more than average and would lead one to suspect overflow incontinence

You check serologies on a patient exposed to hepatitis B. His serologies are shown below: • HBsAg: Negative • HBeAg: Negative • IgM anti-HBc: Negative • IgG anti-HBc: Negative • Anti-HBs: Positive • Anti-HBe: Negative Which of the following terms best describes his disease status? a. Acute infection, early phase b. Acute infection, window phase c. Acute infection, recovery phase d. Previous exposure with immunity e. Vaccination

E. vaccination

You are talking with a 33-year-old woman who is complaining of headaches. She has had these headaches for 5 months, and they are increasing in frequency. She reports that the headaches may last anywhere from an hour to several days. They are now occurring about 5 to 10 times a month, without relationship to her menstrual cycle. She describes the headache as bilateral, and the pain is described as a pressure around her forehead. She denies nausea, is not sensitive to sound, but is sensitive to light during an attack. On examination, she has no obvious neurologic deficit. Which of the following is the best approach to take at this point? a. Prescribe a triptan for abortive therapy. b. Prescribe NSAIDs and follow up if no improvement. c. Order blood work to rule out secondary cause. d. Order a CT of the brain. e. Order an MRI of the brain.

NSAIDS

A 26-year-old man complains of heartburn. He also complains of regurgitation, belching, and occasional dry cough. His symptoms are worse when he is lying down. He denies melena, weight loss, or dysphagia. What is the appropriate next step, if you suspect GERD in this patient?

Treat with H 2-receptor antagonists, a proton pump inhibitor, or a prokinetic agent and evaluate the response.

You suspect acute cystitis in an otherwise healthy woman. Which of the following features decrease the likelihood of a urinary tract infection (UTI)?

d. Absence of dysuria

You are caring for a 28-year-old man with a rash. The rash has been present for 3 to 4 years and has remained fairly stable. A picture of his rash is shown below. He also has fingernail changes also shown below. Which of the following is the best therapeutic choice for his rash? a. Topical fluorinated glucocorticoids b. Topical pimecrolimus (Elidel) c. Oral Penicillin d. Oral retinoids (Accutane) e. Oral Methotrexate

a. topical steroids d/t psoriasis

You are caring for a patient who complains of transient insomnia. His problems are associated with stressful times at work, and generally inhibits his activities approximately three to four times per month. When he is impacted, he generally can fall asleep easily, but has difficulty maintaining sleep. He wakes up at around 1:30 AM and finds it difficult to get back to sleep. He has eliminated caffeine and has maintained effective sleep hygiene. Which of the following medications work best to maintain sleep? a. Zolpidem (Ambien) b. Eszopicione (Lunesta) c. Zaleplon (Sonata) d. Diphenhydramine (Benadryl) e. Melatonin

Zaleplon (Sonata)

You are caring for a 51-year-old man complaining of neck pain for several weeks. He denies injury or illness. The pain is aggravated by movement, worse after activities, and there is a dull ache in the interscapular region. His examination reveals a limited range of motion, no tenderness to palpation, no radiation, and no neurologic signs. Which of the following is the most likely diagnosis? a. Osteoarthritis b. Chronic mechanical neck pain c. Cervical nerve root irritation d. Whiplash e. Cervical dystonia

a. Osteoarthritis

You are counseling one of your patients who is planning a trip overseas. He is concerned about becoming ill while traveling. Which of the following is the most common illness reported by international travelers?

a. Diarrhea

A 21-year-old generally healthy college student is seeing you in your office after having "passed out" playing basketball. This has never happened before. He has no significant past medical history and takes no medications. On examination, you note a harsh crescendo-decrescendo systolic murmur, heard best at the apex and radiating to the axilla. Which of the following tests is most likely to reveal the etiology of his syncopal episode? a. Echocardiogram b. Holter monitoring c. ECG d. Stress testing e. Tilt table testing

a. Echo pts with murmur and syncope need Echocardiogram to rule out valvular disease and ID's HCOM ECG- arrhythmias Stress test- ischemia tTlt table test - unexplained recurrent syncope where cardiac cases are ruled out

An 11-year-old patient has a history of recurrent pharyngitis with repeatedly positive streptococcal rapid antigen detection tests. You test him when he is asymptomatic and find that the test continues to be positive. Assuming that the patient has no allergies, what would be the best treatment for him? a. No antibiotics are required. b. Penicillin (PenVK). c. Amoxicillin (Amoxil), using high dosages. d. Azithromycin (Zithromax). e. Clindamycin (Cleocin)

a. No antibiotics are required we used to treat them but not anymore as there are no serious complications that arise from group A beta hemolytic strep

You are evaluating a 6-year-old boy. His mother has brought him in because he wets the bed. He has never been dry at night and his parents are starting to get concerned. You obtain a thorough voiding history, and find the child to be completely normal on physical examination. He is otherwise developmentally normal. His urinalysis is normal and his postvoid residual is also normal. What should be the next step in the workup of this patient?

a. Observation

You are discussing enuresis therapy with the mother of an 8-year-old girl that you care for. The mother is not interested in pharmacologic therapies, but would like to discuss using a moisture-sensitive alarm. Which of the following is true regarding the use of these alarms for nocturnal enuresis?

a. The goal of this alarm is to wake the child just after the initiation of urination

You are evaluating a 40-year-old male patient in the office who is complaining of chest pain. His father had a myocardial infarction at age 42, and the patient is quite concerned. Which characteristic, if included in the history, decreases the likelihood that his chest pain is cardiac in origin?

a. The pain is worse with inspiration

You are caring for a healthy woman whose cousin was just diagnosed with unilateral breast cancer at age 33. Your patient has no other relatives with known histories of breast or ovarian cancer. Which of the following is true regarding the current recommendations for genetic screening for breast cancer mutations?

a. The patient should not be offered testing . unless: • Two first-degree relatives with breast cancer, one of whom was diagnosed when younger than the age of 50. • A combination of three or more first- or second-degree relatives with breast cancer regardless of the age at diagnosis. • A combination of breast and ovarian cancer among first- and seconddegree relatives. • A first-degree relative with bilateral breast cancer. • A combination of two or more first- or second-degree relatives with ovarian cancer, regardless of age at diagnosis. • A first- or second-degree relative with both breast and ovarian cancer at any age. • A male relative with breast cancer.

A 28-year-old woman comes to you for wart removal. On examination, she has a single wart on the lateral aspect of her index finger near the distal interphalangeal joint. She has no other medical conditions, but is trying to become pregnant. Her last period was 4 weeks ago. Which of the following treatment options would be best in this situation? a. Topical treatment with liquid nitrogen b. Topical treatment with podophyllum resin c. Topical treatment with local interferon inducer like Imiquimod d. CO2 laser treatment e. Injection with bleomycin

a. topical liquid nitrogen

On screening physical examination of a 36-year-old woman, you find a single left breast mass. It is approximately 1 cm in size, firm, and smooth. You perform a mammogram which is characterized as BI-RADS 2. What does this most likely indicate?

a. The physician should continue routine screening at the usual intervals.

You are evaluating a 69-year-old woman with a history of asthma and ischemic cardiomyopathy who is complaining of dyspnea. You are not sure if her symptoms are related to asthma or congestive heart failure (CHF), and you order a b-type natriuretic peptide to help in her evaluation. The level is found to be 76 pg/mL (normal is 0-100 pg/mL). Which of the following is most correct regarding the interpretation of this laboratory value?

a. The probability that her symptoms are related to CHF is near zero.

You are caring for a 70-year-old hospitalized male who is currently 1 day out from a carotid endarterectomy. You are called to the floor at 3 AM because the patient removed his peripheral IV and is demanding to go home. Reviewing his chart, you see he has a history of hypertension and hyperlipidemia, both of which are well controlled with medication. He is working part time as an auto mechanic and lives at home with his wife. On evaluation, he is agitated but responds to questions, is oriented to person only, and denies chest pain, palpitations, shortness of breath, dizziness, or other problems. Which of the following characteristics points to delirium instead of dementia in this case? a. The acute onset of his symptoms b. The fact that he is disoriented to time and place c. His history of hypertension d. The fact that he is responsive to questions e. The fact that this happened in the early morning hours

a. acute onset of symptoms

You are evaluating a 14-year-old female patient whose mother brought her in for evaluation. Despite the fact that all of her friends have started menstruating, the daughter has not. On examination, she has no breast development, no axillary or pubic hair, and her pelvic examination reveals normal-appearing anatomy. She has not lost weight recently and is not excessively thin. Which of the following is the most likely cause of her primary amenorrhea? a. Gonadal dysgenesis b. Hypothalamic failure c. Pituitary failure d. Polycystic ovarian syndrome e. Constitutional delay of puberty

a. gonadal dysgenesis

A 30-year-old male cyclist comes to your office complaining of knee pain. He describes lateral knee pain when he goes for a long bike ride that does not improve with activity. On examination, he has tenderness over the lateral aspect of the knee just above the joint line. Which of the following is the mostly likely diagnosis? a. Iliotibial band syndrome b. Patellofemoral pain syndrome c. Medial collateral ligament sprain d. Anterior cruciate ligament sprain e. Medial meniscal tear

a. iliotibial band syndrome = MCC of lateral knee pain in an athlete MC seen in athletes who participate in repetitive knee flexion acitivities like distance runners and cyclist

You are caring for a teenager who complains of acne. It is most apparent on his forehead and his cheeks. It is causing a great deal of stress in his life and impacting his self confidence. Of the following, which is most likely a contributing factor to his condition? a. Leaning his face on his hands while sitting at his desk at school b. Eating fast food c. Not washing his hair often enough d. Eating chocolate e. Not eating enough vegetables

a. leaning face on his hands at school

Your patient describes a recent vaginal discharge. She reports more discharge than usual and an unusual odor after intercourse with her husband. A KOH preparation of the discharge produces a fishy odor, and a saline preparation is shown below. Which of the following is the treatment of choice for her condition? a. Metronidazole b. Doxycycline c. Clotrimazole d. Imiquimod e. Acyclovir

a. metronidazole imiquimod treats HPV infx's clotrimazole treats fungal infx's

You are evaluating a 16-year-old girl who has never menstruated. She has normal secondary sexual characteristics and her laboratory evaluation is negative. She has no withdrawal bleeding after a progestin challenge and you choose to perform an estrogen-progestin challenge. She has no withdrawal bleeding after that challenge as well. Which of the following is the most likely reason for her amenorrhea? a. Outflow tract obstruction b. Hypergonadotropic amenorrhea c. Hypogonadotropic amenorrhea d. Polycystic ovarian syndrome e. Pituitary adenoma

a. outflow tract obstruction

After returning from a ski trip in the mountains, your 35-year-old patient developed a rash. He has multiple erythematous pustules over his legs, arms, and chest. They are not pruritic and do not seem to be spreading. He denies any new soaps, lotions, foods, or medications. He did spend time in a hot tub on the trip. A picture of his rash is shown below. Which of the following is the best treatment option for this patient? a. Reassurance and follow-up if no improvement b. Topical steroid medication c. Systemic steroid medication d. Topical antibiotics with activity against Streptococcus and Staphylococcus species e. Oral antibiotics with activity against Pseudomonas species

a. reassurance and f/u if no improvement hot tub folliculitis, casued by pseudomonas but is self limited

A patient comes to see you after a skiing accident 6 days ago. She reports twisting her left knee during a fall, feeling a "pop," and noting significant immediate swelling. She was able to bear weight immediately, but did not ski for the rest of the trip. Her pain is now improved, and she is ambulating, but she says the knee feels unstable. On examination, she has a tense effusion in her left knee and is unable to extend her knee fully. Which of the following is the most likely cause of her symptoms? a. Patellofemoral pain syndrome b. Anterior cruciate ligament (ACL) tear c. Posterior cruciate ligament (PCL) tear d. Meniscal injury e. Medial collateral ligament sprain

b ACL tear twisting injury pop soud bear weight immediately *patellofemoral - no acute pain p/s injury PCL injury = direct force to knee meniscal = locking, catching, giving way sprain = pain w/ no swelling over joint

You are seeing a 6-year-old boy with nausea and vomiting. His symptoms began acutely last evening, starting with malaise, headache, lowgrade fever, body aches, and diarrhea. On examination, he has dry mucous membranes, but no orthostatic symptoms. He has diffuse mild abdominal pain without rebound or involuntary guarding. Which of the following is the best treatment for his condition? a. Nothing by mouth until his symptoms improve b. Oral rehydration with clear liquids, advancing the diet as tolerated c. IV rehydration, advancing to oral as tolerated d. Antiemetics, given intravenously or intramuscularly e. Trimethoprim/sulfamethoxazole therapy

b oral rehydration with clear fluids advancing with diet as tolerated

You are seeing a 66-year-old man complaining of right-sided neck pain. He suffers from neck stiffness and complains that his right hand has become "numb." On examination, you confirm paresthesia of his fingers that continues up the back of his arm, and his pain worsens when he turns his head to the right. Which of the following studies, if any, should be your next step in the workup? a. Cervical spine radiographs. b. CT scan of his neck. c. MRI of his neck. d. Electromyography (EMG). e. No testing is necessary.

b. CT Neck

You are seeing a 23-year-old man for shortness of breath. He has no history of asthma or wheezing and is otherwise healthy. His lung examination does reveal significant wheezing bilaterally. Which of the following tests is necessary? a. Observation and treatment b. Chest x-ray c. Peak flow testing d. Pulmonary function tests e. CBC

b. CXR, as this is the first episode/ work up for SOB Once that has been done, PFTs and CBC can be used as dx tools

A 30-year-old man returned from a vacation in Mexico 1 day ago. He spent the last 3 days of his trip with loose, more frequent bowel movements that are continuing without resolution. He has not had bloody stool or fever. His examination is normal, except for mildly diffuse lower abdominal pain. Which of the following is the best empiric treatment option for his condition?

b. Ciprofloxacin

A 42-year-old woman presents to your office complaining of the recent onset of abdominal pain. She describes pain that starts in the midepigastric region, radiating to the back. It is associated with nausea and vomiting. What is the most likely diagnosis?

b. Pancreatitis

You are evaluating a 35-year-old patient with known asthma. He comes to your office complaining of increased shortness of breath despite compliance with his usual asthma regimen. He reports cough, but denies fever or sputum production. His pulmonary examination reveals wheezing bilaterally without crackles or rhonchi. Which of the following is most useful in this setting? a. Chest x-ray b. Peak flow testing c. Pulmonary function testing d. CBC e. Nasopharyngeal washing

b. Peak flow testing, used to monitor asthmatic patients

You are discussing fatigue with one of your 53-year-old female patients. She reports that her symptoms have occurred for the past 6 months, and have been getting progressively worse. She reports increased stress and working longer hours at work, and she is drinking a glass of red wine each evening after work to relax. Which component of her history points to a physical cause of her fatigue?

b. Progressively worsening

You are counseling a parent whose 7-year-old son wets the bed at night. Which of the following interventions has proven to be the most effective cure for this condition?

b. Use of an alarm that wakes the child when he wets at night

You are seeing a 53-year-old man who was hospitalized for pancreatitis. His admission laboratory studies include a WBC count of 18,000/mm 3, glucose of 153 mg/dL, lactate dehydrogenase (LDH) of 254 IU/L, and AST of 165 U/L. According to Ranson criteria, which of these factors suggest a poor prognosis in this patient

b. WBC count

You are evaluating a 31-year-old man with the acute onset of nausea and vomiting. It is associated with significant epigastric pain that radiates to the back and occurs after eating any type of food. It is somewhat better if he does not eat at all. Which of the following tests is most likely to be abnormal in this case? a. CBC b. Amylase and lipase level assessment c. Hemoccult testing of the stool d. Abdominal x-rays e. Upper endoscopy

b. amylase/lipase

You are seeing a young girl whose mother brings her in for evaluation. She has had 3 days of low-grade fever and runny nose. Today, she awakened with an erythematous rash on her cheeks as shown below. Which of the following is the most likely cause of her symptoms? a. An enterovirus b. A parvovirus c. A parainfluenza virus d. A varicella virus e. Cytomegalovirus

b. parvovirus B19

You are seeing a 12-year-old girl who asked her mother to take her to the doctor for nausea and vomiting. She was diagnosed as having viral gastroenteritis in the emergency department more than 6 weeks ago, but since that time has had difficulty keeping food down. She states that whenever she eats, she gets nauseated and vomits within 10 to 30 minutes. She has been using antiemetics to control her symptoms, but they do not work consistently. She has always done well in school, and denies social stressors. Her medical history is unremarkable, but she was treated for depression last year. On examination, she is well-nourished, interactive, and in no distress with no signs of dehydration. Her weight is 147 lb (5 lb less than at her well examination 6 months ago) and her height is 5 ft. Which of the following is the most likely cause of her symptoms? a. Chronic gastroenteritis b. Psychogenic vomiting c. Anorexia nervosa d. Bulimia nervosa e. Central nervous system malignancy

b. psychogenic vomiting

You are seeing a 6-year-old patient whose mother brought him in for severe ear pain and fever. On examination, he is febrile with a temperature of 102.5°F, and his right tympanic membrane is shown below: Which of the following would be the best initial treatment? a. A weight-adjusted dose of Tylenol b. A weight-adjusted course of amoxicillin c. A weight-adjusted course of amoxicillin-clavulanate d. A weight-adjusted 3-day course of azithromycin e. A weight-adjusted 5-day course of azithromycin

c. A weight-adjusted course of amoxicillin-clavulanate

A 42-year-old woman is seeing you to follow up with a new complaint of "dizziness." She reports that symptoms first began several months ago. At that time, she reported a subjective hearing loss and a ringing in her left ear only. Symptoms were mild, and her physical examination was normal, so you elected to follow her. Since that time, her symptoms have progressed to include dizziness and some facial numbness. Which of the following is her most likely diagnosis?

c. Acoustic neuroma

You are caring for an 18-month-old infant, whose mother brings him in for "wheezing." She reports that he has had a runny nose and a slight cough for 2 days, along with a low-grade fever. On examination, he does not appear to be in respiratory distress, but his lung examination does reveal bilateral wheezing. Which of the following is the most likely diagnosis? a. Acute viral respiratory tract infection b. Pneumonia c. Bronchiolitis d. Aspiration e. Asthma

c. Acute Viral respiratory tract infection -occurs in winter season -passive smoke exposure

You are seeing a 24-year-old woman who presents to your office complaining of "wheezing." She reports acute shortness of breath that occurred while she was shopping, and her wheezing is associated with pleuritic pain. She is otherwise healthy, only taking oral contraceptives. On examination, she is tachypneic, but not in acute distress. Auscultation of her lungs is normal. After the appropriate diagnostic workup, what is the best treatment option for this patient? a. Reassurance and observation b. Antibiotic therapy c. Anticoagulation d. Bronchodilators e. Steroids

c. Anticoagulation as she has the symptoms of a suspected PE after appropriate diagnostic tools have been offered.

While performing an abdominal examination on a 42-year-old woman in your office, she suddenly stops inspiratory effort during deep palpation of the right upper quadrant. Of which of the following problems is this most suggestive?

c. Cholecystitis

You are seeing a 21-year-old man with a skin infection on his lip. It began with a burning at the site of the infection, then an eruption of vesicles. The rash is shown below. Which of the following is true of the treatment for this infection? a. Oral antiviral agents are less effective in treating primary infections than treating recurrences. b. Oral therapy begun within 2 days of onset is the best treatment for recurrent outbreaks. c. Chronic suppression with daily therapy is beneficial for oral herpes. d. Acyclovir resistance makes it a poor choice for therapy. e. Famciclovir is less effective for cutaneous infections than valacyclovir.

c. Chronic daily supprese. sion

A 56-year-old man is found to have asymptomatic diverticulosis on screening colonoscopy. He is concerned about his risk for GI bleeding from the diverticula. Which of the following statements is most accurate regarding his concern? .

c. Diverticular bleeding resolves spontaneously in the vast majority of cases

You are seeing a 56-year-old man who reports a recent syncopal episode. Which of the following tests is always indicated in the workup? a. CBC b. TSH assessment c. ECG d. Echocardiogram e. Tilt table testing

c. EKG, as it could point out a heart abnormality fairly quickly and easily

You are evaluating a 55-year-old man with hypertension and hyperlipidemia who complains of unilateral lower extremity edema. It has been present on and off for almost a year. He denies dyspnea or recent trauma, and has no evidence of inflammation. Which of the following would be the best treatment option for his condition?

c. Elastic stockings

A 34-year-old man reports a 1-day history of hematemesis. He feels well, but does describe occasional abdominal discomfort. He denies alcohol use. On examination, his abdomen is slightly tender without peritoneal signs. His stool is not bloody, but his fecal test for occult blood is positive. Which of the following is the most appropriate next step?

c. Endoscopy

You are caring for a 65-year-old man with new-onset jaundice. Laboratory evaluation reveals conjugated hyperbilirubinemia. Statistically speaking, which of the following is the most likely cause of his condition? a. Hemolytic anemia b. Viral hepatitis c. Extrahepatic obstruction d. Metastatic disease e. Heart failure

c. Extrahepatic obstruction

You are caring for a 20-year-old male patient with knee pain. He is a runner and reports bilateral anterior knee pain. He is unable to localize the pain to a specific region. The pain is worse with stairs and he has been unable to run because of the pain. When he sits for prolonged periods of time, the knees start to ache. Strengthening of which of the following muscles will lead to improvement of this problem? a. Hip abductors b. Hip adductors c. Quadriceps d. Hamstrings e. Internal rotators

c. Quadriceps 'due to patellofemoral pain syndrome MC dx for anterior knee pain, worse with exercise long periods,

You are seeing a 4-year-old male child 2 weeks after being diagnosed with left acute otitis media. He completed his therapy, and is afebrile, acting well, and apparently back to normal. On examination, he has a persistent effusion in the left ear. There is no erythema, purulence, or hearing loss. Which of the following is the most appropriate next step?

c. Regular use of a decongestant and reevaluation in 2 weeks

A 22-year-old woman is seeing her physician with complaints of breast pain. It is associated with her menstrual cycle and is described as a bilateral "heaviness" that radiates to the axillae and arms. Examination reveals groups of small breast nodules in the upper outer quadrants of each breast. They are freely mobile and slightly tender. Which of the following statements is most accurate?

c. The patient has bilateral fibrocystic changes, and reassurance is all that is necessary.

You are determining whether or not to use a rapid streptococcal antigen test to screen for streptococcal pharyngitis. You find that 2% of people with strep throat actually test negative using this test. Which of the following statements best describes this situation?

c. The test has a 2% false-negative rate.

A 28-year-old monogamous married woman comes to you for emergency contraception. She and her husband typically use condoms to prevent pregnancy, but when they had sex approximately 36 hours ago, the condom broke. She does not want to start a family at this time. Which of the following statements is true regarding the use of emergency contraception pills (ECPs)? a

c. There are no medical contraindications to the use of ECPs, other than allergy or hypersensitivity to the pill components.

A 16-year-old woman comes to your office complaining of unpredictable menstrual periods. She began her periods at age 14 and they have never been predictable. She denies sexual activity in her lifetime, has no systemic illness, uses no medications regularly, and her physical examination is normal. Which of the following is her most likely diagnosis? a. Pregnancy b. Ovulatory bleeding c. Anovulatory bleeding d. Uterine leiomyoma e. Endometrial polyposis

c. anovulatory bleeding

You are evaluating a 63-year-old man who complains of abdominal pain, distension, nausea, and vomiting. It began rather suddenly this morning, though he has had mild pain for several days. His past history is significant for a recent partial sigmoid resection for diverticulosis and an appendectomy at 23 years of age. On examination, he is afebrile, his mucous membranes are dry, but he has no orthostatic symptoms. His abdomen is distended and diffusely tender, and his bowel sounds are hyperactive. Which of the following is the most likely cause of his nausea and vomiting? a. Gastroenteritis b. Ileus c. Obstruction d. Diverticulosis e. Diverticulitis

c. obstruction

You are in the emergency room caring for a 47-year-old man who was brought in by his wife. She states that he had the acute onset of confusion. His past medical history is unremarkable, without evidence of drug or alcohol use. On examination, you find his blood pressure to be 210/130 mm Hg, his pulse to be 97 beats/min, and his respirations to be 20 breaths/ min. His temperature is 98.4°F. Strength, sensation, and gait are normal. He has no tremor. Which of the following would you expect to find on ophthalmologic examination? a. Pinpoint pupils b. Dilated pupils c. Papilledema d. Sixth cranial nerve palsy e. Anisocoria of 1 mm

c. papilledema

A 7-year-old boy comes to see you for a sore throat. He reports fevers, chills, myalgias, and pain on swallowing. On examination, you note anterior adenopathy, erythematous tonsils, and edema of his uvula. He has no drug allergies. Which of the following would be the best treatment for his condition? a. Symptomatic care b. Antiviral therapy c. Doxycycline (Vibramycin) d. Amoxicillin (Amoxil) e. Erythromycin (Emycin)

d. Amoxicillin PCN allergic = 1st gen ceph/macrolide

You are counseling a patient regarding contraception options. She is 36 years old, she smokes one pack of cigarettes daily, weighs 145 lb, and has no medical illnesses. She is sexually active, but is not in a monogamous relationship. Which of the following is her best contraception option?

d. An injectable form of long-acting progestin

391. You are seeing a 65-year-old woman with a history of diabetes and hypertension. She is overweight and does not exercise regularly. You are concerned that she may have renal failure, given her risk factors. Which of the following is the best test to detect the presence of renal insufficiency in this patient? a. Her blood urea nitrogen (BUN) level b. Her serum creatinine level c. Her BUN to creatinine ratio d. Her calculated or estimated glomerular filtration rate (GFR) e. Her urine microalbumin level

d. Her calculated or estimated glomerular filtration rate (GFR)

A 42-year-old woman is seeing you to evaluate nausea and vomiting. It happens about 60 minutes after eating a big meal and is associated with pain in the epigastric area. Which of the following tests is most likely to be abnormal in this case? a. Amylase and lipase level assessment b. Hemoccult testing of the stool c. Abdominal x-rays d. Ultrasound e. Upper endoscopy

d. ultrasound

A mother brings her son in to see you emergently. He is almost 2 years old, and began to have significant abdominal pain, vomiting, and bloody stool today. He is currently hemodynamically stable, in obvious pain, and on examination you notice a palpable firm tube-like mass in his left lower quadrant. Which of the following is the most likely diagnosis?

d. Intussusception

You are evaluating a 47-year-old woman complaining of bilateral lower extremity edema. She denies dyspnea, and on examination has no rales, JVD, or ascites. Her cardiac examination is normal. What should be the next step in the evaluation of her edema?

e. Urinalysis

You are evaluating a 45-year-old woman with significant jaundice. Her alkaline phosphatase is seven times normal, and her transaminases are twice normal. You perform an ultrasound of her right upper quadrant, and it is negative for obstruction and shows no bile duct dilation. You still suspect obstruction. Which of the following should be the next step in the workup? a. CT of the abdomen b. Endoscopic retrograde cholangiopancreatography (ERCP) c. Percutaneous transhepatic cholangiography (PTC) d. Magnetic resonance cholangiopancreatography (MRCP) e. Nuclear scintigraphy of the biliary tree (HIDA)

d. Magnetic resonance cholangiopancreatography (MRCP)

You perform a Spurling test on a 36-year-old woman complaining of neck pain. The test causes neck discomfort only. What is the most likely diagnosis for this patient? a. Herniated disk b. Spinal stenosis c. Osteoarthritis d. Mechanical neck pain e. Cervical dystonia

d. Mechanical neck pain

A 19-year-old male patient presented to your office with a 3-day history of fatigue, sore throat, and low-grade fevers. On examination, his temperature was 100.3°F, and you noted an exudative pharyngitis with cervical adenopathy. You sent a throat culture and started him on amoxicillin prophylactically. Two days later, he presents for follow-up with continued symptoms and a diffuse, symmetrical erythematous maculopapular rash. Which of the following is the most likely cause of his symptoms?

d. Mononucleosis

You are seeing a 21-year-old college student who complains of congestion, headache, sinus pressure, and tooth pain for more than 2 weeks. She is otherwise healthy, but feels like she's "having trouble shaking this cold." She has used over-the-counter decongestants with limited relief. A CT scan of her sinuses demonstrates acute sinusitis. Which of the following is the most common organism causing her symptoms? a. Moraxella catarrhalis b. Staphylococcus aureus c. Group A β-hemolytic streptococcal species d. Streptococcus pneumoniae e. A polymicrobial mixture of many organisms

d. Strep pneumonia MC in bacterial sinusitis h. flu, m cat, and group A B hemolytic strep are also seen

You are evaluating a 23-year-old swimmer who is complaining of episodes of symptomatic rapid heart beating. Twice during swim practice, he develops a sensation that his heart is racing. When he measures his heart rate, he finds it to be between 140 and 160 beats/min. The first episode lasted approximately 4 minutes and the second lasted more than 10 minutes. He denies light-headedness or other symptoms during the events. Limited laboratory evaluation and ECG are normal. Which of the following is the next step in the evaluation? a. Reassure and continue observation. b. Ambulatory ECG monitoring. c. Consultation with an electrophysiologist. d. Stress testing. e. Echocardiography.

d. Stress Testing

You are caring for a 49-year-old type 2 diabetic woman who presents to you after passing out. The event occurred 1 day ago, while she was walking up steps to her seat at a movie theatre. She reports that she felt breathless, became hot and sweaty, and the next thing she remembers, she was waking up on the floor. Her diabetes has been fairly well-controlled with metformin, and her last glycosolated hemoglobin 1 month ago was 7.9%. Her examination is benign, as is her ECG. Which of the following tests would be most likely to reveal the cause of her syncope? a. Serum glucose assessment b. Hemoglobin A 1C c. Echocardiogram d. Stress testing e. Twenty-four hour Holter monitoring

d. Stress Testing, becamse breathless, hot/sweaty *Holter monitor if evidence of palpitations/EKG abnormalities

You are seeing a 44-year-old woman with hypertension controlled with lisinopril, who presents with severe nausea and vomiting. She reports having months of occasional right upper quadrant pain, usually after eating out with her husband, that resolves within a couple of hours. Over the last 24 hours, her symptoms have been severe, and she is unable to eat or drink without vomiting. Her pain is significant, radiates to her back, and is better when she leans forward. On laboratory evaluation, her amylase is elevated, and her ALT is elevated. Which of the following would be the best approach to avoid recurrent problems in her case? a. Discontinue lisinopril. b. Avoid calcium in the diet. c. Work with the patient to remain sober. d. Remove the patient's gallbladder. e. Use medication to lower the patient's triglyceride level.

d. remove patients gallbaldder

You are evaluating a homeless person in the emergency department who is displaying hyperalert confusion. Withdrawal from which of the following substances is most likely to cause this state? a. Levothyroxine b. Fluoxetine c. Oxycodone/acetaminophen d. Alcohol e. Amphetamine

d. alcoholism

A healthy 60-year-old woman is seeing you to evaluate vaginal bleeding. She has not had a menstrual period for approximately 7 years, but 3 months ago noted occasional pink spotting. Since then, it has increased in amount and has become almost continuous. She is currently sexually active with her husband. On examination, she appears well, her pelvic examination is normal, and screens for sexually transmitted infections are negative. Which of the following should be your next step? a. Pelvic ultrasound to evaluate for fibroids. b. Pelvic CT scan to evaluate for pelvic tumor. c. Laparoscopy to evaluate for endometriosis. d. Endometrial biopsy. e. Begin hormone-replacement therapy to regulate bleeding

d. endometrial biopsy

You are seeing a 16-year-old high school football player to discuss a recent injury. Last night, during football practice, he dislocated his shoulder. His trainer took him to an urgent care where an x-ray confirmed the diagnosis. They relocated his shoulder and put him in a sling. This is the first time he has dislocated his shoulder. Which of the following represents the best appropriate next steps? a. If his range of motion is normal, allow him to return to play without restriction. b. Immobilization until an MRI can be obtained. c. Referral for physical therapy as soon as possible. d. Immobilization for 7 to 10 days, then begin physical therapy. e. Immediate surgical referral.

d. immobilization for 7-10 days then begin PT

You are treating a 26-year-old woman for nausea. Which of the following antiemetics is most likely to cause extrapyramidal reactions in the patient? a. Trimethobenzamide (Tigan) b. Prochlorperazine (Compazine) c. Promethazine (Phenergan) d. Metoclopramide (Reglan) e. Ondansetron (Zofran)

d. metoclopramide

You are evaluating a 71-year-old male patient with the complaint of shortness of breath. It mainly occurs with exertion. He also complains of fatigue, and needs to sleep propped up on two pillows. On physical examination, you note a large apical impulse and jugular venous distension (JVD). He has fine crackles in the bases of both lungs with decreased breath sounds. Which of the following would be the most appropriate treatment?

e. Diuretics

You are evaluating a 26-year-old man with rectal pain. The pain was initially associated with bright red blood on the toilet paper after a bowel movement. Over the last day, his pain has worsened. On examination, he has an exquisitely tender purple nodule distal to the dentate line. Which of the following is the best treatment for his condition?

e. Excision

You are caring for a 21-year-old woman complaining of pelvic pain. She reports a gradual onset of bilateral pain associated with fever, vomiting, vaginal discharge, and mild dysuria. Her pelvic examination demonstrates uterine, adnexal, and cervical motion tenderness. Which of the following is the best treatment option? a. Ceftriaxone 250 mg IM in a single dose b. Oral doxycycline 100 mg twice a day for 14 days c. Ceftriaxone 250 mg IM in a single dose plus oral doxycycline 100 mg twice a day for 14 days d. Ceftriaxone 250 mg IM in a single dose plus oral doxycycline 100 mg twice a day for 14 days plus oral metronidazole 500 mg twice a day for 14 days e. Inpatient admission for parenteral antibiotics

e. The patients has PID, so inpatient admission for parenteral antibiotics is in order outpatient regimen = ceftriaxone IM + doxycycline for 14 days

You are evaluating a 28-year-old woman who has had several episodes of passing out. In general, the events are unpredictable, and are not preceded by any prodrome. Her examination has been consistently normal. Initial workup, including a pregnancy test, hematocrit, serum glucose, orthostatic blood pressures, and ECG were normal. She underwent 24-hour Holter monitoring and long-term ambulatory loop ECG evaluation, both of which were negative. Which of the following is the most appropriate next test? a. Psychiatric evaluation b. Carotid Doppler c. MRI of the brain d. Stress testing e. Tilt table testing

e. Tilt table testing abnl result suggests vasovagal syncope

You are caring for a 58-year-old postmenopausal woman who is not on estrogen replacement therapy. You perform her annual Pap test, and the results are reported as "atypical cells of undetermined significance." HPV testing was not performed. Which of the following is the most appropriate next step?

e. Treat with a 4-week course of vaginal estrogen cream and repeat Pap testing.

An 80-year-old man presents with mild, crampy, bilateral lower quadrant pain, decreased appetite, and low-grade fever for about 48 hours. Which of the following is the most likely diagnosis?

Appendicitis

In the prenatal workup for a 24-year-old patient, you discover she is not immune to rubella. When is the best time to vaccinate her against rubella?

At least 4 weeks postpartum

A 32-year-old woman comes to your office for a complete physical examination. When discussing her vaccinations, you discover that she received her primary tetanus series as a child, and her last tetanus booster was 11 years ago. Which of the following is true?

The patient should receive a diphtheria-tetanus-pertussis (DTP) immunization.

You are counseling a mother about her child's immunization schedule. She asks specifically if her child would benefit from the Haemophilus influenzae type b (Hib) vaccine. Which of the following statements is true about this vaccine?

The vaccine is between 95% and 100% effective in preventing invasive Hib disease.

A 33-year-old healthy nonsmoking man presents to you for evaluation of his chronic cough. He says the cough has been present for about 8 weeks. Initially, he went to an urgent care where he received antitussives and a bronchodilator. Those did not help, and he returned 1 week later and was given a course of azithromycin. His cough has continued to persist. His symptoms are worse when he lies down for sleep, and are associated with a sore throat. He has also noticed that when he drinks caffeine or alcohol, the cough seems to worsen. Which of the following is the most likely diagnosis?

a. GERD

You are seeing a patient who reports the abrupt onset of deep epigastric pain with radiation to the back associated with nausea, vomiting, sweating, and weakness. On examination, his abdomen is distended and tender in the epigastric area. Which of the following is the most common cause of this condition?

a. Gallstones

A laboratory analysis of one of your patients reveals a microcytic anemia. The red cell distribution width (RDW) is elevated. Which of the following is the most likely diagnosis?

a. Iron deficiency

You are caring for a 41-year-old man who is complaining of difficulty with defecation. He reports a recent onset of having fewer bowel movements per week that include difficult passage of stool. He denies hematochezia, melena, or weight loss. He has no family history of colon cancer or inflammatory bowel disease. His physical examination and fecal occult blood test in the office are negative. What initial laboratory testing is indicated in the workup of his condition?

a. No blood tests are needed.

A 15-year-old adolescent boy comes to your office complaining of bilateral breast enlargement. He is otherwise healthy and on no medications. On examination, there is mildly tender palpable breast tissue bilaterally. The rest of his physical examination, including his testicular examination, is normal. Which of the following is true?

a. No further workup is necessary.

You are completing a preoperative evaluation on a 66-year-old man who will be undergoing prostate surgery. He has hypertension but no other diagnosed medical problems. He has smoked half pack of cigarettes daily since he was 21 years old. He denies dyspnea or cough. Which of the following is true regarding his preoperative evaluation

a. No pulmonary evaluation is necessary.

You are performing a physical examination on a student traveling to Mexico with her college Spanish class. She is concerned about traveler's diarrhea, and asks about antibiotic prophylaxis. Which of the following best represents the current guideline from the Centers for Disease Control and Prevention (CDC) for prevention of traveler's diarrhea?

a. The CDC does not have an antibiotic guideline regarding antibiotic prophylaxis for traveler's diarrhea.

A 52-year-old man comes to your office for a complete physical examination. He is interested in prostate cancer screening. Which of the following best represents current guidelines for prostate cancer screening?

a. There is insufficient evidence to recommend for or against prostate cancer screening.

You are seeing a 14-year-old high school wrestler for a skin condition. About a week ago, he noted a patch of erythematous skin on his right thigh. The patch has enlarged since he first noted it, and the central part of the lesion seems to be clearing. He reports that it is mildly pruritic. You scrape the lesion and evaluate the shavings under the microscope using potassium hydroxide. The slide is shown below. Which of the following is the most likely diagnosis?

a. Tinea corporis

You are discussing vaccinations for a patient who is traveling internationally. Because of a significant fear of needles, he is unwilling to obtain any vaccination unless it is required. Which of the following is the only mandatory travel vaccine?

a. Yellow fever

You are reading a medical journal and come across an article about diabetes. The study followed 10,000 patients over 3 years. At the start of the study, 2000 people had diabetes. At the end of the study, 1000 additional people developed diabetes. What was the incidence of diabetes during the study?

b. 12.5%

You note that in your practice, a large number of women with a family history of breast cancer in a first-degree relative develop breast cancer themselves. You evaluate a number of charts, and find that 5% of the women in your practice who have breast cancer have a family history, but only 2% of women without breast cancer have a family history. Given this information, what is the sensitivity of using family history as a predictor of breast cancer in your patient population?

b. 5%

You are discussing cancer screening with a patient. Her father was diagnosed with colorectal cancer at age 62. When should you recommend she begins colorectal cancer screening?

b. 50 years (10 years before the relative was dx'd)

You are considering using a new influenza screening test. You find a study that evaluated 1000 patients with this new test. Of these 1000 patients, 400 had the disease. Three hundred of those had positive tests, and 100 of those had a negative test. Of the 600 that did not have the disease, 200 had positive tests, and 400 had negative tests. What is the positive predictive value of this test?

b. 60%

A 3-year-old African-American boy is brought in by his parents with inconsolable crying. He reports extreme pain in his hands and upper extremities. Laboratory evaluation reveals a hemoglobin of 8.2 mg/dL. His peripheral blood smear is shown as follows: Which of the following measures would be most likely to reduce these events in the future?

b. Adequate hydration

You are completing a preoperative evaluation on a 46-year-old woman who has recently been diagnosed with gall stones. She is generally healthy, and besides being obese has no chronic medical problems. Given her history, what is the potential surgical complication that would be most likely be lethal for her?

b. Cardiac complications

You are treating a 52-year-old woman with a 40-pack-year history of smoking. She reports a productive cough that has been present for the last 3 to 4 months, beginning in the fall. She remembers having the same symptoms last year in the fall, and attributed it to a "cold that she just couldn't kick." She does not have fevers, reports mild dyspnea when walking up stairs, and denies hempotysis. Which of the following is the most likely diagnosis?

b. Chronic bronchitis

You are seeing a 75-year-old patient with complaints of heartburn, regurgitation, and belching. You suspect GERD. Which symptom, if present, would necessitate a referral for an upper endoscopy?

b. Dysphagia

A 52-year-old generally healthy female patient of yours has recently been diagnosed with breast cancer. She is presenting for presurgical evaluation for her breast biopsy. What is the risk category associated with this surgery?

b. Low risk

You are seeing a patient who is complaining of an itching scalp. There are erythematous papules on her scalp, and you note small black bulbs at the bases of several hair follicles. Which of the following is the preferred treatment option for this condition?

b. Permethrin 1%

You are seeing a 25-year-old male patient with a rash. It began as pink spots on his extremities, but the lesions have begun to coalesce and become purple in color. He recently returned from a hiking trip in the western mountains. Which of the following is the most likely cause?

b. Rocky Mountain spotted fever

A recently retired 67-year-old woman presents to you to establish care. She was a smoker for a long time, but quit 5 years ago. She is generally healthy, but her prior physician told her that she has "emphysema." She was prescribed an "inhaler" to use as-needed and only uses it rarely. She asks about necessary immunizations. Her social history indicates that she lives with her daughter and often cares for her infant granddaughter. Her chart indicates that she had a pneumococcal polysaccharide vaccine at age 63 and a Td shot at age 63. Which of the following vaccines should she receive?

b. Tdap

You are evaluating a 41-year-old man in your office who reports abdominal pain. He says the pain began suddenly and is located in the right lower quadrant. He describes the pain as "gnawing" and it seems to get worse after eating. He has vomited twice since the pain began. Which historical feature would lead you toward an emergent evaluation

b. The fact that the pain began suddenly

A 20-year-old man presents to you 30 minutes after being stung by a bee on his right thigh. He was stung by a bee twice last year. The first sting caused a 3-cm × 3-cm area of erythema, induration, and pain around the sting site. The second sting caused a similar 5-cm × 7-cm area. When you examine him, he has an expanding 2-cm × 2-cm area of erythema, induration, and pain around the sting site on his thigh. He reports pruritis, fatigue, and some nausea, but denies dyspnea. Which of the following is true?

b. This is a large local reaction, and the patient has minimal risk for the development of anaphylaxis upon subsequent exposure.

You are caring for a 45-year-old man with fatigue. Workup revealed hereditary hemochromatosis, an autosomal recessive disorder. Neither of his parents ever showed signs of the disease, though they were never tested while alive. Your patient has one sister. What is the chance that his sister also has hereditary hemochromatosis?

c. 25% chance

You are doing a preoperative clearance for a 60-year-old man undergoing an elective knee replacement. He has diabetes, hyperlipidemia, and a history of a prior myocardial infarction (MI) 4 months ago. After his heart attack, he had triple vessel bypass surgery. Since that time, he has done well and has been asymptomatic from a cardiac standpoint. Which of the following is true in this case?

c. He should have a stress test prior to surgery.

A 56-year-old is complaining of gnawing abdominal pain in the center of her upper abdomen associated with a sensation of hunger. She has a long history of alcohol abuse, and notes darker stool over the last 3 weeks. Which of the following is the most likely cause of her illness?

c. Helicobacter pylori infection

You are caring for a 28-year-old healthy unmarried woman. She is sexually active and has had more than one partner in her past. She had a Pap test 1 month ago that reported "atypical squamous cells of undetermined significance." HPV testing was not done, and you decide to perform colposcopy. The colposcopy did not reveal cervical intraepithelial neoplasia (CIN). Which of the following is the most appropriate next step?

c. Repeat the Pap test in 1 year. .

You are evaluating a 26-year-old woman with fatigue. She also complains of light-headedness and paresthesias in her hands and feet. On examination, her vital signs are normal, but you note pallor and glossitis. Laboratory evaluation reveals a hemoglobin of 9.8 g/dL (L) and an MCV of 102 fL (H). Which of the following would be most likely to treat her condition?

c. Vitamin B 12 supplementation

You are reading a population study that reports 90% of people with lung cancer are smokers. Thirty percent of the people without lung cancer are also smokers. Given this information, what is the specificity using smoking as a predictor of lung cancer?

d. 70%

You find that many of your patients that have gone to the emergency department with chest pain have a negative set of initial cardiac enzymes. Most of those with a negative set of initial enzymes did not have a heart attack. You decide to evaluate 100 of your patients who have gone to the emergency department with chest pain to find out if an initial set of negative enzymes by itself is a good predictor of those that are not having an MI. Of those 100 patients, 20 of them had acute MIs. Of those 20, 10 had a positive set of enzymes initially. Of the 80 that did not have an acute MI, none of them had a positive set of initial enzymes. Given this information, what is the negative predictive value of the initial set of cardiac enzymes in your patient population?

d. 89%

You are evaluating a 75-year-old woman with diabetes and hyperlipidemia who is complaining of chest pain. She reports having occasional chest pain with exertion for years, but yesterday she reported chest pain while walking up steps, then she passed out. On examination, she is afebrile with mildly elevated blood pressure. Cardiac auscultation demonstrates a harsh, rasping crescendo-decrescendo systolic murmur heard best at the second intercostal space at the right upper sternal border. Her carotid pulse is small and rises slowly. Which of the following is the most likely diagnosis?

d. Aortic stenosis

During a routine appointment to discuss an upper respiratory infection, you find that your 18-year-old female patient has become sexually active for the first time. According to current guidelines, when should you begin cervical cancer screening on this patient?

d. At the age of 21

A 44-year-old woman is admitted to the hospital for acute right upper quadrant pain consistent with biliary colic. Her symptoms have been present for 4 hours, and she also has fever and a positive Murphy sign. She has a history of asymptomatic gallstones, identified incidentally several years ago. Her laboratory evaluation is as follows: White blood cell (WBC): 17.5 K/μL (H) with a left shift Aspartate aminotransferase (AST): 88 U/L (H) Alanine aminotransferase (ALT): 110 U/L (H) Alkaline phosphatase: 330 U/L (H) Bilirubin (total): 3.2 mg/dL (H) What would the next test of choice be?

d. Endoscopic retrograde cholangioancreatography (ERCP)

You are counseling a patient who is planning a trip with his wife to celebrate their 30th anniversary. They are going on an African safari, and wonder about health risks associated with international travel. What would you tell him is the most common cause of death among international travelers?

d. Heart disease

You are performing a pre-surgical evaluation on a 44-year-old otherwise healthy African-American male who is undergoing a laparoscopic cholecystectomy. His CBC is shown below: Hemoglobin: 10.6 g/dL (L) Mean corpuscular volume (MCV): 54 fL (L) Red blood cell (RBC) count: 6.3 M/μL (H) Red cell distribution width (RDW): 14.1 (NL) What is the most appropriate step prior to surgery?

d. Hemoglobin electrophoresis

You are caring for a 28-year-old generally healthy woman. She is currently not sexually active, but has had multiple partners in the past. You recently completed her annual examination. Her Pap smear reports "atypical squamous cells of undetermined significance, favor low-grade squamous intraepithelial lesion." Which of the following is the most appropriate next step?

d. Perform colposcopy

You are caring for a 24-year-old generally healthy woman. She is sexually active and currently in a monogamous relationship, using oral contraceptives. You recently completed her annual examination. Her Pap smear reports "atypical squamous cells of undetermined significance." HPV testing was positive. Which of the following is the most appropriate next step?

d. Perform colposcopy.

You are caring for a 34-year-old generally healthy woman. She is sexually active and currently in a monogamous relationship with her husband, using oral contraceptives. You recently completed her annual examination. Her Pap smear reports "atypical glandular cells," but does not specify if those cells are endocervical or endometrial in origin. She has not had any abnormal vaginal bleeding. Which of the following is the most appropriate next step?

d. Perform colposcopy.

You are counseling a patient over the phone who has been taking oral contraceptives regularly for 2 years. Her husband surprised her with a weekend "get away" vacation, and she forgot to bring her pills. She therefore missed taking one active pill. She and her husband had intercourse during their trip, and are not interested in being pregnant at this time. Which of the following is most correct?

d. She should use emergency contraception immediately and restart her pills on the following day.

A 35-year-old woman presents to you concerned about a breast mass. Examination reveals no skin changes, diffusely nodular breasts bilaterally with a more dominant, firm, and nontender fixed nodule on the left side. The nodule is approximately 7 mm in size, in the upper outer quadrant of the left breast. Her mammogram is negative. Which of the following statements is true?

d. The patient should be referred for an ultrasound and possible biopsy.

You are caring for a 30-year-old woman who asks you about the human papillomavirus (HPV) vaccination. She is recently divorced and not in a monogamous relationship. She has a history of genital warts, and had an abnormal Papanicolaou (Pap) test 2 years ago, for which she underwent colposcopy, biopsy, and cryotherapy. Subsequent Pap tests have been normal. Which of the following is true?

d. Vaccination is not recommended for 30-year-old women.

Of the following, which patient would be considered the most appropriate candidate for vaccination against herpes zoster?

e. A 66-year-old man with a personal history of shingles at age 56

You are caring for a person who presents with severe symptoms. He started with fatigue, myalgias, arthralgias, headache, and low-grade fever several weeks ago. He also noted a "rash" on his upper back near the right scapula that looked "like a bull's eye." That rash has since resolved. Currently, he complains of musculoskeletal pain and attacks of joint pain and swelling for the past week, and today he reports pleuritic chest pain. On examination, he has lymphadenopathy, tenderness in his joints, and right axillary adenopathy. You also notice a friction rub. What is the best treatment for this condition?

e. Ceftriaxone intravenously for 2 to 3 weeks

In a routine examination, a 33-year-old woman asks you about breast self-examination as a breast cancer screening method. Which of the following best represents the current American Academy of Family Physicians recommendations regarding breast self-examination (BSE)?

e. Clinicians should not teach their female patients how to perform effective breast self-examinations.

You are caring for a 2 1/2-year-old boy who is coming to your office for the first time. Reviewing his immunization record, you find that he has never received vaccination for invasive pneumococcal disease using the 7-valent pneumococcal conjugate vaccine. Which of the following is true regarding recommendations in his case?

e. He should be vaccinated, but with a modified schedule for immunization.

You are reading a study that compares cholesterol levels in children whose fathers died from an MI with cholesterol levels in children whose fathers died from other causes. The p value obtained in the test was < 0.001. What does this value indicate

e. If the null hypothesis is true, there is a less than 0.1% probability of obtaining a test statistic equal to or more extreme than the one obtained.

You started a 20-year-old woman on COCs 2 months ago. She returns to your office asking to discontinue their use because of side effects. Statistically speaking, which side effect of COCs is most frequently cited as the reason for discontinuing their use?

e. Irregular bleeding

You are seeing a 46-year-old man who reports 3 months of discomfort centered around his upper abdomen. It is associated with heartburn, frequent belching, bloating, and occasional nausea. What is the most likely result that will be found after workup for these symptoms?

e. No cause is likely to be identified

You are evaluating a patient new to your practice who is complaining of abdominal pain. The pain has been present on and off for more than 2 years, and has been present more often than not for the preceding 6 months. She reports that her pain is related to defecation and is associated with diarrhea. Which of the following is true regarding diagnostic testing for her condition?

e. No tests are necessary to diagnose this condition.

A 43-year-old woman with a history of well-controlled hypertension and diabetes presents to your office complaining of intermittent chest pain for the last 3 months. The last episode was 1 week ago, after climbing four flights of stairs at work. The pain was relieved with rest. An ECG in your office is shown below. She is currently asymptomatic. Which of the following is the most appropriate next step?

e. Obtain the patient's treadmill stress echocardiogram.

You are caring for a 46-year-old generally healthy woman. She is sexually active with her husband only. You recently completed her annual examination. Her Pap smear reports "atypical glandular cells" and are reported to be of endometrial origin. She does not report any abnormal vaginal bleeding. Which of the following is the most appropriate next step?

e. Perform endometrial biopsy.

You are seeing a healthy 26-year-old woman for a routine health visit. She mentions that she and her husband are thinking about starting a family soon. She has never been pregnant before. Which of the following interventions, if done prior to pregnancy, has been shown to have a clear beneficial outcome for this woman and her potential child?

e. Prescribing 0.4 to 0.8 mg of folic acid daily

A 29-year-old obese woman with type 2 diabetes mellitus is asking you about progestin-only pills as a method of contraception. Which of the following is true?

e. Progestin-only pills should be taken every day of the month, without a hormone-free period.

You are caring for a 23-year-old healthy homosexual male who works as an accountant and lives alone. He had the "typical childhood vaccinations" and provides documentation of his immunization record. He is up-to-date on tetanus, and was primarily immunized against diphtheria, pertussis, polio, hepatitis B, measles, mumps, rubella, andH influenzae type b. Which of the following vaccinations is indicated for this patient?

Hepatitis A

You are examining a normal-term newborn whose mother is hepatitis B virus (HBV) surface antigen positive. Which of the following protocols is recommended for the child?

Hepatitis B vaccination and hepatitis B immune globulin within 12 hours of birth, a second dose of hepatitis B vaccine at 1 to 2 months, and a third dose of vaccine at 6 months.

You are seeing a 55-year-old patient for her annual physical examination. She has been married to her husband for 32 years and reports that both have been monogamous. Records indicate that she has had normal Pap smears every 1 to 2 years for the last 20 years, and has never had an abnormal Pap smear. At what age is it appropriate to discontinue Pap screening on this patient?

a. 55 years

You are caring for a young family who just had a child with multiple malformations of unknown etiology. What type of testing would be best for identifying the diagnosis?

a. Cytogenetic analysis

You are reevaluating a 32-year-old woman in your office. You started her on combination oral contraceptives (COCs) 3 months ago, and at each of three visits since then, her blood pressure has been elevated. Which of the following is the most appropriate next step?

a. Discontinue the oral contraceptive and recommend a barrier method.

You are caring for a 38-year-old male patient who reports episodic chest pain. He reports that the pain feels like "tightness," is located right behind his sternum, lasts less than 3 minutes, and is relieved with rest. He takes no medications, has no family history of coronary disease, and has never smoked. His ECG in the office is normal. Which of the following tests should be done to determine whether or not his chest pain is due to ischemia?

a. Exercise ECG


Ensembles d'études connexes

Finance 3325 Final Exam Review (Test 2)

View Set

GIVE ME LIBERTY! By Eric Foner Chapter 24

View Set

Unit 2 "Writing and Testing Code", Unit 3 too

View Set

APUSH chapter 13 test multiple choice

View Set

Powerpoint 2019/365 - Concept Review 1

View Set

NUR326 Mental Health Medications

View Set

Chapter 11, The Health Care Delivery System

View Set

Chapter 14: Environmental Liability Insurance

View Set